Vous êtes sur la page 1sur 65

ADrPlexus Digital Strides 2016

NEET PG MOCK 4
The conduction velocity of an action potential along an
1. A young librarian has been exceedingly shy and axon is proportional to the axon's diameter for both
fearful of people since childhood. She longs to make unmyelinated and myelinated axons. The resting
friends, but even casual social interactions cause her a membrane potential, the duration of the relative refractory
great deal of shame and anxiety She has never been at period, and the magnitude of the action potential are
a party, and she has requested to work in the least dependent on the type and density of electrically excitable
active section of her library, even though this means gates and the ability of the Na to establish and maintain
lower pay She cannot look at her rare customers the concentration gradients. These characteristics are not
without blushing, and she is convinced that they see related in any systematic way to the axon diameter.
her as incompetent and clumsy. Which anxiety
disorder is most likely to be confused with this 4. The characteristic of a water-insoluble substance
personality disorder? most important in governing its diffusibility through a
1. Generalized anxiety disorder cell membrane is its.
2. Specific phobia 1. Hydrated diameter
3. Agoraphobia 2. Molecular weight
4. Social phobia 3. Electrical charge
4. Lipid solubility
Ans. 4
A voidant personality disorder can be difficult to Ans. 4
differentiate from social phobia. One differentiating factor Materials that are not soluble in water can only cross the
is that in social phobia specific situations, rather than membrane through the lipid bilayer. The most important
interpersonal contact, cause distress and are avoided. factor determining how well a substance can diffuse across
the lipid bilayer is the substance's lipid solubility If two
materials have the same lipid solubility, then the
2. Melatonin permeability of the smaller particle will be greater.
1. Is synthesized in the anterior pituitary gland
2. Regulate skin pigmentation in humans 5. A couple comes to you because they have been
3. Secretion is increased by darkness unable to conceive even after 1 year of trying. You
4. Secretion is inhibited by norepinephrine from the examine the man and notice a darkish mass and
sympathetic nervous system fullness of the left scrotum / spermatic cord compared
to the smaller right scrotum / spermatic cord. You
Ans. 3 suspect which of the following?
Synthesis and secretion of melatonin are increased in the 1. Undiagnosed cryptorchidism of the right testicle
dark via input. from norepinephrine secreted by 2. Acquired left sided varicocele
postganglionic sympathetic neurons. Melatonin is 3. Acquired left femoral hernia
synthesized in the pineal gland from the amino acid 4. Acquired right direct femoral hernia
tryptophan. Pinealomas (tumors of the pineal gland) that
destroy the pineal gland and reduce secretion of melatonin Ans. 2
and cause hypothalamic damage may cause precocious A dark mass within the scrotum would most likely be one
puberty by removing the inhibitory effect of melatonin on of two things: varicocele or indirect inguinal hernia.
the pituitary response to gonadotropin releasing hormone. Varicoceles are a stasis of blood within the pampiniform
Melatonin causes amphibian skin to become lighter in plexus and generally occur most frequently on the left side
color but has no role in the regulation of skin color in because the testicular vein on the left drains into the
humans. higher pressure left renal vein whereas the right testicular
vein drains into the inferior vena cava. The presence of a
3. Which of the following characteristics of an axon is varicocele is associated with reduced fertility
most dependent on its diameter?
1. The magnitude of its resting potential 6. Which of the following statements concerning
2. The duration of its refractory potential erection, emission, and ejaculation in the male is
3. The conduction velocity of its action potential correct?
4. The overshoot of its action potential 1. Contraction of the internal urethral sphincter is under
control of the parasympathetic nervous system.
Ans. 3 2. The parasympathetic nerves stimulate closure of
helical arteries

1
3. Sympathetic neurons stimulate the helical arteries to the pelvic brim to the lateral pole of the ovary. It contains
dilate and increase blood flow to the corpora the ovarian artery. ovarian vein, ovarian lymphatics. and
cavernosum ovarian nerves (ovarian neurovascular bundle). Volvulus
4. Contraction of the bulbospongiosus muscles impedes of the ovary (usually associated with an ovarian tumor)
the drainage of blood from the corpus spongiosum. may constrict the neurovascular bundle with marian
infarct and pain referred to the inguinal and hypogastric
Ans. 4 regions.
The bulbospongiosus muscle is innervated by the
pudendal nerve (S2-S4) and its contraction helps to keep
blood within the shaft of the penis. Contraction of the 9. Rakesh, 30-yrs has marked swelling on palmar
internal urethral sphincter is under control of the aspect of the wrist. Persistent flexion of fingers and
sympathetic nervous system. 'Con-comitant with dilation apparent shortening of middle finger is seen. There is
of the helicine arteries under parasympathetic innervation, paresthesia over palmar aspect of the thumb, index,
which allows increased blood to flow into the cavernous middle, and a questionable portion of the ring finger,
spaces, contraction of the bulbospongiosus and yet when wrist is gently flexed, intense pain spreads
ischiocavemosus muscles at the base of the cavernous over this area. Sensation over palm is normal. Partial
bodies reduce blood from leaving, resulting in flexion of fingers in this case is best explained by
engorgement and penile or clitoral erection. Emission of which of the following?
seminal fluid, prostatic secretions, and sperm from the vas 1. Compression of the radial artery
deferens is due to contraction of smooth muscle under 2. Compression of the recurrent bra ich of the median
sympathetic control. nerve
3. Impingement of the flexor tendons by a dislocated
7. In a clinical trial, two drugs A and B were carpal bone
administered to alternate patients in 100 cases of 4. Paralysis of the dorsal interossei muscles
hypertension and the effect of these two drugs was
studied statistically by applying chi square test. The Ans. 3
value of chi square was 4.12 with degree of freedom A fall on the extended hand will frequently dislocate the.
equal to one against the table value of 3.84 at 5% level. lunate bone anteriorly This dislocated bone may then
Which of the following conclusions can be drawn from impinge on the tendons of the extrinsic' digital flexor
this study? muscles and thereby prevent flexiori of the fingers.
1. Null hypothesis is proved Compression of the median nerve in the carpal tunnel
2. There is no significant difference between the effect of cannot explain this observation because the prime flexors
two drugs of the digits are the extrinsic flexors (flexors digitorum
3. Null hypothesis is rejected superficialis and profundus). which receive their
4. The probability of the effect of the two drugs beings innervation in the forearm. well proximal to the injury. The
the same are more than 0.05 dorsal interossei. innervated by the ulnar nerve, are digital
extensors. The recurrent branch of the median nerve
Ans. 3 innervates the thenar muscles.
The null hypothesis in this trial is that the two drugs A and
B are equally effective. The calculated chi-square value of 10. Paresthesia at anterolateral region of the thigh
4. I 2 at one degree of freedom on comparison with the may occur in obese persons. It results from an
table value of 3.84 indicate the statistically significant abdominal panniculus adiposus that bulges over the
difference between the drug effectiveness. inguinal ligament and compresses which underlying
This also means that the null hypothesis of equality is nerve?
rejected at 5% level of significance or P < 0.05. 1. Femoral branch of the genitofemoral nerve
2. Femoral nerve
8. Which of the following contains the ovarian 3. Iliohypogastric nerve
Neurovascular bundle? 4. Lateral femoral cutaneous nerve
1. Broad ligament
2. Mesosalpinx Ans. 4
3. Mesovarium The lateral femoral cutaneous nerve passes beneath the
4. Suspensory ligament inguinal ligament just medial to the anterior superior iliac
spine. It innervates the lateral aspect of the thigh. The
Ans. 4 iliohypogastric nerve innervates a portion of the gluteal.
The mesosalpinx. mesovarium. and suspensory ligament Inguinal, and pubic regions. The ilioinguinal nerve and the
are all continuous with the broad ligament .. which is a femoral branch of the genitofemoral nerve supply the
reflection of peritoneum over the female reproductive upper portions of the anterior thigh. The sensory
organs. The mesovarium attaches the ovary to the broad distribution of the femoral nerve is the anterior thigh and
ligament. The suspensory ligament of the ovary runs from medial leg.

2
11. Abdominal wall structures that usually contribute Ans. 4
directly to the spermatic cord include: The venous drainage from each adrenal gland tends to be
1. External oblique muscle through a single vein. The left adrenal gland usually drains
2. Falx inguinalis into the left renal vein superior to the point where the
3. Internal oblique muscle gonadal vein enters the left renal vein. The left adrenal
4. Rectus sheath vein usually anastomoses with the hemizygous vein and
may provide an important route of collateral venous
Ans. 3 return. The right adrenal gland usually drains directly into
Several abdominal structures are involved in the the inferior vena cava.
formation of the spermatic cord. The deep fascia
contributes the external spermatic fascia. The cremaster 14. Failure of the sixth aortic arch arteries to form
muscle, a contribution of the internal oblique muscle. joins would lead to loss of blood supply to which of the
spermatic cord as the inguinal canal passes through that following?
layer. The transverses abdominis muscle, which usually 1. Right side of the heart
terminates as the falx inguinalis just superior to the deep 2. Face
ring. contributes to the cremaster muscle in than 5% of all 3. Thyroid gland
males. The transversalis fascia contributes the internal 4. Lungs
spermatic fascia.
Ans. 4
12. Rekha, a 35-year consults an ophthalmologist Branches of the arteries of the sixth aortic arches form the
because of double vision and droopy eyelids. She also pulmonary arteries, In addition the left sixth arch artery
has complaints of generalized muscle weakness. IV forms the ductus arteriosus. The blood supply to the right
injection of edrophonium dramatically, but only side of the heart is primarily derived from the right and
briefly, reverses her symptoms. This patient's left coronary arteries and are derived from the right and
probable disease has a pathophysiologic basis that is left coronary arteries and are derived from the truncus
closest to that of which of the following conditions? arteriosus.
1. Bullous pemphigoid The face and thyroid gland receive blood primarily from
2. SLE the facial and superior thyroid arteries, respectively These
3. Idiopathic Addison disease are branches of the common and external carotid arteries
4. Insulin resistance which, in turn, are derivatives of the second and third
aortic arch arteries.
Ans. 4
This patient has myasthenia gravis, which was suspected 15. Regarding the "pump handle" movement during
based on the woman's clinical presentation and confirmed respiration, which statement is correct?
with the response to the short-acting antichol in esterase 1. There is decrease in the anterior -posterior diameter
drug edrophonium. Myasthenia gravis is an autoimmune of the chest
disease in which antibodies directed against the 2. No movement occurs at the costovertebral joints
acetylcholine receptor of the muscle side of the 3. There is an increases in the superior -inferior
neuromuscular junction block the ability of the receptor to diameter of the chest
bind to acetylcholil1l.:. Of the diseases listed above. only 4. Movement occurs at the sternomanubrial joint
insulin resistance IS produced by a similar mechanism, i.e.
antibodies to insulin receptors block the receptors' ability Ans. 4
to bind to insulin. Bullous pemphigoid is caused by Contraction of the inter costal muscles causes rotation of
antibodies directed against the basement membrane of the the costovertebral joints and elevation of the sternal ends
skin, which damage the basement membrane and cause of the upper (2 to 6) ribs. Along with slight movement of
blister formation. the sternomanubrial joint, particularly in the young, this
Addison diseases are characterized by humoral, and "pump-handle movement" increases the anteroposterior
probably cell-mediated. reactions against the cells in the (AP) diameter of the chest. The transverse diameter of the
endocrine tissue. thoracic cavity increases when contraction of the
SLE has circulating and locally generated immune intercostal muscles also elevates the mid portion of the
complexes mediating the pathophysiology. ribs (bucket-handle movement). Contraction of the
diaphragm increases the vertical diameter of the thoracic
13. Normally, the left adrenal venous drainage is into cavity
which of the following?
1. Inferior vena cava 16. Which of the following hormones interacts with a
2. Left azygos vein cytoplasmic receptor and then localizes in the nucleus
3. Left inferior phrenic vein and directs protein and nucleotide synthesis?
4. Left renal vein 1. Thyrotropin - releasing hormone

3
2. Epinephrine can contribute to the generation of tension. However, the
3. LH transmission of force from the cross bridges to the tendon
4. Cortisol (or bone or measuring device)" does not occur until the
series elastic component (SEC) of the muscle is stretched.
Ans. 4 Repetitive firing increases the amount of SEC stretch by
Cortisol, like other steroid hormones, diffuses into target maintaining cross-bridge cycling for a longer period of
cells and interacts with intracellular receptors. The time. Repetitive firing increases neither the concentration
steroid-receptor complex has a high affinity for the of Ca2+ within the myoplasm, the number of myofibrils that
steroid- responsive element of DNA. 'Once bound to DNA, are activated, nor the magnitude of the end-plate potential.
the hormone-receptor complex acts as a transcription Because all of the cross bridges are activated each time a
factor to regulate gene expression and formation of skeletal muscle fiber is activated, an increase in Ca2+
specific messenger RNAs. concentration would have no effect on muscle strength.

17. Secondary suturing of third degree perineal tear: 19. The alpha rhythm appearing on an
1. Should be done two weeks after delivery electroencephalogram has which of the following
2. Should be done one month after delivery characteristics?
3. Should be done three months after delivery 1. It produces 20 to 30 waves per second
4. Can be done any time after delivery 2. It disappears when a patient's eyes open
3. It is replaced by slower, larger waves during REM
Ans. 3 sleep
Secondary suturing of third degree perineal tear should 4. It is associated with deep sleep
be done three months after delivery.
In third degree perineal tear there is tear of perineal body Ans. 2
involving the sphincter ani extern us with or without In a totally relaxed adult with eyes closed, the major
involvement of the anorectal mucosa. It is also called component of the electroencephalogram (EEG)' will be a
complete perineal tear. In first degree tear there is regular pattern of 8 to 12 waves per second, called the
laceration of remnants of hymen, the fourchette, lower alpha rhythm. The alpha rhythm disappears when the eyes
part of vagina and the perineal skin but the perineal body are opened. It is most prominent in the parieto-occipital
is intact. In second-degree tear there is laceration of region. In deep sleep, the alpha rhythm is replaced by
posterior vaginal wall and varying degree of tear of larger, slower waves called delta waves. In REM sleep, the
perineal body excluding the anal sphincter. A third degree EEG will show fast, irregular activity
perineal tear is called old when passed beyond an
arbitrary period of 3 month following the injury. The main 20. A young woman had a repeat lower segment
cause of third degree perineal tear' is mismanaged 2nd caesarean section at term. On the sixth postoperative
stage of labour. day her clothes were soaked by serosanguineous
discharge from the abdominal wound. The most likely
Treatment: diagnosis is:
The definitive surgery is repair of the perineal tear with 1. Vesico-cutaneous fistula
restoration of perineal body. It should be done between 3- 2. Uretero-cutaneous fistula
6 months after the injury. However the best I time is 3. Faecal fistula
within 24 hours of injury. By three months the local 4. Wound dehiscence
infection subsides and the general condition of the patient
improves and if kept more than 6 months' then there more Ans. 4
fibrosis on the margins. Surgery is done by flap method. Discharge of serosanguinous. fluid indicates wound
dehiscence. In vesicocutaneous or urethrocutaneous is
18. Repetitive stimulation of a skeletal muscle fiber fistula there is discharge of urine from the fistula. In faecal
will cause an increase in contractile strength because fistula there is discharge of faecal material from the fistula.
repetitive stimulation causes an increase in which of
the following? 21. A 29-yrs-old man is brought to the emergency
1. The duration of cross-bridge cycling room after a traffic accident causing a traumatic brain
2. The concentration of calcium in the myoplasm injury. Within several hours he begins eating objects
3. The magnitude of the end-plate potential such as paper, is unable to maintain attention, and
4. The number of muscle myofibrils generating tension displays increased sexual activity. This occurs due to
bilateral lesions of which of the following regions of
Ans. 1 the brain?
Each time a skeletal muscle fiber is stimulated by an alpha 1. Temporal lobe
moto neuron, enough Ca2+ is released from its 2. Hypothalamus
sarcoplasmic reticulum (SR) to fully activate all the 3. Olfactory lobe
troponin within the muscle. Therefore, every cross bridge 4. Hippocampus

4
CORRECT 4. She most likely has osteoporosis of bone with
Ans. 1 accelerated bone loss leading to the propensity for
The Kluver-Bucy syndrome is produced in animals by fractures. Physical inactivity further accelerates bone loss
removal of the amygdala from both temporal lobes. The and decreases muscle mass and agility that contribute to
syndrome is characterized by a tendency to examine falls.
objects orally and excessive sexual behavior. The full Incorrect 1. A poor diet could be lacking in vitamin D,
syndrome is rarely encountered in humans but many of its though endogenous vitamin D production occurs with
characteristics are observed in patients with bilateral sunlight exposure. The findings are those of osteomalacia
'temporal lobe lesions produced by encephalitis or and are similar clinically to osteoporosis.
traumatic injury Incorrect 2. An osteomyelitis is not typical at this age and
does not usually present as fractures in multiple locations.
22. An otherwise healthy 44-year-old man with no Incorrect 3. This inherited disorder of collagen synthesis is
prior medical history has had increasing back pain initially diagnosed in fetuses and young children.
and right hip pain for the past decade. The pain is
worse at the end of the day. On physical examination 24. A 10-yrs-old boy displays hyperactivity and is
he has bony enlargement of the distal interphalangeal unable to focus on his schoolwork because of an
joints. A radiograph of the spine reveals the presence inability to focus on the activity. Which of the following
of prominent osteophytes involving the vertebral drugs might prove effective for relieving the boy's
bodies. There is sclerosis with narrowing of the joint main symptoms?
space at the right acetabulum seen on a radiograph of 1. Methylphenidate
the pelvis. Which of the following diseases is he most 2. Terbutaline
likely to have? 3. Dobutamine
1. Gout 4. Pancuronium
2. Rheumatoid arthritis
3. Osteoarthritis Ans. 1
4. Osteomyelitis Methylphenidate is similar to amphetamine and acts as a
CNS stimulant, with more pronounced effects on mental
Ans. 3 than on motor activities. It is used to treat narcolepsy and
CORRECT 3. Degenerative osteoarthritis is a common and attention-deficit hyper activity disorders.
progressive condition that becomes more frequent and
symptomatic with aging. There is erosion and loss of 25. Decubitus ulcer in a case of prolapse uterus occurs
articular cartilage with joint space narrowing. There is mainly due to:
minimal inflammation. 1. Infection
Incorrect 1 gouty arthritis is more likely to be 2. Friction
accompanied by swelling, and deformity with joint 3. Venous congestion
destruction. The pain is not related to usage. 4. Malignant change
Incorrect 2. Rheumatoid arthritis typically involves small
joints of the hands and feet most severely, and there is a Ans. 3
destructive pannus that leads to marked joint deformity. Decubitus ulcer in prolapsed uterus is due to venous
Incorrect 4. Osteomyelitis represents an ongoing infection congestion. Friction does not play any major role. Though
that produces marked bone deformity, not just joint in 13th edition of Shaw friction is mentioned as a cause but
narrowing. in previous edition the main emphasis has been placed on
venous congestion. Please read the earlier editions for
23. An 80-year-old woman has had no major medical details.
problems, but she has never been physically active for
most of her life. One day she falls out of bed and 26. We administer hexamethonium to a subject. He is
immediately notes a sharp pain in her left hip. She is in the supine position. Which of the following
subsequently unable to ambulate without severe pain. responses would you expect in response to this drug?
Radiographs show not only a fracture of the left 1. Bradycardia
femoral head, but also a compressed fracture of T10. 2. Increased GI tract motility
Which of the following conditions is she most likely to 3. Increased salivary secretions
have? 4. Vasodilation
1. Vitamin D deficiency
2. Acute osteomyelitis Ans. 4
3. Osteogenesis imperfect Hexamethonium is an autonomic ganglionic blocking drug.
4. Osteoporosis So by blocking neurotransmission across all autonomic
ganglia (and activation of the adrenal medulla), we
Ans. 4 essentially denervate distal structures. Now we observe
how things change.

5
For the structures and functions listed (and several others increase the strength but diminish the frequency of uterine
that weren't), it is the parasympathetic nervous system contraction, Rest of three statements are correct.
that exerts the predominant resting tone. Only control of.
vascular smooth muscle tone (and, therefore, of blood 29. An SSRI antidepressant (eg. Fluoxetine) will be
pressure) is primarily regulated by the sympathetic prescribed for an adult patient. You should advise him
nervous system. or her that two most likely side effects or adverse
Block the ganglia and vasodilation will occur as responses that may eventually occur at therapeutic
predominant SNS influences are removed. As far as the blood levels are which of the following?
other structures go-all of which are mainly influenced by 1. Intense dizziness upon standing and palpitations.
the PNS at rest-we would observe a rise of heart rate, 2. Migraine headache and involuntary skeletal muscle
decreases of bladder and gut tone (e.g. reduced tone of the twitching
bladder detrusor and longitudinal muscles of the gut), 3. Polyuria and insatiable thirst
mydriasis, and, reduced salivary secretions (xerostomia). 4. Sexual dysfunction and weight gain.
Sweat gland secretions, which also have pre dominant
sympathetic control, would decrease too. Ans. 4
Sexual dysfunction (probably of CNS origin) and weight
27. A 26-yrs-old woman with reactive depression gain (sometimes significant) are relatively common
complains of missing her period and having milk problems associated with SSRI therapy Orthostatic
discharge from her breasts. Pregnancy tests are hypotension and . reflex tachycardia sometimes occur with
negative. Which of the following is most likely to have older antidepressants (tricyclics, e.g. imipramine), but are
caused these findings? rare with SSRIs. Migraine is not likely; blockade of
1. Amoxapine serotonin reuptake might actually reduce the .risk (note
2. Clomipramine that triptans exert antimigraine actions by acting as
3. Fluoxetine serotonin agonists). Lithium is the drug likely to cause
4. Sertraline polyuria and polydipsia. None of the other side effects or
adverse responses listed in the question are likely with an
Ans. 1 SSRI.
Amoxapine a tricyclic antidepressant, effectively , blocks
neuronal norepinephrine reuptake, is somewhat less 30. Na+ is reabsorbed from the basolateral surface of
potent in terms of blocking serotonin uptake, and is far the renal epithelial cells by which of the following?
less potent in terms of blocking dopamine reuptake. 1. Na/H exchange
A metabolite of amoxapine blocks dopamine receptors 2. Na-glucose cotransport
quite well, and that effect makes it the most likely choice as 3. Na-K pump
the drug that caused the patient's menstrual irregularities 4. Facilitated diffusion
and galactorrhea. You should recall that such drugs as the
phenothiazine, which exert their main antipsychotic Ans. 3
effects via blockade of dopamine receptors, may also cause The intracellular Na concentration of renal epithelial cells
amenorrhea galactorrhea by the same mechanism, and is pumped out of renal epithelial cells by Na- K pumps
drugs that activate dopamine receptors (e.g., located on the basolateral surface of the epithelial cells.
bromocriptine) can be used to manage this endocrine The Na/H exchanger and the Na- glucose transporter are
dysfunction. Amoxapine's dopamine receptor blockade located on the apical surface of the epithelial cells. Na + is
also seems to account for why the drug exerts some transported from the peritubular spaces to the capillaries
antipsychotic properties, theoretically making it useful for by solvent drag.
patients with both psychosis and depression.
31. Glomerular filtration rate would be decreased by
28. All of the following physiological or biochemical which of the following?
effects require progesterone except: 1. Constriction of the efferent arteriole
1. Stimulation of myometrial contraction 2. An increase in afferent arteriolar pressure
2. Promotion of differentiation and growth of lactiferous 3. Compression of the renar capsule
ducts 4. A decrease in the concentration of plasma protein
3. Formation of cortisol by foetal adrenal cortex in early
pregnancy Ans. 3
4. Formation of aldosterone by the foetal adrenal cortex GFR will decrease is there is a decrease in the net
in early pregnancy glomerular capillary pressure or the flow of fluid through
the glomerulus. The net glomerular capillary pressure (for
Ans. 1 Starling forces) is equal to the glomerular capillary
Progesterone inhibits myometrial contraction. pressure minus the sum of the plasma oncotic pressure
Pregesterone relaxes smooth muscles and hence relax and intrarenal pressure. Compression of the renal capsule
uterus in pregnancy. It causes myohyperplasia and hence increases the intrarenal pressure and therefore decreases

6
the net capillary filtration pressure. Constriction of the
efferent arteriole increases glomerular capillary pressure. 35. Drug Causing anaphylactoid reaction:
Decreasing the concentration of plasma protein will 1. Propofol
decrease the plasma oncotic pressure and lead to an 2. Alcuronium
increase in GFR. 3. Thiopentone
4. Glycopyrrolate
32. Mendelson syndrome is due to:
1. Aspiration pneumonitis Ans. 2
2. Chemical pneumonitis Development of true allergy or antibody formation may
3. Esophagitis of course occur in days or weeks following exposure to any
4. Esophageal spasm of non depolarising muscle relaxants (i.e. ALCURONIUM)
They also cause TACHYPHYLAXIS.
Ans. 1
Mendelson's syndrome is acid aspiration syndrome. 36. Which of the following is a function of cells
Aspiration of acid gastric contents cause a chemical Labelled A ?
trauma to bronchial and alveolar epithelia i.e. acute
exudative pneumonitis
Mendelson's. syndrome usually occur with material
(gastric acid) at a pH of 2. 5 or below, but known. to occur
with fluid of a neutral r" as well.

33. The supraoptic nucleus of the hypothalamus is


believed to control secretion of which of the following
hormones?
1. Antidiuretic hormone
2. Oxytocin
3. Growth hormone
4. Adrenocorticotropic hormone
1. Secretion of FSH into the tubular lumen
Ans. 1 2. Secretion of testosterone into the tubular lumen
Secretion of antidiuretic hormone (ADH) and oxytocin by 3. Maintenance of the blood-testis barrier
the neurohypophysis is regulated in the hypothalamic 4. Synthesis of estrogen after puberty
supraoptic and paraventricular nuclei, respectively. This
hypothalamic control of secretion of pituitary hormone Ans. 3
(inhibitory as well as releasing) in the case of the
neurohypophysis is by direct neural connection, and, in the Of course If you focus on the Picture without reading
case of the adenohypophysis, by humoral factors conveyed the options you cannot guess the StructureSo
bya microcirculation known as the hypothalamic- Whenever you are not sure about the HISTOLOGY
hypophyseal portal system. Picture, read the options and almost 99% you can get
to know the structure
34. Which of the following is the principal steroid Picture : Structure of Seminiferous tubule
secreted by the fetal adrenal cortex?
1. Cortisol
2. Corticosterone y
3. Dehydroepiandrosterone
4. Progesterone

Ans. 3
Because it lacks 3-dehydroxysteroid, the enzyme that
converts pregnenolone to progesterone (the initial step in
both glucocorticoid and mineralocorticoid synthesis), the
fetal cortex synthesizes primarily
dehydroepiandrosterone. This steroid is released as its
sulfate and is metabolized further to estrogen and
androgen by the placenta. During fetal life, the adrenal
cortex consists of a thin subcapsular rim, which eventually
gives rise to the adult cortex, and a thick inner fetal cortex,
which constitutes 80% of the gland. This zone undergoes
rapid involution after birth.

7
The Sertoli cells rest on a basal lamina and form a layer Ans. 4
around the periphery of the seminiferous tubules. They are There is substantial protection against ischemia and
attached to each other by specialized junctional complexes hypoxia is provided by. just 1-3C hypothermia.
that limit the movement of fluid and solute molecules from Hypothermia reduces the tissue metabolic rate about
the interstitial space and blood to the tubular lumen, and 8%/OC.
thus form a blood testis barrier that provides an It decreases the cerebral metabolic rate and is
immunologically privileged environment for sperm cerebroprotective during episode of cerebral ischemia The
maturation. Sertoli cells are intimately associated with protection afforded by mild hypothermia is so great that
developing spermatozoa and play a major role in germ-cell reduced core temperature 34 probably indicated in
maturation. They secrete a variety of serum proteins and - Carotid artery surgery
an androgen protein into the tubular fluid in response to - Neurosurgery
FSH and testosterone stimulation. Testosterone is - Procedures where tissue ischemia can be
synthesized and secreted by the interstitial Leydig cells. anticipated
Estrogen is produced in small amounts by the Sertoli cells - Traumatic brain injury
before puberty. - ARDS
- Malignant hyperthermia .
37. Astigmatism is: Hypothermia is used in cardiac surgery as cooling does
1. Defect in curvature slow heart and greatly depresses body metabolism .
2. Defect in thickness of cornea Dysrhythmia occurs at temperature below 30C,
3. Defect in refractive index spontaneous ventricular fibrillation may be seen but is not
4. Defect in opacity of lens likely above 28C. So hypothermia is arrhythmogenic.

Ans. 1 40. Which of the following receptors is responsible for


Astigmatism It is that condition of refraction wherein a measuring the intensity of a steady pressure on the
point focus of light cannot be formed' upon the retina. It skin surface?
includes those anomalies in the optical system wherein an 1. Pacinian corpuscle
appreciable error is caused by the unequal refraction of 2. Ruffini ending
light in different meridians. Hence it may be an error 3. Merkel's disk
either of curvature, of centering or of refractive index. 4. Meissner's corpuscle
There fore this question is controversial as both the
options are given. But since the commonest form of Ans. 2
astigmatism is due to curvature defect, this will be the The Ruffini ending is a tonic receptor that produces a train
answer. of action potentials proportional to the intensity of
pressure applied to the skin. The Pacinian corpuscle is a
38. A 10-yrs-old girl with type I diabetes develops a very rapidly adapting receptor that fires once or twice in
neuropathy limited to sensory neurons with free response to skin deformation. It can produce a continuous
nerve endings. Quantitative sensory testing will reveal train of action potentials if the stimulus is repetitively
higher-than-normal thresholds for the detection of applied and withdrawn. Therefore, the Pacinian corpuscle
which of the following? is used to encode vibration.
1. Fine touch
2. Vibration 41. Inert particles are sensitized with either antigen or
3. Pressure antibody. Which of the following tests is used
4. Temperature extensively to detect microbial antigens rapidly (five
minute or less)?
Ans. 4 1. Coagglutination (COA)
Free nerve endings contain receptors for temperature, 2. Counter immune electrophoresis (CIE)
pain, and crude touch. However, fine touch, pressure, and 3. Enzyme-linked immunosorbent assay (ELISA)
vibration are detected by nerve .endings contained within 4. Latex agglutination
specialized capsules that transmit the stimulus to the
sensory receptors. Muscle length is encoded by the Ans. 4
primary' nerve endings of Ia fibers, which are located on Of the many methods available for antigen and antibody
intrafusal fibers within the muscle spindle. detection, LA, ELISA, RIA, CIE, and COA are the most widely
used. Latex agglutination (LA) employs latex polystyrene
39. Hypothermia is used in all except: particles' sensitized by either antibody or anti gen. LA is
1. Neonatal asphyxia more sensitive than CIE and COA but slightly less sensitive
2. Cardiac surgery than either RIA or EIA. LA has been used to detect H.
3. Hyperthermia influenzae, Neisseria meningitidis, and S. pneumoniae
4. Arrythmia antigens in cerebrospinal fluid. LA has also been used for
detection of cryptococcal antigen. Most recently, LA has

8
been widely used for rapid detection of group. A 3. Antibiotics
streptococcal antigen directly from the pharynx. The test is 4. Laser iridotomy
rapid (five minutes), sensitive (approximately 90%), and
specific (99%). Ans. 4

42. The obstetric conjugate diameter is the distance 45. Which one of the following aids is helpful in the
between the sacral promontory and the: diagnosis of bowel strangulation?
1. Nearest point on the posterior surface of the 1. Marked abdominal distension
symphysis pubis 2. Persistent local tenderness
2. Upper part of the inner surface of the symphysis pubis 3. Profuse vomiting
3. Apex of the pubic arch 4. Multiple air fluid levels on X-ray
4. Ilio-pectineal eminences
Ans. 2
Ans. 1 Steady severe pain without quiescent periods is usually
Obstetric conjugate is the distance between the sacral indicative of strangulation.
promontory and the .nearest point on the posterior surface In non strangulated obstruction of intestine there ma be an
of the symphysis pubis. area of localized tenderness at the site of obstruction; in
strangulation there is always localized tenderness
Important points: associated with rigidity/rebound tenderness.
1. Obstetric conjugate measures 10 cm, It cannot be Rest of three conditions may be present in both
clinically measured but is to be inferred from diagonal obstruction as well as strangulation. The presence of shock
conjugate or by lateral radiopelvimetry. indicates ischaemia of bowel. In strangulation pain is
2. True/anatomical conjugate is the distance between never completely absent.
midpoint of sacral promontory to the inner margin of CT. Remember the following important points about intestinal
upper border of symphysis pubis. It measures 11 cm, Its obstruction and strangulation:
measurement is inferred by subtracting 1.2 cm from ft - Four cardinal symptoms and signs of intestinal
diagonal conjugate. obstruction are crampy abdominal pain, vomiting,
3. Diagonal conjugate is the distance between the lower obstipation and abdominal distention. The finding of
border of symphysis pubis to the midpoint on the s localized tenderness, fever, tachycardia and leukocytosis are
promontory. It is measures 12 cm. It is measured clinically supportive of: but not diagnostic for strangulation.
during pelvic assessment.
4. Obstetric conjugate can be calculated by subtracting 1.5- 46. Which statement regarding transmission of viral
2 cm from the diagonal conjugate. illness through homologous blood transfusion is true?
1. The most common viral agent transmitted via blood
43. Elevated IgG and IgM antibody titers to parvovirus transfusion is HIV
suggest a diagnosis of which one of the following? 2. Blood is routinely tested for CMV because CMV
1. Acute Lyme disease infection is often fatal
2. Fifth disease 3. The most frequent infectious complication of blood
3. Possible hepatitis B infection transfusion continues to be viral meningitis
4. Possible subacute sclerosing pan encephalitis 4. Up to 10% of those who develop post transfusion
hepatitis will develop cirrhosis or hepatoma or both.
Ans. 2
B. burgdorferi, the causative agent of Lyme disease, elicits Ans. 4
an acute antibody response. IgM appears within days to a CMV is harbored in blood leukocytes. While acute CMV
few weeks following tick bite, and 19G appears a few infection may cause transient fever, jaundice, and
weeks later. 19G persists; IgM does not. Cross-reactions hepatosplenomegaly in cases of large blood donor
occur with other treponemes. exposures, post transfusion l.CMV infection
Fifth disease is a viral exanthema commonly seen in (seroconversion) is not a significant clinical problem in
children 8 to 12 years old. Children are ill for a few days immunocompetent recipients, and therefore blood is not
but recover without incident. Unfortunately, if a pregnant routinely tested for the presence of CMV Post transfusion
female acquires the disease in the first trimester of non-A, non-S hepatitis, however, not only represents the
pregnancy, the fetus is at risk. The causative agent is most frequent infectious complication of transfusion but
thought to be a Parvovirus (Parvovirus B 19). also is associated with an incidence of chronic active
hepatitis up to 16% and an 8 to 10% incidence of cirrhosis
44. A patient has developed an acute congestive or hepatoma or both. The etiologic agent in over 90% of
glaucoma in his right eye. The prophylactic measure to cases of post transfusion hepatitis has' been identified as
be taken in the fellow eye is: hepatitis C.
1. Topical steroids
2. Peripheral iridectomy

9
47. A 60-yrs-old diabetic man undergoes resection of lanreotide etc. Surgery is done whenever possible to
an infected boil on his back. The wound is left open for remove the tumours.
packing and cleansing. Week by week, the wound
grows smaller. Which of the following is true 49. Following blunt abdominal trauma, A 2 year old
concerning wound contracture? girl develops upper abdominal pain and vomiting. An
1. It is the primary process affecting closure of a sutured upper gastrointestinal series reveals a total
or stapled surgical wound obstruction of the duodenum with a coiled spring
2. Bacterial colonization significantly slows the process appearance in the second and third portions.
of contraction Appropriate management is
3. It may account for a maximum of 50% decrease in the 1. Gastrojejunostomy
size of a wound 2. Nasogastric suction and observation
4. It is based on specialized fibroblasts that contain actin 3. Duodenal resection
myofilaments 4. Duodenojejunostomy

Ans. 4 Ans. 2
While epithelialization is responsible tor the healing of a Duodenal hematomas result from blunt abdominal trauma.
closed incision, wound contraction is the primary method They present as a high bowel obstruction with abdominal
of closure in open wounds. During this process, the skin pain and occasionally a palpable right upper quadrant
surrounding the wound is pulled over the wound surface mass An upper gastrointestinal series is almost always
and may account for up to a 90% reduction in the size of diagnostic with the classic coiled. spring appearance of the
an open wound. In areas of greater adherence of skin to second and third portions of the duodenum secondary to
underlying tissue, the ability of contraction to close the the crowding of the valvulae conniventes (circular folds)
wound is hindered due to the decreased mobility of the by the hematoma. Nonsurgical management is the
skin, Therefore, in areas of tight skin adherence such as the mainstay of therapy because the vast majority of duodenal
leg, contraction may only account for- 30 to 40% reduction hematomas resolve spontaneously Simple evacuation of
in wound size. Fibroblasts in the open wound, which the hematoma is the operative procedure of choice.
predominate during the proliferative phase, contain However, bypass procedures and duodenal resection have
increasing numbers of actin microfilaments, thereby been performed for this problem. In patients with
becoming myofibroblasts, These specialized fibroblasts are duodenal obstruction from the superior mesenteric artery
thought to be responsible tor wound contraction either syndrome, the obstruction is usually the result of a marked
through intrinsic cellular contraction or attachment to weight loss and, in conjunction with this, loss of the
collagen strands. retropentoneal fat pad that elevates the superior
mesenteric artery from the third and fourth portions of the
48. A 35-yrs-old woman complains of attacks duodenum. Nutritional repletion and replenishment of this
of breathlessness, cyanosis and flushing. Apart fat pad will elevate the artery off the duodenum and
from occasional diarrhea she has no relieve the obstruction.
abdominal symptoms. Abdominal examination
reveals an enlarged nodular liver. If 50. A 42- year old man is undergoing chemotherapy
laparotomy is done, we could expect to find: after resection of a cecal adenocarcinoma with
1. An ovarian tumor positive lymph nodes. Which of the following
2. A multicentric hepatoma potentially operable complications is a common
3. An appendicular carcinoid occurrence among patients receiving systemic
4. Crohn's disease chemotherapy?
1. Acute cholecystitis
Ans. 3 2. Perirectal abscess
History of breathlessness, cyanosis and flushing points 3. Appendicitis
towards carcinoid syndrome. Enlarged nodular liver 4. Diverticulitis
means hepatic metastasis, which is essential to produce
carcinoid syndrome. Ans. 2
Appendicular carcinoids are detected in I in ever) 200-300 A surgeon is frequently asked to evaluate patients who are
appendicectomies. It is one of the commonest sites for receiving systemic chemotherapy Most complications of
carcinoid tumours. chemotherapy do not require surgical therapy. Perirectal
Carcinoid tumours are commonly seen in appendix, small abscesses are more common in these immunosuppressed
intestine, lungs, stomach etc. patients. 'Gastrointestinal bleeding occurs secondary to
Flushing, diarrhoea, wheezing, pellagra like skin lesions mucosal irritation and thrombocytopenia, Pancreatitis is
are the common features. uncommon, but is associated with L-asparaginase use. Up
Carcinoid tumours secrete large amount of serotonin. to 20% of patients treated with floxuridine by continuous
Treatment is with somatostatin analogues like octreotide, hepatic artery infusion develop some degree of
inflammation and obstruction of the bile duct. Systemic

10
chemotherapy does not increase the likelihood of acute 4. The most common underlying cause today is infection
cholecystitis. appendicitis, incarcerated femoral hernia, or with MDR tuberculosis
diverticulitis.
Ans. 3
51. The operation of choice for the below condition Failure to recognize adrenal cortical insufficiency,
particularly in the postoperative patient, may be a fatal
error that is especially regrettable because therapy
(exogenous steroids) is effective and easy to administer.
Adrenal insufficiency may occur in a host of settings
including tuberculosis (formerly the most common cause),
autoimmune states" severe infections, (classically,
meningococcal septicemia), pituitary insufficiency, after
burns, during anticoagulant therapy, and-most commonly
today-after interruption of chronically ,administered
exogenous steroids. Although the adrenal gland is an
occasional site for distant metastases, such as from lung or
breast, it is rare for there to be enough destruction of the
glands to produce clinical adrenal insufficiency Chronic
adrenal insufficiency (classic Addison's disease) should be
recognizable preoperatively by the constellation of skin
igmentation, weakness, weight loss, hypotension, nausea,
1. Division of annular ring vomiting, abdominal paib hypoglycemia, hyponatremia,
2. Gastrojejunostomy and hyperkalemia. ' Death may occur within hours of
3. Duodenojejunostomy surgery if a patient with Addison's disease is operated on
4. Pancreaticoduodenal tom without cognizance of adrenal insufficiency and
pretreatment with exogenous steroids. Patients who have
Ans. 3 adrenal insufficiency as a result of interruption of
The treatment of annular pancreas is duodenojejunostomy chronically administered exogenous steroids may not
or gastrojejunostomy either of which bypasses the develop the classic electrolyte abnormalities until the
obstruction. preterminal period. Adrenal insufficiency may also
According to Bailey and Love duodenoduodenostomy or develop insidiously in the postoperative period,
duodenojejunostomy are usual operation. Obstructing progressing over a course of several days. This insidious
pancreatic ring must not be simply divided. Since this may course is seen when adrenal injury occurs in the
not relieve the obstruction and pancreatic fistula may perioperative period, as would be the case with adrenal
result. damage from hemorrhage into the gland in a patient
Annular pancreas is formed by a thin band of normal receiving postoperative anticoagulant therapy
pancreas tissue that completely encircles the second Measurement of blood corticosteroid levels, urinary
portion of duodenum and is continuous with head of corticosteroid secretion, urinary sodium levels, and
pancreas anteriorly and posteriorly. 40% patients have response to exogenous steroids is helpful in establishing
associated duodenal stenosis or atresia. the diagnosis of adrenal insufficiency
Symptoms of duodenal obstruction (gastric distention and
vomiting) occur in about one third of cases in the first 53. A 70- year old woman has, vomiting abdominal
week of life and about one half of cases in the first year. distension and episodic crampy midabdominal pain.
Rests are asymptomatic until adulthood, when abdominal She has no history of previous surgery. Her abdominal
pain, nausea and vomiting may occur. radiograph reveals a spherical density in the right
lower quadrant. Correct treatment should consist of
52. A patient with mild skin pigmentation comes to 1. Ileocolectomy
you because of sudden abdominal pain, fever and a 2. Cholecystectomy
rigid abdomen. Her lab report show blood sugar of 55, 3. IIeotomy and extraction
Na of 119, and K of 6.2. Her BP is 88/58. She undergoes 4. Nasogastric tube decompression
an exploratory laparotomy. Which of the following
statement is true? Ans. 3
1. Treatment with exogenous steroids is usually Gallstone ileus is due to erosion of a stone from the
ineffective gallbladder into the gastrointestinal tract (most commonly
2. This condition is commonly seen as a consequence of the duodenum). The stone becomes lodged in the small
metastasis of distant cancers, such as lung or breast to bowel (usually in the terminal ileum) and causes small-
the a al glands bowel obstruction. Plain films of the abdomen that
3. Death from untreated chronic adrenal insufficiency demonstrate small-bowel obstruction and air in the biliary
may occur within hours of surgery tract are diagnostic of the condition. Treatment consists of

11
ileotomy, removal of the stone, and cholecystectomy if it is tract, vagina, or perineum They occur in approximately 1
technically safe. If there is significant inflammation of the in 2000 live births. Depending on the type of anomaly
right upper quadrant, ileotomy for stone extraction (whether the rectum ends above or below the level of the
followed by an interval cholecystectomy is often a safer levator ani complex), a variety of surgical procedures has
alternative. Operating on the biliary fistula doubles the been devised to treat the problem.
mortality rate compared with simple removal of the However, even when anatomic integrity is established, the
gallstone from the intestine. prognosis for effective toilet training is poor in 50% of
cases continence is never achieved. Cervical spine
54. A 24-yrs-old male, who has been having fever for abnormalities, hydrocephalus, duodenal atresia, and
15 days, starts having acute pain and distension of corneal opacities have no significant association with
abdomen. Abdominal examination reveals generalized congenital anorectal anomalies.
tenderness with guarding. The most likely diagnosis is:
1. Acute appendicitis 56. A 63-yrs-old man is having facial swelling
2. Acute pancreatitis
3. Enteric perforation
4. Duodenal ulcer perforation

Ans. 3
The clinical picture described is classical of enteric
perforation and peritonitis.
In enteric fever ulcers occur in intestine, which may
perforate if not treated properly. This complication is seen
in third or fourth week of fever. It is a life threatening
complication and urgent medical and surgical intervention
is required;
GIT haemorrhage is another life threatening complication What condition is the most likely cause of these
of enteric fever that occurs in third or fourth week of fever. findings?
In acute appendicitis and acute pancreatitis abdominal 1. Histoplasmosis
pain is the main feature rather than fever of 15 days 2. Substernal thyroid
duration. In DU perforation fever is absent. 3. Thoracic aortic aneurysm
4. Bronchogenic carcinoma
55. A neonate is found to have a condition as shown
below. You recommend studies to search for other Ans. 4
anomalies. Associated abnormalities include which of Superior vena cava obstruction is almost always due to
the following? malignancy and, in three out of four cases, results from
invasion of the vena cava by bronchogenic carcinoma.
Lymphomas account for most of the remaining cases of the
superior vena cava syndrome Fibrosing mediastinitis as a
complication of histoplasmosis or ingestion of
methysergide may occur but is rare.' Rarely a substernal
thyroid or thoracic aortic aneurysm may be responsible
for the obstruction.
Although constrictive pericarditis may decrease venous
return to the heart, it does not produce obstruction of the
superior vena cava. Whatever the cause of the superior
vena cava syndrome, the resultant increased venous
pressure produces edema of the upper body, cyanosis,
dilated subcutaneous collateral vessels in the chest, and
1. Abnormalities of the cervical spine headache. Cervical lymphadenopathy may also be present
2. Hydrocephalus as a result of either stasis or metastatic involvement, When
3. Duodenal atresia carcinoma is the cause of the superior vena cava
4. Heart disease syndrome, the treatment is usually palliative and consists
of diuretics and radiation.
Ans. 4
Congenital anorectal anomalies are frequently associated 57. Spinal Anesthesia should be injected into the space
with other congenital anomalies including heart disease; between:
esophageal atresia; abnormalities of the lumbosacral 1. T12-L1
spine; double urinary collecting systems; hydro nephrosis; 2. L3 L4
and communication between the rectum and the urinary 3. L1_ L2

12
4. L5-S1
Ans. 3
Ans: (2) The thoracic outlet syndrome designates a symptom
The LA is injected in the sub arachnoid space complex whose precise cause is unknown. It is felt to result
between L2-L3 or L3-L4. from compression of the brachial plexus or subclavian
Spinal cord extends from medulla oblongata to lower vessels, or both, in the anatomic space bounded by the first
border of L, in adult. rib, the clavicle, and the scalene muscles. Since objective
Spinal; cord extends from medulla oblongata to determinants of disease may be lacking or imprecise, the
lower border of L3 in infant & neonate., (adult level diagnosis often is established by resectional surgery.
Carpal tunnel syndrome (compression of the median nerve
achieved at 2 years of age)
as it passes through the carpal tunnel of the wrist) and
In infancy spinal; anesthesia given at L4-L5. Level
cervical disk disease are the two entities most commonly
In adult spinal anaesthesia given at L3-L4 OR L4rL5
confused with the thoracic out let syndrome, whose
space, (even at L2~L3) symptoms and signs include pain, paresthesias, edema,
In. adult usually spinal anesthesia given at l3-l4 level venous congestion, and digital vasospastic changes.
Remember structures encountered during spinal Positional dampening or obliteration of the radial pulse is
anesthesia an unreliable finding since it is present in up to 70% of the
1-skin, 2-subcutaneous tissue, 3-supraspinous normal population. Neurologic abnormalities may be
ligament, 4-lnterspinous ligament, 5-ltgament documented by nerve conduction studies. Angiographic
flavum, 6-dura, 7-arachnoid. studies are often negative. Conservative management,
which generally should precede surgery, consists of an
58. A 50-yrs-old male has a severe vomiting and exercise program to strengthen shoulder girdle muscles
retching spell with 'sharp substernal pain. He comes to and decrease shoulder droop. Operative treatment
you 4 h later and has a chest film in which the left includes division of the scalenus anticus and medius
descending aorta is outlined by air density. Optimum muscles, first rib resection, cervical rib resection, or a
strategy for care would be: combination of all three.
1. Immediate thoracotomy
2. Serial ECGs and CPKs to rule out MI 60. A 4-yrs-old boy is brought to the physician by his
3. Left chest tube and spit fistula parents for episodes of waking in the middle of the
4. Esophagogastroscopy to establish diagnosis night screaming. The parents state that when they get
to the boy's room during one of these episodes, they
Ans. 1 find him in his bed, thrashing wildly, his eyes wide
The presence of air in the mediastinum after an episode of open. He pushes them away when they try to comfort
vomiting and retching is virtually pathognomonic of him. After 2 min, the boy suddenly falls asleep, and the
spontaneous rupture of the esophagus (Boerhaave next day he has no memory of the episode. Which of
syndrome). The evidence is overwhelming that without the following medication is used to treat this disorder?
prompt surgical exploration of the mediastinum by left 1. Haloperidol
thoracotomy, the patient has little chance for a short-term 2. Diazepam
outcome of low morbidity. The aspiration of highly acidic 3. Methylphenidate
gastric contents into the mediastinum creates havoc in the 4. Amitriptyline
tissues exposed to it. The surgical procedure must include
extensive opening of the mediastinal pleura and removal Ans. 2
of any particulate debris that might have been aspirated The child in the question is experiencing episodes of sleep
into the thorax from the stomach. Closure of the terror disorder, a dyssomnia characterized by sudden
esophageal laceration with reinforcement by a pleural flap partial arousal accompanied by piercing screams, motor
and secure chest tube drainage of the pleural space are agitation, disorientation, arid autonomic arousal. The
mandatory. episodes take place during the transition from deep sleep
to REM sleep. Children do not report nightmares (which
59. A 27-yrs-old woman seeks your advice regarding would be associated with REM sleep) and do not have any
pain and numbness in the right arm and hand .. She memory of the episodes the next day Sleep terrors occur in
reports that it is exacerbated by raising her arm over 3% of children and I % of adults. Although specific
her head. A provisional diagnosis is made. Which treatment for this disorder is seldom required, in rare
statement is true of this conditions? cases it is necessary. Diazepam in small doses at bedtime
1. It is associated with cervical spine disk disease improves the condition and sometimes completely
2. It is reliably diagnosed by positional obliteration of the eliminates the attacks.
radial pulse
3. If conservative measures fail, it is best treated by 61. A 37-yrs-old alcoholic is brought to the
surgical decompression of the brachial plexus emergency room after he was found unconscious
4. It most commonly affects the median nerve in the street. He is hospitalized for dehydration

13
and pneumonia. While being treated, he becomes schizophrenia, severe tardive dyskinesia, and psychotic
acutely confused and agitated He cannot move symptoms in patients with Parkinson's disease.
his eyes upward or to the rig , and he is ataxic.
Which of the following diagnosis is most likely? 64. A 51-year-old man has noted right hip pain for the
1. Alcohol intoxication past 3 months. On physical examination he has
2. Korsakoff's syndrome diminished range of motion of the right hip. A
3. Alcohol delirium radiograph reveals a 10 x 13 cm mass involving the
4. Wernicke's encephalopathy right ischium of the pelvis. The mass has irregular
borders and there are extensive areas of bony
Ans. 4 destruction along with some scattered calcifications.
Wernicke's encephalopathy occurs in nutritionally The lesion is resected, and grossly the mass has a
deficient alcoholics and is due to thiamine deficiency and bluish-white cut surface. Which of the following
consequent damage of the thiamine- dependent brain attributes is most likely to describe this mass?
structures, including the mammillary bodies and the 1. The most frequent primary tumor of bone
dorsomedial nucleus of the thalamus. It presents with 2. Usually seen in distal skeletal bones
mental confusion, ataxia, and sixth nerve paralysis. 3. More common in females
Wernicke's encephalopathy is a medical emergency and 4. Sometimes seen to arise in benign cartilagenous
can rapidly resolve with immediate supplementation of tumors
thiamine.
Ans. 4
62. Hill - Sach's lesion seen in: CORRECT 4. Though probably a sporadic occurrence in the
1. Anterior dislocation of hip case desccribed here, chondrosarcomas can occur in
2. Posterior dislocation of hip association with osteochondromatosis.
3. Recurrent dislocation of shoulder
4. Posterior dislocation of shoulder Incorrect 1. Osteosarcoma is the most frequent primary
malignancy of bone, not chondrosarcoma.
Ans: (3) Incorrect 2. Distal cartilagenous tumors are more likely to
Hill Sach's lesion and Bankart lesion causes recurrent be benign, whereas those in a central location are virtually
dislocation of shoulder always malignant.
In previous year they asked about the Luxation Inferior Incorrect 3. Primary malignancies of bone, including
dislocation. chondrosarcoma, are more common in males.
Most common Type of Shoulder dislocation anterior
dislocation of shoulder (sub-coracoid type) dislocation.
65. M/C used in Brachytheraphy of Carcinoma cervix:
Surgeries for Recurrent dislocation of shoulder:
1. Radium-226
Bankart s operation 2. Cobalt-60
Puti platt's operation 3. Gold-198
Bristow's operation. 4. Caesiom-137

63. A schizophrenic patient has no interest in social Ans: (4)


contacts or vocational rehabilitation. His affect is flat, Brachytherapy: Type of radiation in which radioactive
and he speaks very little and spends most of his day sources are placed in dose proximity to or directly into
sitting in front of the TV, unwashed and unshaven. He
tumors.
has some chronic, delusions of persecution, but these
Interstial RT Needles Placed into Prostate/penis/other
do not impact his functioning as much as the previous
soft tissue.)
symptom. Which of the following anti psychotics.
Would-tie most appropriate to use in treating this Intracavitary
patient? eg. In urethra (m/c used csl37) M/C used source in
1. Molindone Brachytherapy - Irl 90. C.S.I. REMEMBER, some
2. Haloperidol decanoate commonly asked half life of ISOTOPES
3. Chlorpromazine Radium 2261620 years
4. Olanzapine Caesium- 13730 years
Cobalt 60 ----- 5.3 years
Ans. 4 Iridium 19274.4years
The schizophrenic patient in the question manifests a Iodine 125 ---- 60 days
prevalence of negative symptoms (flat affect, abulia, lack of Phosphorus3214.3 days
motivation). Two antipsychotic medications have been Gold 1982.7 days
proved to be effective for negative symptoms: clozapine
and olanzapine. However, due to clozapine's high risk for 66. A 38-yrs-old woman with bipolar disorder has
agranulocytosis, its use is limited to refractory cases of been stable on lithium for the past 2 years. She comes
14
to her psychiatrist's office in tears after a 2week
history of a depressed mood, poor concentration, loss
of appetite, and passive suicidal ideation. Which of the
following steps should the psychiatrist take next?
1. Start the patient on a second mood stabilizer
2. Start the patient on a long-acting benzodiazepine
3. Stop the lithium and start an antidepressant
4. Start an antidepressant and continue the lithium

Ans. 4
Since lithium and other mood stabilizers are more effective
in the prevention of manic episodes than in the prevention
of depression, antidepressants are used as an adjunctive
treatment when depressive episodes develop during
maintenance with a mood stabilizer. Since the incidence of
antidepressant-induced mania is high (up to 30%), and
since antidepressant treatment may cause rapid cycling,
the antidepressant should be tapered and discontinued as
soon as the depressive symptoms remit. Among the
antidepressants in common use, buproprion is considered
to carry a slightly lower risk of triggering mania.

67. Which of the following statements about


obsessive-compulsive disorder is correct?
1. It usually has an onset in middle age
2. It has a lifetime prevalence of 2.5%
3. It is usually caused by traumatic events 1. Ulcerative colitis
4. It is not responsive to pharmacological intervention 2. Carcinoma colon
3. Crohn's disease
Ans. 2 4. Whipple's disease
Obsessive-compulsive disorder usually has its onset from
late childhood to early adulthood and has a lifetime Ans, 1
prevalence of about 2.5%. Stress is usually associated with Pipestem colon is seen in ulcerative colitis
worsening of symptoms, but traumatic events do not cause String sign of Kantor is seen in Crohn 's disease.
this disorder. While a minority of patients do have periods
of complete remission, about one-third have a fluctuating 69. A 40-year-old man is brought by his friends to you.
course and up to 60% have a constant or progressive Apparently, he has ingested some unknown
course. Many patients are markedly improved by medication. Patient is disoriented. His temperature is
pharmacological intervention. The most commonly used 103F, BP 120/85, pulse 100/min, and respirations
agents are serotonin reuptake inhibitors such as fluoxetine 221min. The skin is flushed and dry. Has dilated pupils
and sertraline, although the tricyclic clomipramine is also and muscle twitching. ECG reveals prolonged QRS
used. complexes. Hepatic transaminases are normal, and
ABG shows a normal pH. These findings are most likely
68. Below X-ray is seen in: due to intoxication by which of the following
substances?
1. Acetaminophen
2. Alcohol
3. Benzodiazepines
4. Tricyclic antidepressants

Ans. 4
This patient's clinical picture is consistent with
intoxication with tricyclic antidepressants such as
amitriptyline and imipramine. Toxic effects 'are mediated
by peripheral anticholinergic activity and "quinidine-like"
action. The anticholinergic effects include mydriasis,
tachycardia, impaired sweating with flushed skin. dry
mouth, constipation, and muscle twitching. Quinidine-like
effects (due to block of sodium channels in. the heart)
result in cardiac arrhythmias, especially ventricular
15
tachyarrhythrnias, In this setting, prolongation of the QRS ejaculation, anorgasmia, and decreased libido. Buproppion,
complex is particularly important in the diagnosis. QRS mirtazapine, trazodone, and nefazodone, in contrast, do
width is, in fact. an even more faithful parameter of drug not affect sexual functions in a negative way Trazodone
toxicity than serum drug levels. In severe intoxication, and nefazodone, however, have been implicated in cases of
patients will develop seizures, severe hypotension, and priapism and should not therefore be used as first-line
coma. medications in male patients.

70. A 50-year-old man is brought to the emergency 73. DNA sequencing by the Sanger method take:
department by ambulance. His respirations are advantage of which property of DNA synthesis to
shallow and infrequent, his pupils are constricted, and generate a sequencing ladder?
he is stuporous. He was noted to have suffered a 1. Addition of nucleotides requires a free 01-I group on
grandmal seizure in the ambulance. Which of the the 3' end of the DNA strand
following drugs is this man likely to have overdosed 2. DNA has a free 5' phosphoryl group
on? 3. DNA polymerase has proofreading capabilities
1. Cocaine 4. Nucleotides are linked by phosphodiester bonds
2. LSD
3. Meperidine The answer is 1.
4. PCP The Sanger method of DNA sequencing is also known as
chain termination sequencing. In this method, DNA strands
Ans. 3 are synthesized in the presence of a low concentration of
Severe opiate intoxication is associated with respiratory dideoxynucleosides (ddNTPs) in addition to
depression, stupor or coma, and sometimes pulmonary deoxynucleotides (dNTPs). Incorporation of a dNTP allows
edema. Less severe intoxication is associated with slurred synthesis to continue as there is a free 3-OH group for
speech; drowsiness, and impaired memory or attention. addition of the next nucleotide. Incorporation of a ddNTP
Early on, the pupils are constricted, but they dilate if the terminates synthesis of the DNA strand as there is no free
patient becomes 'anoxic due to the respiratory depression. 3OH group for addition of the next nucleotide.
BP is typically. Meperidine intoxication in a chronic user is In Sanger sequencing, four reactions (A, C, T, and G) are
often complicated by delirium or seizures due to the run, each containing the appropriate ddNTP. The mixture
accumulation of nor meperidine, a toxic metabolite with of DNA strands generated in each reaction includes
cerebral irritant properties. strands that are terminated at each given nucleotide (e.g.,
the A reaction contains DNA strands terminated at each A).
71. Filament in X-ray made of: The four reactions are run adjacent to one another on an
1. Tungsten filament acrylamide gel to separate the strands by size. The
2. Chromium sequence of the DNA is determined by reading the order of
3. Gallium the terminated chains going up the gel.
4. Selenium
74. Investigation of choice in a person brought with
Ans: (1) traumatic paraplegia is:
Please remember: Target is not Tungsten in 1. MRI
mammography, its made up off molybdenum. 2. CT
3. Myelography
4. USG
72. A 25-year-old man with major depression '
discusses the potential benefits and side effects of Ans: (1)
All spinal imaging is done by MRI
various antidepressants with his psychiatrist. He
MRI is safe in pregnancy Unit of MRI
clearly indicates that he does not want a medication
that could decrease his libido or interfere' with his is TESLA.
ability to obtain and maintain an erection. Which of
the listed antidepressants would be appropriate for 75. Which of the following characterizes Q fever?
this patient? 1. Has an incubation period of four to six weeks
1. Bupropion 2. is an acute febrile illness caused by Coxiella Burnetii
2. Clomipramine 3. Is an illness confined to the upper respiratory tract
3. Amitriptyline 4. Is most commonly found in tropical region
4. Sertraline
Ans. 2
Ans. 1 Q fever is an acute, flulike illness caused by C. burnetii. It is
Tricyclic antidepressants such as clomipramine and the one rickettsial disease not transmitted by the bite of a
amitriptyline and SSRIs such as paroxetine and sertraline, tick. C. burnetii is found in high concentrations in the
as well as MAOIs, can cause erectile dysfunction, delayed urine, feces, and placental tissue/amniotic fluid of cattle,

16
goats, and sheep. Transmission to humans is by aerosol Third stage larva in mosquito is infective form.
inhalation of those specimens. Adult worm may survive for 15 years or more.
5. Duration of mosquito cycle (extrinsic incubation period)
76. Which one of the following organism normally is between 10 and 14 days.
inhabits the healthy human oral cavity? 6. Main vectors in India are: C. quinquefasciatus (C.
1. M. fermentans fatigans) for Bancroftian filariasis and Mansonia
2. M. hominis mosquitoes for Brugian filariasis.
3. M. pneumoniae 7. Prepatent period: It is the time interval between
4. M. orale inoculation of infective larvae and the first appearance of
detectable Mf.
Ans. 4 8. Clinical incubation period: Time interval from invasion
Members of the mycoplasma group that are pathogenic for of infective larvae to the development of clinical
humans include M. pneumoniae and U. urealyticum. M. manifestations.
pneumoniae is best known as the causative agent of
primary atypical pneumonia (PAP), which may be 78. H. capsulatum, is found in soil heavily
confused clinically with influenza or legionellosis. It also is contaminated with bird droppings. Which of the
associated with arthritis, pericarditis, aseptic meningitis, following statements best describes the presence of
and the Guillain-Barn5 syndrome. M. pneumoniae can be the organism in tissue biopsies?
cultivated on special media and identified by 1. Arthrospores
immunofluorescence staining and "fried egg" colonies on 2. Oval budding yeast inside macrophages
agar. 3. Single-cell yeast with pseudohyphae
U. urealyticum (once called tiny, or T, strain) has been 4. Spherules containing endoscopores
implicated in cases of nongonococcal urethritis. As the
name implies, this organism is able to split urea, a fact of Ans. 2
diagnostic significance. U. urealyticum is part of the H. capsulatum is a dimorphic fungus that forms two types
normal flora of the genitourinary tract, particularly in of spores: tuberculate macroconidia and microconidia.
women. Inhalation of the microconidia transmits infection. Inhaled
Both M. orale and Mycoplasma salivarium are inhabitants spores (microconidia) are engulfed by macrophages and
of the normal human oral cavity. These species are develop into yeast forms. Most infections remain
commensals and do not play a role in disease. asymptomatic; small granulomatous foci heal by
The only other species of Mycoplasma associated with calcification. However, pneumonia can Occur. The
human disease is M. hominis. A normal inhabitant of the heterophile anti body test is useful for early diagnosis of
genital tract of women, this organ ism has been infectious mononucleosis.
demonstrated to produce an acute respiratory illness that
is associated with sore throat and tonsillar exudate, but 79. The diagnostic characteristics of Plasmodium
not with fever. falciparum are best described by which one of the
M. fermentans is an animal isolate. following statement?
1. An important diagnostic feature is the irregular
77. Which one of the following is characteristic of appearance of the infected red blood cell
microfilaria of W. bancrofti? 2. A period of 72 Hour is required for development of the
1. Cephalic space twice as long as broad mature schizont, which resembles a rosette with only
2. Excretory pore prominent 8 to 10 oval merozites
3. Nuclear column smudged 3. Except in infection with very high parasitemia, only
4. Caudal end uniformly tapering to a delicate point, and ring forms of early trophozoites and the gametocytes
no terminal nuclei present are seen in the peripheral smear
4. Schuffner stippling is routinely seen in red blood cells
Ans. 4 that harbor parasites
The features of microfilaria of W. bancrofti are:
General appearance: Graceful, sweeping curves. Ans. 3
Free cephalic space: As long as broad. P. falciparum infection is distinguished by the appearance
Excretory pore: Not prominent: of ring forms of early trophozoites and gametocytes, both
Caudal end: Uniformly tapering to a delicate point and no of which can be found in the peripheral blood. The size of
terminal nuclei present. the RBC is usually normal. Double dots in the rings are
5. Nuclear column: Nuclei discrete; Important points common.
1. Microfilaria of W. bancrofti and B. malayi in India
display a nocturnal periodicity. Maximum density of Mf in 80. Alopecia, Hyper pigmentation, Hypogonadism
blood is reported between 10 pm and 2 am. characteristic for deficiency of
2. Man is the definitive host and mosquito the intermediate 1. Zinc
host of Bancroftian and Brugian filariasis. 2. Magnesium

17
3. Selenium are generally CD16+, CD56+, and CD3-, which contrasts
4. Copper them with I cells which are CD3+, CDl6-, and CD56

Ans: (1) 83. Isomorphism not seen in:


The essential feature of Zinc deficiency. 1. Lichen planus
1. A distinctive rash most often around the eyes, 2. Warts
nose, mouth, anus and distal parts called 3. Psoriasis
Acrodermatitis Enteropathica. 4. Vitiligo
2. Growth retardation in children.
3. Impaired wound healing Ans: (4)
4. Hypogonadism with diminished reproductive Isomorphism or Koebner phenomenon-It means linear
capacity. lesions produced by scratching a primary lesion which
5. Anorexia often accompanied by diarrhoea results in new lesions developing along the line of the
6. Depressed mental function. scratch. Eg: Lichen planus, warts and psoriasis. But
Other important disease Koebner phenomenon is characteristic for Lichen
Selenium deficiency causes Keshan disease. planus.
Copper deficiency causes Menke's disease.
84. The sodium gradient across the nerve cell
81. Survival of allografts is increased by choosing membrane IS:
donors with few MHC mismatches with recipients and 1. A result of the Donnan equilibrium
by use of immunosuppression in recipients. Which one 2. Significantly changed during an action potential
of the following procedure might be useful measure of 3. Used as a source of energy for the transport of other
immunosuppression? ions
1. Administration of corticosteroids to recipient 4. An important determinant of the resting membrane
2. Administration of immunoglubin to recipient potential
3. Destruction of donor B cells
4. Destruction of donor T cells Ans. 3
The sodium- potassium pump uses the energy contained in
Ans. 1 ATP to maintain the sodium gradient across the
Allograft rejection is primarily a T cell response to foreign membrane. The sodium gradient, in turn, is used to
tissue. The corticosteroids reduce inflammatory response transport other substances across the membrane. For
and are generally administered by cytotoxic drugs, such as example, the NaCa exchanger uses the energy in the
cyclosporine. Lymphoid irradiation is usually done so that sodium gradient to help maintain the low intracellular
the bone mar row is shielded. This removes lymphocytes calcium required for normal cell function. Although
from lymph nodes and spleen while allowing the patient to sodium enters the cell during an action potential, the
have the capacity to regenerate new T and B cells. quantity of sodium is so small that no significant change in
Likewise, antilymphocyte globulin will destroy the intracellular sodium concentration occurs. Because the
recipient's lymphocytes, especially T cells. Destruction of sodium transference is so low, the sodium equilibrium
donor B cells and T cells would not play a role in the potential is not an important determinant of' the resting
immunosuppression of the graft recipient. In graft crises, membrane potential.
monoclonal antibody to CD3 is sometimes given. This
targets mature T lymphocytes for destruction. 85. Decrease number of melanocyte is seen in:
1. Albinism
82. A laboratory analysis report of a specific fraction of 2. Piebaldism
a patient's lymphocytes the following: 3. Ichthyosis
HLA, Band C+,PHA+, CD3-, CD16+, CD11a/ CD18+, 4. Chemical leucoderma
CD56+, and in vitro blastogenesis with IL12 What are
the lymphocytes this set describes? Ans: (2)
1. B lymphocytes Piebaldism is a Autosomal dominant disease characterized
2. Cytotoxic T lymphocytes by patches of skin that lack melanin as a result of defect in
3. Natural killer cells migration of melanoblast from neural crest to skin.
4. T helper 1 subset Skin biopsy: Hypo melanotic area-few to no melanocytes.
Vitiligo: is an acquired disease and similar patch of loss of
Ans. 3 melanin, but the loss develops after birth and progressive.
NK cells do not express a cell surface TCRlCD3 complex Albinism: Congenital inability to synthesize melanin.
and are CD4-. About half of human NK cells are CD8+. Also,
most NK cells express an Fe IgG receptor, known as CDI6,
and CD56, a neural cell adhesion molecule variant. NK cells 86. Not a cutaneous manifestation of TB:
1. Lupus pernio
18
2. Lupus vulgaris Down syndrome or mongolism is caused by trisomy of
3. Erythema nodosum chromosome 21.
4. Scrofuloderma - It is characterized by short stature and small rounded
head, narrow, tilted eye slits, malformed ears, short broad
Ans: (1) hands, lax limbs, mental retardation, cardiac defects and
Lupus pernio is a skin manifestation of Sarcoidosis. atresia of alimentary tract.
Sarcoidosis does not involve kidney. - The frequency of mongolism increases with rising
Sarcoidosis mimics like Berylliosis. maternal age but is unaffected by the age of the father.
Polygenic or multifactorial disorders are essential
87. An ECG obtained from a 57-yrs-old male during a hypertension, schizophrenia, mental retardation, duodenat
routine physical examination reveals atrial ulcer, ischaemic heart disease of early onset, diabetes
fibrillation. Which of the following is most likely to mellitus, congenital heart malformation, familial cancer
accompany this condition? syndromes including familial polyposis coli, familial non
1. An increased venous 'a' wave polyposis colon cancer and some thyroid thncers. Also in
2. An increased left atrial pressure cancer of colon and breast, there is a major genetic
3. A decreased heart rate component but it is difficult to distinguish familial from
4. An increased stroke volume non familial cases.
The mode of inheritance of multifactorial disorders, is
Ans. 2 complex because environmental factors are also involved.
Atrial fibrillation is an arrhythmia in which the electrical The relative contributions of genetic predisposition and
activity of the atrium becomes disorganized and therefore environmental factors to the aetiology will vary greatly
unable to produce a coordinated atrial contraction. The from patient to patient.
absence of an atrial pulse reduces the emptying of the atria
during diastole and results in an enlarged left atrium and 90. Which of the following tend to show marked and
increased left atrial pressure. The venous A wave homogeneous enhancement on Gd-DTPA enhanced
represents atrial contraction and disappears due to the MRI?
absence of an atrial beat. Decreased filling of the heart 1. Meningiomas and acoustic neuromas
results in a decrease in stroke volume. Heart rate increases 2. Oligodendrogliomas and Metastases
because the continuous electrical activity of the atria 3. Meningiomas and low grade gliomas
initiates a high rate of ventricular activity Systemic blood 4. Glioblastoma multiformies and acoustic neuromas
pressure typically falls because of inadequate filling of the
ventricles and the resulting decrease in stroke volume. Ans. (1)
Meningiomas and acoustic neuromas tend to show marked
88. Stroke volume can be decreased by which of the and homogeneous enhancement 0." Gd-DTPA-enhanced
following? MRI Ring-shaped contrast enhancement occurs in most
1. Increasing ventricular contractility glioblastoma8, whereas low-grade gliomas may show lack
2. Increasing heart rate of changes on postcontrast images or only patchy
3. Decreasing total peripheral resistance enhancement.
4. Decreasing systemic blood pressure Depending on tumor size, metastases present as small
homogeneously enhancing lesions or as ring-shaped
Ans. 2 structures secondary to central necrosis, which adds to the
Stroke volume is determined by preload, afterload, and hallmark of metastatic tumors (i.e., multiplicity of lesions).
contractility Increasing heart rate decreases the time for
filling during diastole and may decrease preload and 91. Which of the following test is positive in Anterior
therefore stroke volume. Increasing preload by increasing dislocation of shoulder?
central venous pressure will increase stroke volume. 1. Thomas test
Similarly, decreasing afterload by decreasing total 2. Barlow's test
peripheral resistance or systemic blood pres sure will 3. McMurry's test
cause an increase in stroke volume, Increasing 4. Duga's test
contractility will also increase stroke volume.
Ans. 4
89. Which one of the following is not a polygenic The following signs, associated with anterior dislocation
disorder? of shoulder.
1. Down's syndrome a. Duga's test
2. Diabetes mellitus b. Hamilton ruler test
3. Coronary heart disease c. Caliway's test.
4. Congenital heart disease
92. A neonate with respiratory distress and the
Ans. 1 following chest X ray most likely has:

19
Poison producing dilatation of pupils - Dhatura, Cyanides
Alternative contraction and dilatation &f pupils are found
in early stage of barbiturate poisoning.

95. Lumbar hemivertebra results due to the


abnormal development of:
1. Dorsal sclerotome.
2. Intermediate cell mass.
3. Notocord.
4. Ventral sclerotome.
1. Cystic fibrosis
2. CLE Ans. (4)
3. CCAM The embryonic mesoderm on either side of the notochord
4. Pneumonia segments into paired somites, which differentiate into
sclerotome. myotome .. and dermatome portions. The
Ans. 2 human vertebral column develops by endochondral bone
formation from the paired sclerotome elements. The
vertebral bodies are formed by fusion of a more compact
93. Use of condom prevents which of the following caudal portion of each sclerotome with the less dense
diseases? cranial portion of the sclerotome below, and dorsal
1. Carcinoma cervix migration of sclerotome tissue will give rise to the paired
2. Carcinoma vulva neural arches. At birth, ossification centers are present for
3. Herpes simplex genitalis the centrum of the vertebral body and for each half of the
4. All of the above neural arches, which have a cartilaginous union to the
centrum at the neurocentral synchondroses.
Ans. 4
The incidence of CA cervix is low amongst woman whose Segmentation anomalies like with hemivertebrae, butterfly
partners use condom because of prevention of vertebrae, and block vertebrae occur due to abnormal
transmission of viral infection (HPV) causing CA cervix. development of Ventral sclerotome.
Similarly it can also reduce the risk of CA vulva because it
is common in women suffering from STD. Herpes genitalis 96. Which one of the following statement regarding
is a DNA virus. It is transmitted sexually and is an the female contraceptive "Today" is true?
aetiological agent of CA cervix. The incubation period is 3- 1. It is polyurethane foam saturated with 9nonoxynol
6 days. Vesicles appear on erythematous base on vulva, 2. It is an oral pill containing norethisterine acetate
vagina and rarely cervix. These vesicles soon rupture 1.0mg and ethinyl oestradiol 0.03 mg
leaving multiple shallow ulcers which are painful after 3. It is an oral pill containing D-norgesterel 0.50 mg and
bacterial infections. Treatment is with acyclovir. Besides ethinyl oestradiol 0.3 mg
CA cervix it can also causes CA vulva and condom can 4. It is made of "gossypol" a derivative obtained from
prevent the transmission. Female latex condoms are most cotton seed oil
useful in prevention of CA vulva.
Ans. 1
94. The visual disturbance of methylalcohol poisoning Today is a polyurethane foam that contains the
include all of the following except: spermicidal agent 9-nonoxynol.
1. Pin point pupils
2. Photophobia and blurred vision 97. Quality control over drugs manufactured in India is
3. Concentric diminution of visual fields for colour and the responsibility of:
form 1. Pharmaceutical Council of India
4. Sudden failure of vision 2. Central drugs laboratory
3. Indian Medical Council
Ans. 1 4. Drug Technical Advisory Board
Clinical Features of Methanol Poisoning
- Headache, dizziness, Nausea and vomiting Ans. 1
- Pain in abdomen, Muscular weakness, Acidosis Central drug laboratory analyses imported and
Blurred vision, diminution of visual field temporary manufactured drugs to know their constituents, purity and
blindness or permanent blind in severe case ofpoisoning. potency.
Intestinal contraction is diagnostic of methanol poisoning. Drugs technical advisory Board advise the central and
Poisons producing constriction of pupils are the state government on different technical matters
following: related to the administration of the act.
- Opium, Physostigmine, Phenol, Chloralhydrate
- Organophosphorus compound
20
Pharmaceutical council of India has control and regulation Intent
regarding quality of drugs .
Drugs and cosmetics rule of 1945 regulates the import, 7-12 years Responsibility Sec. 83 IPC Guilty of
manufacture distribution and sale of drugs in India. offence
< 12 years Criminal Parents given
98. On MRI the differential diagnosis of spinal cord Responsibility 89 IPC consent for
edema is: Consent noninvasive
1. Mylodysplasia procedure
2. Myelomalacia
3. Myelochisis
4. Cord tumors 101. In a case of death due to burns, which of the
following statement is correct?
Ans. (2) 1. Red line of demarcation around the vesicle is
Both cord edema and myelomalacia appear as always present
On T1 Ill-defined hypointense area 2. Red line of demarcation around the vesicle is always
On T2 Hyperintense area absent
Cord Edema Myelomalacia I Cord 3. Red line of demarcation disappears soon after death
atrophy 4. Red line of demarcation, mayor may not form
Etiology Reflects In end stage of cord
focal trauma and result Ans. 1
accumulation release of vasoactive
of intracellular & substances & cellular
interstitial fluid in enzyme Antemortem Post mortem
response of
trauma. 1. Line of Present Absent
ischemia and redness
to injury 2. Vesicles Contain a Contain air
MRI Ill-defined Poorly marginated albuminous
T1WI hypointense area hypointense area fluid
T2WI Focus of Hyperintense cord
hyperintensity parenchyma
3. Infection Pus and Nil
Sloughing
99. Autoimmune haemolytic anemia is a feature of? 4.Healing Granulation Nil
1. ALL
2. CML
3. Burkett's Lymphoma 5. Carboxy Present Absent
4. Lymphoma Hb blood

Ans. 4
Causes of Autoimmune Hemolytic Anemias 6.Enzymes Increase in No such
Idiopathic (50%) enzymes increase
Drugs - Methyl dopa;
CLL, Lymphomas
M. Pneumoniae infection. 102. A 6 years old boy presented with Complex
IMN, SLE and other connective tissue disorders. seizures per day in spite of adequate 4 drug
antiepileptic regime. He had history of repeated
100. Nothing is an offence which is done by a child high-grade fever in childhood. MRI for epilepsy
under seven years of age' is included under which of protocol revealed normal brain scan. Which of the
the following section: following will not be helpful for functional imaging
1. Section 82 IPC of brain?
2. Section 83 IPC 1. MDCT
3. Section 84 IPC 2. SPECT imaging
4. Section 85 IPC 3. Interictal 18F-FDG PET
4. Video EEG with Ictal 99mTc- HMPAO study of brain
Ans. 1
Medicolegal importance of age Ans. (1)
Age Medicolegal IPC Ex planation Brain perfusion study using perfusion agents like Tc-99m
HMPAO, i.e., Tc-9W6HMPAO brain SPECT is indicated in
<7 year Criminal Sec.82 IPC No criminal
21
acute cerebral infarct imaging before evidence of CT/MRI too much air for the amount of blood at the apex; that is,
pathology, when positive findings are seen with 1 hour of the apex has a high VA/QC.
event. Brain metabolic rate and functional activity study by The more negative intrapleural pressure at the apex
using 18F FDG PET is indicated in focal epilepsy prior to distends airways causing the resistance to flow to be lower
seizure. at the apex than at the base.
MDCT is not useful for functional studies of brain.
105. Intramembranous ossification is the source of ?
103. Digoxin toxicity may result from the concurrent 1. A Flat bone
administration of digoxin with all of the following 2. B. Long bone
drugs EXCEPT 3. C. Cancellous bone
1. Quinidine 4. D. Bone of the epiphysis
2. Hydrochlorothiazide
3. Triamterene Ans. 1
4. Furosemide Intramembranous ossification is the development of bone
from mesenchymal condensation. Mesenchyme cells give
Ans. 3 rise to osteoblasts, which lay down bone matrix and
Triamterene is a potassium-sparing diuretic that may eventually form bone spicules. When the osteoblasts
protect against diuretic-induced digoxin toxicity. Digoxin become enclosed, they are called osteocytes. The frontal,
toxicity may be caused by drugs that increase serum parietal, squamous portion of the temporal, and parts of
digoxin levels or increase the binding of digoxin to its the occipital bone are derived from intramembranous
receptor, the sodium-potassium adenosine tn phosphatase ossification.
(ATPase). Quinidine decreases digoxin volume of Long bones develop from a cartilage model, which is called
distribution and clearance. Verapamil also decreases the endochondral ossification Growth in length occurs by
clearance of digoxin. Both drugs may division of cells in the epiphyseal plate. Cancellous bone is
thereby increase serum digoxin levels and precipitate spongy, or trabecular bone, which can result after either
digoxin toxicity. Diuretics (e.g., hydrochlorothiazide, intramembranous or endochondral ossification.
furosemide) may cause hypokalemia and
hypomagnesemia, both of which may predispose to cardiac 106. Which substance, if used for a long time is
arrhythmias. Furthermore, hypokalemia increases di-goxin associated with memory loss?
binding to sodium-potassium ATPase. 1. Marijuana
2. Alcohol
104. All of the following statement about a normal 3. Cocaine
subject in the erect position are true EXCEPT 4. LSD
1. Alveoli at the apex of the lung are less compliant that
those at the base Ans. 2
2. Airway resistance is greater at the apex than at the Alcohol amnestic disorder (Korsakoff syndrome) is the
base result of long-term alcohol abuse.
3. More gas exchange occurs at the base of the lungs than
at the apex 107. The oculomotor nerve passes between which pair
4. The ventilation/ perfusion ratio (VA/Qc) is lowest at artery?
the base of the lung 1. A Superior cerebellar artery and posterior cerebral
artery
Ans. 2 2. Posterior inferior cerebellar artery and anterior-
When the thorax is in the erect Position, the pressure cerebellar artery
generated by the right ventricle is not sufficient to perfuse 3. Posterior cerebellar artery and middle cerebral Artery
apical alveoli to the same extent as it perfuses alveoli at the 4. Posterior cerebral communicating artery
base of the lungs. Approximately IS times more blood
flows through the capillaries at the base than at the apex. Ans. 1
In addition, while erect, the more negative intrapleural The oculomotor nerve emerges from the midbrain and
pressure at the apex causes the overstretched alveoli in passes between the two cerebral peduncles. The nerve
this region to resist further stretch more strongly; i.e., the then passes between the superior cerebellar artery and the
compliance of the alveoli is less at the apex than at the posterior cerebral artery before entering the dura of the
base. Consequently, about four times more gas goes to the lateral wall of the cavernous sinus Because of the close
base than to the apex. Most of the gas exchange in the relationship between the oculomotor nerve and, the
lungs takes place at the base when the thorax is erect superior cerebellar, posterior cerebral, and basilar
because the base is the place where most of the blood and arteries: an aneurysm of any of these three arteries can
gas are located. However, there is too much blood for the compress the oculomotor nerve.
amount of air at the base; that is, the base has a low
ventilation/perfusion ratio (VAIQe). Conversely, there is 108. Uncontrolled flailing of an arm is a symptom of?

22
1. Amyotrophic lateral sclerosis innervate the infrahyoid strap muscles: sternohyoid,
2. Lower motor neuron syndrome sternothyroid, and omohyoid. The geniohyoid muscle is
3. Dysdiadochokinesias innervated by cervical nerve fibers that are carried with
4. Hemiballismus the hypoglossal nerve. The mylohyoid is innervated by a
branch of the mandibular division of the trigeminal nerve.
Ans. 4 The cricothyroid and cricoarytenoid muscles are intrinsic
Hemiballismus is associated with a lesion of the sub muscles of the larynx and are innervated by branches of
thalamic nucleus of the motor system. Lesions in this the vagus nerve.
nucleus produce uncontrolled flailing and involuntary
movement, especially of the upper extremity. Amyotrophic
lateral sclerosis, a disease of the lower motor neurons, 112. Which muscle has the phrenic nerve on its
produces weakness in skeletal muscles, which may anterior surface?
eventually lead to paralysis and muscle atrophy. 1. Sternocleidomastoid
Athetosis refers to slow writhing movements of the limbs. 2. Anterior digastric
This finding is characteristic of lesions in areas of the basal 3. Superior belly of the omohyoid
ganglia that are not subthalamic. 4. Anterior scalene

109. Which nerve is a branch of a root of Ans. 4


the brachial plexus? The phrenic nerve lies on the anterior surface of the
1. Suprascapular nerve anterior scalene muscle. The nerve is held tightly against
2. Dorsal scapular nerve the surface of the muscle by the deep cervical fascia The
3. Upper subscapular nerve roots of the brachial plexus and the subclavian artery pass
4. Lateral pectoral nerve posterior to the anterior scalene muscle and anterior to
the middle scalene muscle.
Ans. 2
The roots of the brachial plexus are the anterior rami of 113. A 58-year-old man has the sudden onset late one
spinal nerves Cs-T1. These five roots form the three trunks evening of severe pain in his left great toe. There is no
of the plexus. Each trunk divides into an anterior and a history of trauma. On examination there is edema with
posterior division. The divisions form the three cords of erythema and pain on movement of the left 1st
the brachial plexus. The dorsal scapular nerve is a branch metatarsophalangeal joint, but there is no overlying
of the C, root. The supra scapular nerve is a branch of the skin ulceration. A joint aspirate is performed and on
upper trunk. The upper subscapular nerve and the microscopic examination reveals numerous
thoracodorsal (middle subscapular) nerve are branches of neutrophils. Over the next 3 weeks, he has two more
the posterior cord. The lateral pectoral nerve is a branch of similar episodes. On physical examination between
the lateral cord. these attacks, there is minimal loss of joint mobility.
Which of the following laboratory test findings is most
110. In schizophrenia, hallucinations are to be? characteristic for his underlying disease process?
1. Visual 1. Hyperglycemia
2. Kinesthetic 2. Positive antinuclear antibody
3. Auditory 3. Hyperuricemia
4. Olfactory 4. Hypercalcemia

Ans. 3 CORRECT 3. Many patients with gout will have


In schizophrenia, the most common type of hallucination is hyperuricemia, but not all. The big toe is the most common
auditory. Other types of hallucinations, such as cenesthetic joint involved with gout. Acute gouty attacks are
(altered sensations of body organs), visual, olfactory, and associated with intense pain from acute inflammation.
kinesthetic, can also occur.
Incorrect 1. Hyperglycemia is a feature of diabetes
mellitus, which is associated with peripheral vascular
111. Which muscle receives its nerve supply from a disease that increases the risk for foot ulcerations and
direct branch of the ansa cervicalis ? osteomyelitis.
1. Mylohyoid Incorrect 2. The arthralgias common to many autoimmune
2. Geniohyoid diseases with a positive ANA, such as systemic lupus
3. C Sternohyoid erythematosus, are not usually associated with joint
4. Cricothyroid swelling, redness, or deformity.
Ans. 3 Incorrect 4. There can be bone luceny with
The ansa cervicalis is formed by nerve fibers from the hyperparathyroidism, but the joints are not greatly
anterior rami C It gives rise to nerves that affected, nor is there swelling or redness.

23
1. A Rapid supination and pronation of the
114. Which of the following bone hands- cerebellar function
disease/pathogenesis relationships is NOT CORRECT? 2. Stereognosis -sensory cortex
1. Osteogenesis imperfect: decreased mineralization of 3. Clasp knife reflex-upper motor neuron disease
bone 4. Romberg's test-lower motor neuron disease
2. Osteopetrosis: osteoclast dysfunction
3. Achondroplasia: premature closure of the epiphyseal Ans. 4
plate of long bones Romberg's test primarily evaluates joint position, which is
4. Postmenopausal osteoporosis: osteoclastic activity a function of the dorsal columns, the patient is asked to
predominates over osteoblastic activity stand with the feet together and eyes closed. Loss of
balance with the eyes closed that is corrected by opening
Ans. 1 the eyes indicates dorsal column disease (e.g., tabes
Osteogenesis imperfecta, or brittle bone disease, is the dorsalis). Loss of balance with the eyes open or closed
most common hereditary bone condition and, in most indicates a cerebellar problem, but that is not the primary
cases, is an autosomal dominant disease. Resulting from purpose of the test.
abnormal collagen synthesis, osteogenesis imperfecta
causes abnormalities in the skeleton (frequent pathologic 118. Avascular necrosis of bone is LEAST likely to be
fractures), eyes (blue sclera from too little collagen), ears associated with?
(hearing deficits), joints (laxity), and teeth (dentin 1. Osgood -Schlatter disease
deficiency causes blue-yellow discoloration) 2. Legg-Perthes disease
3. Long-term use of corticosteroids
115. Which is LEAST likely to characterize anxiety? 4. Sickle-cell disease
1. Syncope
2. Palpitations Ans. 1
3. GIT disturbances Avascular necrosis is an infarction of bone that is
4. Flight of ideas associated with (1) long-term corticosteroid use, such as in
patients with systemic lupus erythematosus (most
Ans. 4 common cause), (2) sickle-cell disease, (3) trauma, (4)
Anxiety is characterized by restlessness, dizziness, Legg-Perthes disease, or (5) Kienbock disease involving
palpitations, syncope, tingling in the extremities, tachy the scaphoid (navicular) bone. OsgoodSchlatter dis ease is
cardia, tremor, gastrointestinal disturbances, as well as a localized inflammation that produces pain in the tibial
urinary urgency and frequency. Flight of ideas (thoughts tuberosity at the point of attachment of the patellar
following each other in quick succession) is characteristic tendon, which results in prominence of the tibial
of mania. tuberosity that persists throughout life. It usually occurs
during the pubertal growth spurt and is not an example of
116. Characteristics of smooth and cardiac muscle avascular necrosis.
include all of the following EXCEPT?
1. Gap junctions 119. Which of the following is the LEAST likely
2. Central nuclei symptom of hyperthyroidism in a 77-year-old man?
3. Branching fibers 1. Atrial fibrillation
4. Perinuclear organelles 2. Confusion
3. Tremor
Ans. 3 4. Weakness
Cardiac muscle consists of long, branching fibers whereas
smooth muscle cells or fibers are unbranched. Cardiac Ans. 3
muscle is striated and resembles skeletal muscle in its One of the most important principles of caring for the
arrangement of contractile elements. Cardiac fibers that older patient is the recognition that diseases present in an
are specialized to conduct impulses are called Purkinje atypical fashion. For example, hyperthyroidism in a young
fibers. Central nuclei, one per cell, are common to smooth or middle-aged individual usually manifests as tremor,
and cardiac muscle. Skeletal muscle has multiple nuclei per goiter, or exophthalmos. Instead, this endocrinopathy in an
cell. older person is more likely to cause one of the "final
Gap junctions are a component of the cardiac muscle common pathway" symptoms such as weakness, fatigue,
intercalated disk, in addition to desmosomes and a fascia syncope, confusion, or depression Atrial fibrillation is also
adherens (zonula adherens) Gap junctions join smooth a more typical presentation of hyperthyroidism in this age
muscle cells, allowing for a wavelike spread of contraction. cohort than any of the classic symptoms or findings. The
differential diagnosis of a limited number of presenting
117. Which one of the following associations is NOT symptoms in the elderly is both broad and similar amongst
CORRECT?' several syndromes: incontinence, falling, syncope, and
confusion. More over, due to decreased physiologic

24
reserve in the elderly, patients often present at an earlier 1. Adequate analgesia
stage of the disease. 2. Reduced blood loss
3. Reduced stress response of surgery
120. The pressure used in pneumatic tourniquet in 4. Slow onset
upper limb is upto:
1. Systolic Blood Pressure + 50 Ans. 4
2. Systolic Blood Pressure + 75 Advantages of epidural anaesthesia
3. Systolic Blood Pressure + 100 - Profound analgesia
4. None of the above - Profound muscle relaxation
- No need for intubation
Ans. 2 - Reduction of surgical haemorrhage
Pneumatic tourniquet is a narrow rubber bag to be wound - Excellent post operative analgesia
around a limb, pressure being applied by pumping air into - Reduced stress and response
the inflatable cuff.
A bloodless field is obtained by applying pressure through 125. Which of the following inhalational agent is the
pneumatic tourniquet for accurate surgery. Usually most soluble in blood?
pressure applied 75 mmHg over systolic blood pressure 1. Halothane
for not more than 2 hours. 2. Isoflurane
3. Enflurane
121. In hypotensive patients, Induction of 4. Desflurane
anaesthesia should be done with:
1. Propofol Ans. 1
2. Ketamine Blood/gas partition coefficient: The ratio of concentration
3. Thiopentone in blood to that in gas. High value denotes an agent very
4. Any of the above soluble in blood.
Inhalational agent Blood/gas partition
Ans. 2 coefficient
Ketamine produce a state of Dissociative anaesthesia Halothane 2.30
ketamine increase systolic and diastolic blood pressure, Isoflurane 1.40
increases heart rate and intracranial tension. Enflurane 1.90
Ketamine is good as induction agent for hypotensive Desflurane 0.42
patient. Sevoflurane .69

122. Induction agent, having anti-emetic property Most soluble in blood is halothane.
is: Least soluble in blood is desflurane.
1. Thiopentone
2. Methohexitone 126. The only condition associated with X-linked
3. Propanidid recessive inheritance is:
4. Propofol 1. Phenylketonuria
2. Mucopolysaccharidosis
Ans. 4 3. Achondroplasia
Propotol is used as induction agent, it cause decrease in 4. Marfan's syndrome
blood pressure heart rate and vascular resistance.
Nausea and vomiting is very rare (1 -2%), and less than . Ans. 2
with most other used agents. 1. X-linked recessive inheritance
Hemophilia-A
123. Laryngospasm under anaesthesia can be Muscular dystrophy
relieved by: Mucopolysaccharidosis type II (Hunter syndrome)
1. Aminophylline Colour blindness
2. Salbutamol G6PD deficiency.
3. Terbutaline
4. Succinylcholine 2. X-linked dominant inheritance
Hypophosphatemic rickets
Ans. 4 Incontinentia pigmenti.
Suxamethonium is depolarizing relaxant, used to revert
the spasm under anaesthesia. 3. Autosomal recessive inheritance
Sickle cell anaemia
124. Advantage of epidural anaesthesia includes all 0-thalassemia
except: Wilson's disease

25
Xeroderma pigmentosa Despite the history of poor intake of oral fluid, this infant
Inborn error of metabolism (Alkaptonuria, presents with signs of congestive heart failure and fluid
Phenylketonuria, Lysosomal storage disease) overload. In view of these findings, a fluid challenge could
Galactosemia. be dangerous, and it is unlikely that she would respond. If
the oliguria is the result of congestive heart failure and
4. Autosomal dominant inheritance poor renal perfusion, the urine sodium concentration and
Adult polycystic kidney fractional excretion of sodium should be low, the blood
Neurofibromatosis urea nitrogen and serum creatinine should be normal or
Marfan's syndrome (long thin extremities, ectopia slightly elevated, and the child may respond well to
lentis, aortic aneurysm) furosemide.
Osteogenesis imperfecta
Familial hypercholesteremia 129. Proper management of a child with an
A chondrodysplasia and familial polyposis coli. absolute neutrophil count (ANC) of 100/mm3
would be
127. Typical febrile seizures are characterized by all 1. The start of broad-spectrum intravenous
of the following except. antibiotics
1. Commonest cause of seizures In children 2. Careful physical examination and chest
2. Seizures do not last for more than minutes radiograph; close observation pending result of
3. E.E.G. shows persistent abnormality blood cultures
4. No postictal neurological deficit t 3. Granulocyte transfusion
4. Nutritional support with oral iron and
Ans. 3 intramuscular injection or vitamin B 12
Febrile convulsions are the commonest cause of seizures
during early childhood. Ans. 1
These convulsions are not related to degree of A child with an absolute neutrophil count (ANC) of
temperature but frequent if temperature rises abruptly. 100/mm3 is severely neutropenic and is at risk of death
The Febrile convulsion from overwhelming septicemia. Because most signs of
i. Simple or benign infection (e.g., pneumonitis, pyuria) depend on the
ii. Complex or atypical presence of neutrophils, serious infection may be present
Characteristics of simple benign febrile convulsion in such a patient without producing characteristic physical,
The fits occur within 24 hours of onset of fever last less radiologic, or laboratory findings. Thus, broad spectrum
than 10 minutes antibiotics (to treat a variety of possible infections) must
Convulsions are generalized (mostly not always) be initiated promptly after cultures are obtained. Deferring
No postictal neurological deficit treatment until culture results are available may result in
EEG (electroencephalogram) is normal after few days of overwhelming septicemia and death. Granulocyte
seizure transfusions usually are not beneficial in these patients
The age of onset usually between 6 months and 5 years. because of the short survival time of the cells in the
Atypical febrile seizures: bloodstream (plasma half-life is approximately 7 hours).
Characterised by presence of family history of epilepsy. Iron and vitamin B12 are treatments for nutritional
Neurodevelopment retardation anemias and are of no therapeutic' value in neutropenia.
Convulsion due to organic neurological damage are
precipitated by fever, as the cerebral threshold for seizures 130. The most likely diagnosis is
is reduced with elevation of temperature. 1. Histiocytosis X
2. Rhabdomyosarcoma
128. A 15-month-old girl has a history of poor oral 3. Neuroblastoma
fluid intake, occasional vomiting, rapid breathing, 4. Wilms tumor
and decreased urine output. Physical examination
reveals a pulse of 150/min, BP of 120/80, and a Ans. 3
respiratory rate of GO/min. There are bibasilar This is a typical presentation of stage IV neuroblastoma,
rales, and the liver is palpable. All procedures with the primary tumor arising from orie of the
might be helpful in evaluating the oliguria EXCEPT paravertebral sympathetic ganglia and metastasizing to
1. Giving a fluid challenge with isotonic saline, 20 bone marrow, cortical bone, and the retro orbital tissues.
ml/kg Histiocytosis X may have a somewhat similar presentation
2. Determining the urine sodium concentration and involve multiple bones, but it usually involves one
3. Determining the BUN and serum creatinine levels rather than both orbits; the right flank mass and tumor
4. Giving a dose of intravenous furosemide clumps in the marrow of this patient also are not
characteristic" of histiocytosis X. Rhabdomyosarcoma may
Ans. 1 arise in the orbit but usually is unilateral; the tumor also
rarely is associated with systemic metastases at initial

26
diagnosis. Wilms tumor would produce a right renal rather
than suprarenal mass and would metastasize to the lungs Ans. 4
rather than to bone and bone marrow. Lymphoblastic Occupational exposure to arsenic may Occur in the
lymphoma usually arises in the anterior mediastinum or smelting industry (due the presence of arsenic as a
peripheral node's. Even if it were to present with byproduct of purifying ores) and the microelectronics
widespread dissemination, abdominal rather than thoracic industry due to the use of gallium arsenate. Inorganic
disease would be unusual and Lymphoblasts rather than arsenic (currently used successfully and safely for the
tumor clumps would be seen in the marrow. treatment of patients with acute promyelocytic leukemia)
is more toxic than organic arsenic. Arsenic is rapidly
131. A newborn infant with ambiguous genitalia is cleared form the GI tract, kidneys, and lungs, where it
found to have a 46, XX karyotype All of the originally resides, but leaves a long-term residue in the
following are diagnostic possibilities Except integument. Acute arsenic toxicity manifests as increased
1. 21-hydroxylase deficiency vascular permeability and intestinal inflammation.
2. Partial androgen resistance syndrome Cardiomyopathy may be seen with chronic exposure, with
3. True hermaphroditism conduction system disease, including QRS widening, ST
4. Maternal virilizing tumor prolongation, and multifocal atrial tachycardia. Treatment
of chronic poisoning should include chelation with
Ans. 2 dimercaprol.
Infants with partial androgen resistance may have
ambiguous genitalia, but the karyotype. is 46,XY. These 134. A 45-year-old man present7 with a daily
infants have testes and high levels of testosterone, which headache over the past 3 weeks. Each attack lasts
are incompletely effective in Virilizing the external about an hour and awakens the patient from sleep.
genitalia in a normal male pattern. The defect is at the level It is associated tearing and reddening of his right
of the androgen receptor. Female infants with congenital eye. The pain is deep, excruciating, and limited to
adrenal hyperplasia due to 21-hydroxylase deficiency may the right side of the head. The neurologic
exhibit a variable degree of masculinization of the external examination is normal. The most likely diagnosis:
genitalia because of prenatal exposure to high levels of 1. Migraine headache
circulating adrenal androgens' Most infants with true 2. Cluster headache
hermaphroditism have a 46,XX karyotype. However, 3. Tension headache
gonadal tissue includes both ovarian and testicular 4. Brain tumor
elements. The testicular androgens cause external
virilization, leading to ambiguous genitalia. In a similar Ans. 2
fashion, a maternal Virilizing tumor may expose a female Cluster headaches, which can cause excruciating
fetus to high levels of androgen. hemicranial pain, are notable for their occurrence during
characteristic episodes. Usually attacks occur during a 4-
132. All of the following are characteristics of Rett to 8-week period in which the patient experiences one to
syndrome EXCEPT three severe brief headaches daily. There may then be a
1. Autistic behavior prolonged pain-free interval before the next episode. Men
2. Microcephaly between ages 20 and 50 are most commonly affected. The
3. Peculiar wringing motion of the hands unilateral pain is usually associated with lacrimation, eye
4. Autosomal recessive inheritance reddening, nasal stuffiness, ptosis, and nausea. During
episodes alcohol may provoke the attacks. Even though the
Ans. 4 pain caused by brain tumors may awaken a patient from
The genetics of Rett syndrome has not yet been sleep, the typical history and normal neurologic
determined. Although it is thought to be a genetic syn- examination do not mandate evaluation for a neoplasm of
drome, no inheritance pattern and no enzymatic deficiency the central nervous system. Acute therapy for a cluster
or other metabolic explanation has been established for headache attack consists of oxygen inhalation, although
Rett syndrome. The other characteristics (autistic intranasal lidocaine and subcutaneous sumatriptan may
behavior, microcephaly, peculiar wringing motion of the also be effective. Prophylactic therapy with prednisone,
hand, dementia) are common in patients with this lithium, methysergide, ergotamine, or verapamil can be
disorder. administered durinq an episode to prevent further cluster
headache attacks.
133. Which of the following is the most likely
associated laboratory abnormality in a patient 135. Which of the following colonic pathologies is
suffering from arsenic poisoning? thought to have no malignant potential?
1. Neutropenia 1. Ulcerative colitis
2. Macrocytic anemia 2. Villous adenomas
3. Normocytic anemia 3. Familial polyposis
4. Prolongation of the QT interval 4. Peutz-Jeghers syndrome

27
and malignant lesions of bone, but this does not explain
Ans. 4 the hypergammaglobulinemia.
Peutz-Jeghers syndrome is characterized by intestinal Incorrect 2. Fibroblasts produce collagen and are more
polyposis and melanin spots of the oral mucosa. Unlike the numerous with the gross appearance of firm, white scar
Adenomatous polyps seen in familial polyposis, the lesions tissue.
in this condition are hamartomas, which have no Incorrect 3. Osteoblasts are most numerous in repair of
malignant potential. bone, and callus is very firm.

136. During an operation for carcinoma of the hepatic 138. Mixed tumors of the salivary glands are
flexure of the colon, an unexpected discontinuous 3- characterized by which of the following?
cm metastasis is discovered in the edge of the right 1. They occur most commonly on the lips, t09gue, and
lobe of liver. The surgeon should palate
1. Terminate the operation, screen the patient for 2. They grow rapidly
evidence of other metastases, and plan further therapy 3. They rarely recur if simply enucleated
after the reevaluation 4. They present as rock-hard masses
2. Perform a right hemicolectomy and right hepatic
lobectomy Ans. 1
3. Perform a right hemicolectomy and a wedge resection There are approximately 400 to 700 minor salivary glands
of metastasis in the oral cavity. Pleomorphic adenomas (mixed tumors)
4. Perform a cecostomy and schedule reoperation after a can occur in any of them. These round tumors have a
course of systemic chemotherapy rubbery consistency and are slow-growing; all are
potentially malignant. Unless adequately excised, they tend
Ans. 3 to recur locally in a high percentage of cases. The sites
Because approximately 5% of colorectal cancers are most commonly affected by pleomorphic adenomas of the
associated with resectable hepatic metastases, appropriate salivary glands are the lips, tongue, and palate.
preoperative- discussion should include obtaining
permission for removal of synchronous peripheral hepatic 139. The primordial germ cells that eventually from ,
lesions if they are found. If gross tumor is removed, a 25% the oogonia and spermatogonia originate in which of
cure rate can be anticipated. Adequate local resection, the following
either by wedge or by limited partial hepatectomy, may be 1. Dorsal mesentery of the hindgut
carried out whenever no extrahepatic disease is found and 2. Gonadal ridge
the hepatic lesion is technically removable. Any option that 3. Endodermal lining of the yolk sac
leaves the potentially obstructing primary cancer 4. Primary sex cords of the developing gonad
unremoved would be unacceptable.
Ans. 3
137. A 62-year-old man has had back pain for 4 The primordial germ cells are first seen in the endodermal
months. No abnormal findings are noted on physical lining of the wall of the yolk sac (derived from the
examination. A CBC shows a WBC count of hypoblast) at the end of the third or beginning of the
3700/microliter, hemoglobin 10.3 g/dL, hematocrit fourth week in the region of the allantois. During
31.1%, MCV 85 fL, and platelet count embryonic folding, the dorsal part of the yolk sac is
110,000/microliter. His total serum protein is 8.5 incorporated into the embryo as the primitive gut. The
gm/dl with an albumin of 4.1 gm/dl. A chest primordial germ cells subsequently migrate along the
radiograph shows no abnormalities of heart or lung dorsal mesentery of the hindgut and into the gonadal
fields, but there are several lucencies noted in the (genital) ridge by week 6. The primary sex cords grow into
vertebral bodies. A sternal bone marrow aspirate is the mesenchyme underlying the ridge, and the primordial
performed and yields a dark red jelly-like material in germ cells become incorporated into the primary sex
the syringe. Which of the following cell types is most cords. The chorion is the outermost fetal membrane and is
likely to be numerous on microscopic examination of composed of extraembryonic somatic mesoderm,
this aspirate? cytotrophoblast, and the syncytiotrophoblast. It is divided
1. Giant cells into the chorion frondosum, where the villi form and
2. Fibroblasts proliferate. and the smooth chorion, also known as the
3. Osteoblasts chorion laevae. The following diagram illustrates the
4. Plasma cells arrangement of the fetal membranes.

CORRECT 4. The patient has multiple myeloma. The bone The primordial germ cells are the common
marrow needle was in a lytic lesion filled with plasma cells. origins of spermatozoa and oocytes and thus
His serum globulin is high as a result of his monoclonal represent the ancestors of the germline. Like all
gammopathy. other somatic cells these are diploid and in
Incorrect 1. Giant cells may be seen in a variety of benign human embryos can already be found in the

28
primary ectoderm (epiblast) in the second NH2 amide nitrogen of asparagine. 0linked
week. oligosaccharides have sugar residues linked to hydroxyl
groups on the side chains of serine and threonine and are
140. Members of the multi pass, G protein-linked less common than the N-linked species.
family of receptors include the muscarinic cholinergic
and J3-adrenergic receptors and rhodopsin. Which of 142. Meiotic crossover occurs in which of the
the following characterizes these receptors? following?
1. They possess a single hydrophobic transmembrane 1. Leptotene
segment in the form of an a helix 2. Zygotene
2. They can activate plasma membrane-bound enzymes 3. Pachytene
or ion channels 4. Diplotene
3. They posses an intracellular ligand-binding domain
4. They possess intrinsic enzyme activity Ans. 3
Crossover occurs during the pachytene stage of meiosis.
Ans. 2 Meiosis is the mechanism used by the reproductive organs
Binding of ligand to the multipass G protein-linked to generate gametes-cells with the haploid number of
receptors activates or inactivates enzymes bound to the chromosomes. DNA synthesis occurs before meiotic
plasma membrane (adenylyl cyclase or phospholipase C) prophase begins and is followed by a G2 phase. Cells then
or opens or closes ion channels using G proteins. A table of enter meiotic prophase I. During meiotic prophase I,
G proteins and their functions appears below. The maternal and paternal' chromosomes are precisely paired,
muscarinic cholinergic and receptors, as well as and recombination occurs in each pair of homologous
rhodopsin, are multipass transmembrane proteins chromosomes. The first meiotic prophase consists of five
consisting specifically of seven hydrophobic spanning substages: leptotene, zygotene, pachytene, diplotene, and
segments of the single polypeptide chain. The peptide diakinesis. During metaphase I, there is random
bonds of the spanning segments are polar. In the segregation of maternal and paternal chromosomes.
hydrophobic environment of the lipid bilayer, in the Homologous chromosomes are aligned on the metaphase
absence of water, they form hydrogen bonds with each plate of the meiotic spindle in metaphase I. The second
other. In the vast majority of cases this leads to the meiotic division is responsible for the reduction in the
formation of ex helices. In the case of the porins, found in chromosome content of the cell by 50%. In meiotic
the outer mitochondrial membrane, a 0-barrel structure is division II, metaphase consists of daughter chromatids of
established. There is a remarkable homology between the single homologous chromosomes aligned on a metaphase
cell-surface receptors linked to the G proteins. Between plate (metaphase II). Condensation of the chromatids
the segments, the polypeptide chain loops on both the occurs in leptotene. In zygotene, the synaptonemal
extracellular and the intracellular sides of the membrane. complex begins to form, which initiates the close
Ligand binding occurs on the extracellular surface. association between chromosomes known as synapsis. The
Receptors with intrinsic enzyme activity belong to a bivalent is formed between the two sets of homologous
separate class of singlepass transmembrane proteins. All chromosomes (one set maternal and one set paternal
of these transmembrane proteins show a carboxyl equals a pair of maternal chromatids and a pair of paternal
terminus on the cytosolic side and N-linked glycosylation chromatids). The four chromatids form a tetrad (bivalent).
sites on the extracellular surface. Pachytene begins as soon as the synapsis is complete and
includes the period of crossover. The fully formed
141. Which of the following events occurs in the rough synaptonemal complex is present during the pachytene
endoplasmic reticulum? stage. At each point where crossover has occurred
1. Core glycosylation of proteins between two chromatids of the homologous chromosomes,
2. O-linked glycosylation an attachment point known as a chiasma forms. The
3. Sulfation formation of chiasmata and de synapsing (separation of
4. Protein sorting the axes of the synaptonemal complex) occurs in the
diplotene stage. Diakinesis is an intermediate phase
Ans. 1 between diplotene and metaphase of the first meiotic
The rough endoplasmic reticulum (RER) is the site of core division.
glycosylation of proteins using the membrane-bound lipid
carrier, dolichol, catalyzed by an oligosaccharide 143. Vitamin C deficiency results in which of the
transferase. This is N-linked glycosylation, which occurs by following?
an en bloc method in which dolichol is added to the 1. Decreased degradation of collagen
protein. O-linked glycosylation occurs in the Golgi (answer 2. Stimulation of poly hydroxylase
b) by a mechanism involving oligosaccharide (glycosyl) 3. Formation of unstable collagen helices
transferases rather than en bloc. N linked oligosaccharides 4. Excessive callus formation in healing fractures
are the most common oligo saccharides found in
glycoproteins and contain sugar residues linked to the Ans. 3

29
Scurvy, or vitamin C deficiency, results in an inability to Ans. 2
form normal collagen triple helices. In scurvy, the resulting The secretory phase of the menstrual cycle depends on
collagen is less stable and is subject to denaturation and progesterone secretion and follows the proliferative
proteolytic breakdown. That results partially from slower (follicular) phase. The menstrual phase occurs after the
secretion of collagen from fibroblasts. The collagen formed secretory phase. During the follicular phase
is not normally hydroxylated at proline and lysine residues (approximately days 4 to 16), estrogen produced by the
because of the absence of vitamin C, which is a specific ovaries drives cell proliferation in the base of endometrial
cofactor for hydroxylation of proline and lysine. Bone glands and the uterine stroma. The proliferative phase
growth, development of the dentition, and wound and culminates with ovulation. The secretory phase
fracture healing as well as the general stability of adult (approximately days 16 to 25) is characterized by high
organs are inhibited because of the importance of collagen progesterone levels from the corpus luteum, a tortuous
in the maintenance of structural support. Periodontal appearance of the uterine glands, and apocrine secretion
bleeding and ulceration are also common symptoms in by the gland cells. During this phase, maximum
scurvy. endometrial thickness occurs. The menstrual phase
(approximately days 26 to 30) is characterized by
144. In a positive tuberculin skin test, helper T cells decreased glandular secretion and eventual glandular
assist in which of the following ways? degeneration because of decreased production of both
1. Autocrine-mediated inhibition of proliferation of progesterone and estrogen by the theca lutein cells.
helper T cells Contraction of coiled arteries and arterioles leads to
2. The down regulation of IL-2 receptors on helper T ischemia and necrosis of the stratum functionale.
cells
3. Secretion of interleukins that promote T cell 146. Which of the following is directly involved in
proliferation sound transduction?
4. Secretion of IL-I 1. Release of neurotransmitter onto the afferent endings
of cranial nerve VIII
Ans. 3 2. Shearing motion of the basilar membrane against hair
In a tuberculin skin test, T cell proliferation is increased by cell stereocilia
secretion of interleukins. An extract of tuberculin (an 3. Movement of the tectorial membrane resulting in hair
antigen of lipoprotein composition obtained from the cell depolarization
tubercle bacillus) is injected into the skin of a person who 4. Vibration at the round window via the stapes
has had tuberculosis or has been immunized against
tuberculosis. Merriory helper T cells react to the Ans. 1
tuberculin and secrete Il-2, which up regulates Il-2 Shearing of the hair cell stereocilia against the tectorial
receptors. Il-2 binding to Il-2 receptors on the same cell is membrane results in depolarization, and the release of
an example of autocrine regulation in which a cell secretes neurotransmitter onto afferent endings of the auditory
a ligand for a receptor on its own surface. The result of this cranial nerve leads to initiation of an action potential.
up regulation and ligand receptor binding is an increase in Sound waves are directed toward the tympanic membrane
T cell proliferation. T cell derived cytokines such as tumor by the pinna and the external auditory canal of the
necrosis factor-alpha and beta (TN F and ) induce external ear. The vibration of the tympanic membrane is
leukocyte recruitment. Production of gamma-()- transmitted to the oval window by way of the ossicles of
interferon by helper T cells attracts and activates the middle ear. Induction of waves in the perilymph results
macrophages (monocytes comprise most of the cellular in the movement of the basilar and vestibular membranes
infiltrate). -Interferon also converts other cells (such as toward the scala tympani and causes the round window to
endothelial cells) to antigen-presenting cells by induction bulge outward. The movement of the hair cells is
of class II MHC expression, which further augments the facilitated because the tectorial membrane is rigid and the
response. The result of the activity of helper T cells is a pillar cells form a pivot. The stabilization of the pressure
dramatic increase in the number of lymphocytes and between the middle ear and the nasopharynx is not
macrophages at the test site, which produces swelling. IL-I directly related to the mechanism of sound transmission.
is synthesized by antigen-presenting cells and macro
phages with helper T cells as the targets. 147. A patient has extremely enlarged palatine tonsils.
You suggest surgical removal of the tonsils, but you do
145. Which of the following are characteristic of the explain that there is a small risk of the surgery, which
secretory phase of the menstrual cycle? may result in which of the following?
1. It precedes ovulation 1. Loss in the ability to taste salt in the anterior two-
2. It depends on progesterone secretion by the corpus thirds of the tongue
luteum 2. Loss in the ability to protrude her tongue, thus limiting
3. It coincides with the development of ovarian follicles her ability to lick an ice cream cone
4. It coincides with rapid drop in estrogen levels

30
3. Weakness in the ability to open her mouth fully when 2. The atria are represented by cranial portions of
eating an apple due to damage to the innervation to endocardial tubes
the lateral pterygoid muscle 3. The heart bends into an S-shape because the caudal
4. Loss in the ability to taste in the posterior one-third of regions of the endocardial tubes grow faster than the
the tongue and perhaps some difficulty in swallowing cranial regions
4. The left and right sides of the heart result directly
Ans. 4 from the side-by-side apposition
The palatine tonsil sits in the lateral wall of the
oropharynx in the palatine arch posterior to the Ans. 3
palatoglossus muscle and anterior to the The heart forms during the third week by the apposition of
palatopharyngeus muscle. In the bed of the palatine tonsil left and right endocardial tubes as the head fold progresses
runs the glossopharyngeal CN (IX) that carries afferent caudally The endocardial tubes fuse to form a single-tube
information back to the brain regarding both general heart. This fusion begins cranially in the region of the
sensation and the special sense of taste from the posterior bulbus cordis (outflow trunks) and proceeds caudally
one-third of the tongue. The glossopharyngeal nerve is at through the ventricles and the atria to the sinus venosus,
risk for being cut during tonsillectomy. The ability to taste which is incorporated into the atrium after loop formation.
in the anterior two-thirds of the tongue is not at risk Rapid proliferation of the ventricular region results in the
because that information is carried by the lingual nerve, single-tube heart bending into an S-shaped loop. During
below the tongue. The ability to protrude your tongue is this process, the dorsal mesocardium partially breaks
provided by innervation from the hypoglossal nerve, down, which leaves the heart suspended only at the cranial
which innervates all the intrinsic tongue muscles and lies and caudal ends; the discontinuity in the mesocardium is
below the tongue and is not a risk. Neither the ability to the transverse sinus. The left and right sides of the heart
open your jaw wide nor to move your jaw from side to side are established by the subsequent division of the single
is controlled by the mandibular division of the trigeminal tube heart, not by the apposition of left and right
CN (V), which does not course near the palatine arch and endocardial tubes.
would not be at risk.
150. All of the following statements about keloids are
148. Which muscle is a pure elevator of the jaw? true except:
1. Buccinator muscle 1. A keloid does not regress spontaneously
2. Geniohyoid muscle 2. A keloid extends beyond the boundaries of the original
3. Lateral pterygoid muscle wound.
4. Medial pterygoid muscle 3. Keloids or hypertrophic scars are best managed by
excision and careful reapproximation of the wound
Ans. 4 4. Keloid tissue contains an abnormally large amount of
The medial pterygoid muscle, which originates on the collagen
medial side of the lateral pterygoid plate, and the masseter
muscle, which originates from the zygomatic arch, pass Ans. 3
medially and laterally to the ramus of the mandible to form Keloids and hypertrophic scars both occur aftet injuries
a sling about the angle of the mandible. These muscles are operations. Hypertrophic scars remain confined to the
powerful elevators of the jaw. The muscle bundles of the original injury site and usually regress spontaneously. By
anterior portion of the temporalis muscle run nearly contrast, keloids extend beyond the original wound and
vertically into the coronoid process of the mandible, acting rarely regress. Keloids contain abnormally large amounts
as a jaw elevator. The lateral pterygoid muscles run from of soluble collagen and water. Keloids tend to recur after
the lateral side of the pterygoid plate and from the excision, and any surgical therapy should be undertaken
infratemporal fossa to the head of the mandible and the cautiously with the patient aware that the result may not
articular disk of the temporomandibular joint. Contraction be ideal.
of the lateral pterygoid muscles bilaterally protrudes the
jaw. Unilateral contraction swings the jaw toward the 151. A 20-yrs-old man suffers an injury to the
opposite side. The submental muscles, assisted by gravity, posterior urethra. After appropriate initial
are the primary depressors of the jaw. These include the management and follow-up care of this injury, the
geniohyoid and mylohyoid muscles as well as the anterior most likely late complication is:
belly of the digastric muscle, all of which function in 1. Ascending urinary tract infection
conjunction with the infrahyoid strap muscles. 2. Retrograde ejaculation
3. Sterility
149. Which of the following statements is true of 4. Urethral stricture
cardiac development?
1. During formation of the heart loop, a single-tube heart Ans. 4
remains suspended by a complete dorsal Initial management involves passage of catheters through
mesocardium (mesentery) the urethral meatus and through an incision in the bladder.

31
After this has been accomplished, a Foley catheter is 154. A 48-yrs-old male alcoholic with bilateral
threaded through the urethra into the bladder and left in gynecomastia. Which physical findings has cirrhosis
situ. Urethral healing occurs as the hematoma resolves, but has the same pathogenesis as gynecomastia?
a posterior urethral stricture is not uncommon. If the 1. Caput medusa
stricture is not managed appropriately, ascending urinary 2. Asterixis
tract infection may result. 3. Ascites
4. Spider angioma
152. Which of the following changes is most likely to
occur following administration of an ACEI? Ans. 4
1. Increased levels of rennin Gynecomastia in male cirrhotics is caused by
2. Increased levels of angiotensin hyperestrinism secondary to the inability of the liver to
3. Increased levels of aldosterone metabolize estrogens. Hyperestrinism results in the
4. Increased levels of ADH development of female secondary sex characteristics,
palmar erythema, spider angioma, and testicular atrophy
Ans. 1 caused by decreased testosterone.
Inhibition of angiotensin-converting enzyme (ACE)
reduces angiotensin II formation. As a result, those 155. Decreased activity of type I 5'-monodeiodinase
pathways that are stimulated by angiotensin II are also could lead to which physiologic effects:-
reduced. The most important change is a reduction in the 1. Increased plasma triiodothyronine (T3)
release of aldosterone. To a lesser extent, angiotensin II 2. Increased plasma reverse T3
also causes vasoconstriction and release of transmitter 3. Decreased plasma thyroxine T4
from sympathetic nerves, and these direct excitatory 4. Increased TSH
effects are also reduced. Finally, angiotensin II has some
direct excitatory effect on antidiuretic hormone (AD H) Ans. 2
release. Lowering angiotensin II may therefore slightly Type I 5'-monodeiodinase is the enzyme responsible for
reduce ADH levels by this direct mechanism. These effects the peripheral conversion of thyroxine (T4) to
tend to reduce blood pressure and volume. This reduction triiodothyronine (T3), primarily in liver and kidney. This
is sensed by pressure and volume sensors throughout the enzyme also participates in the breakdown of reverse T3
body, leading to enhanced renin release. The effect of ACE (rT3) by converting it to 3,3'-diiodothyro nine. Decreased
inhibitors on ADH levels is complicated. On one hand, activity of type I 5'-monodeiodinase will therefore
angiotensin II has a small direct stimulatory effect on ADH decrease plasma levels of T3 and increase plasma levels of
release, so blockade of angiotensin II synthesis tends to rT3 (by decreasing its breakdown). Type II 5'-
limit ADH release. Furthermore, to the extent that more monodeiodinase is present in the anterior pituitary and
salt than water is lost, plasma osmolaliry decreases, central nervous system. Conversion of T 4 to T 3 within the
leading to a de- crease in ADH levels. On the other hand, anterior pituitary by this enzyme is responsible for normal
any reduction in extracellular fluid volume tends to feedback inhibition of thyroid stimulating hormone (TSH)
enhance ADH levels. In summary, only minor changes in secretion. With decreased type I 5'-monodeiodinase
ADH levels are expected whereas a significant rise in activity, plasma levels ofT4 and TSH usually stay within
plasma renin levels occurs. normal limits because the type II enzyme continues to
insure normal feedback inhibition by anterior pituitary T
153. The ureter lies against the anterior surface of 3.
which of the following muscles?
1. Crus of the diaphragm 156. A pregnant woman with the recent onset of
2. Quadratus lumborum urinary frequency, urgency and dysuria, and evidence
3. Psoas major of bacteriuria should be treated with:
4. Transversus abdominis 1. Ciprofloxacin
2. Norfloxacin
Ans. 3 3. Trimethoprim-sulfamethoxazole
The ureter exits the renal pelvis at about the level of 4. Cephalexin
vertebra L2. As it descends along the posterior abdominal
wall, it lies on the anterior surface of the psoas major. The Ans. 4
psoas major muscle arises from the bodies of the lower Tetracyclines may cause staining of fetal teeth when given
lumbar vertebrae. The psoas major muscle is joined by the during pregnancy. Trimethoprim sulfamethoxazole may
iliacus to form the iliopsoas muscle. The iliopsoas muscle cause neonatal jaundice when administered late in
then attaches to the lesser trochanter of the femur and is pregnancy because sulfonamides may displace bilirubin
the major flexor of the hip. from plasma proteins. Fluoroquinolones, such as
ciprofloxacin and norfloxacin, are best avoided in
pregnancy because of their potential to cause arthropathy
in the fetus caused by damage to cartilage. The 13-lactam

32
antibiotics, including penicillin's such as ampicillin and 3. Increased interstitial pressure
various oral cephalosporin's, may be safely administered 4. Decreased intrarenal nitric oxide
to pregnant women. First- or second-generation
cephalosporins are usually preferred because of their Ans. 1
antimicrobial spectrum. The renal alteration that would most likely preserve the
GRF is type I distal renal tubular acidosis, in which
157. A patient presents to a clinic with complaints of hydrogen ions diffuse back into the distal tubule. This not
headache fatigue. Lab data show serum sodium, 122 only retains hydrogen ions, but also reduces the distal
mEq/L; serum osmolality, 240 mOsm/L; urine tubule reabsorption of bicarbonate. The other choices
osmolality, 455 mOsm/L. which condition best listed are associated with acute tubular necrosis and
correlates with these data? decrease the GFR.
1. Neurogenic diabetes insipidus
2. Nephrogenic diabetes insipidus 160. All of the following structures are normally
3. Diabetes mellitus palpable upon vaginal examination except the:
4. SIADH 1. Uterine tubes
2. Ovary
Ans. 4 3. Rectouterine pouch
Hyponatremia, hypo-osmolality, and inappropriately 4. Sacral promontory
concentrated urine are characteristic of SIADH. The
syndrome results from excessive secretion of vasopressin. Ans. 4
This could be due to ectopic tumor secretion of Normally, the sacral promontory is too distant to be
vasopressin, various central nervous system palpable on vaginal examination. If the sacral promontory
complications, or it could be drug induced. The increased is palpable, this indicates that the conjugate diameter of
vasopressin causes excessive water retention, resulting in the pelvis is smaller than normal, and a normal vaginal
dilutional hyponatremia and hypo-osmolality. Volume delivery may not be possible. The ischial spines are
expansion inhibits the renin- angiotensin system. Plasma palpable. This landmark is used for administering a
aldosterone therefore tends to be low, which partly pudendal nerve block. The pudendal nerve wraps around
explains the inappropriately concentrated urine. the ischial spine and sacrospinous ligament. The
rectouterine pouch is palpable through the wall of the
158. Which of the following clinical findings is more posterior fornix of the vagina. The ovary and uterine tubes
likely to be associated with acute rather than chronic are palpable in the broad ligament.
glomerulonephritis?
1. Osteomalacia 161. Which would lead to an increase In hepatocyte
2. Increased anion gap metabolic acidosis synthesis of primary bile acids?
3. Oliguria 1. An injection of cholecystokinin
4. Preservation of concentrating ability 2. An ileal resection
3. Stimulation of the vagus nerve
Ans. 4 4. Lovastatin
Acute glomerulonephritis is more likely to be associated
with preservation of concentrating ability than chronic Ans. 2
glomerulonephritis, in which tubular dysfunction is The rate of synthesis of primary bile acids from cholesterol
present. Osteomalacia, the inability to mineralize osteoid, is inversely related to the rate at which bile acids are
is more likely to occur in chronic glomerulonephritis extracted from the hepatic portal blood and resecreted
because of the loss of vitamin D synthesis in the tubules. into the bile canaliculi. Therefore, an increase in the rate of
Increased anion gap metabolic acidosis is more likely to synthesis occurs following ileal resection. Ileal resection
occur in chronic glomerulonephritis, because tubular decreases the ability to absorb bile acids from the small
function and the ability to excrete a hydrogen load are intestine and causes a decrease in enterohepatic
disturbed. Oliguria occurs in both acute and chronic recirculation of the bile acids. Therefore, secretion of new
glomerulonephritis. Casts in the urine are present in both bile acids is increased. Injection of cholecystokinin or
diseases. However, red blood cell and white blood cell stimulation of the vagus nerve contract the gallbladder and
casts are more likely to be present in acute relax the sphincter of oddi, causing increased delivery of
glomerulonephritis, whereas waxy and I broad casts are bile acids to the duodenum. These bile acids are absorbed
more likely in chronic glomerulonephritis. in the ileum and returned to the liver in the hepatic portal
blood. Therefore, synthesis of new bile acids is decreased.
159. Which of the following renal alterations would Lovastatin, which decreases the activity of 3 hydroxy3-
most likely preserve the GFR? methylglutaryle coenzyme A (HMGCoA) reductase,
1. Defect in the elimination of hydrogen ions in the distal decreases synthesis of cholesterol and, therefore,
tubule decreases the synthesis of bile acids. A decrease in dietary
2. Luminal obstruction by renal tubular casts

33
cholesterol would decrease the synthesis of primary bile 3. Retropubic space
acids. 4. Deep perineal space

162. All of the following statements about the control Ans. 4


of micturition are true except: The external urethral sphincter is the voluntary sphincter
1. An individual with a spinal cord injury at L1 can still composed of skeletal muscle, the sphincter urethrae
have a micturition reflex muscle. This muscle, along with the deep transverse
2. The micturition reflex can occur without voiding any perineal muscle, are in the deep perineal space and form
urine the muscular layer of the urogenital diaphragm. The
3. Bladder volume can be more than double that present internal urethral sphincter is composed of smooth muscle
when the first urge to void occurs in the neck of the bladder. The internal and external
4. An individual with destruction of the sacral dorsal sphincters are both open during micturition. During
roots can still have a micturition reflex ejaculation, the external sphincter is open and the internal
sphincter is closed.
Ans. 4
Micturition is initiated by activation of afferent sensory 164. All of the following statements concerning ANP
fibers located in the wall of the bladder; these fibers sense are true except:
the degree of stretch of the bladder wall. In addition, these 1. Its secretion is increased by an increase in blood
sensory fibers travel back to the spinal cord via sacral volume
dorsal roots. The sensory information that reaches the 2. It increases glomerular filtration rate
spinal cord also travels to the micturition center in the 3. It inhibits rennin secretion
rostral pons. When sensory activity to the micturition 4. Its effects are mediated by increased cyclic AMP
center is sufficient, a command is sent to the sacral spinal
cord, leading to activation of parasympathetic fibers. The Ans. 4
parasympathetic fibers travel to the bladder via the pelvic ANP is a polypeptide composed of 28 amino acids that is
nerve. Activation of these fibers leads to bladder secreted by endocrine cells in the atria. ANP acts at the
contraction. Destruction of the sacral dorsal roots (as kidney to increase sodium excretion, and is secreted in
occurs with tabes dorsalis) abolishes the reflex because response to blood volume expansion or increased sodium
sensory afferent fibers no longer send signals back to the chloride intake. Circulating levels of ANP are increased in
spinal cord. congestive heart failure, a condition in which atrial
A spinal cord injury at L1 is well above the sacral region pressure is chronically elevated. The exact mechanisms for
where the micturition reflex originates. How ever, the the natriuretic effect are not completely known. ANP
central nervous system (CNS) plays an important role in increases glomerular filtration rate without a sustained
facilitating or inhibiting the micturition reflex, and this increase in renal blood flow, which suggests that the
function is lost with spinal cord injury. Although some peptide may constrict efferent arterioles. Furthermore,
patients with spinal cord injury can still elicit a micturition ANP relaxes mesangial cells in the glomeruli, thereby
response (e.g., stroking of the skin in the genital region), increasing surface area available for filtration. Finally, ANP
the bladder in these patients has increased muscle tone may directly inhibit sodium reabsorption by renal tubules.
and fails to empty completely. As the bladder becomes The actions of ANP are generally opposite to those of the
more and more distended, an involuntary micturition renin-angiotensin-aldosterone system. ANP inhibits the
reflex can occur. However, the CNS can keep urine from secretion of renin and aldosterone. ANP produces its
being voided under these circumstances by maintaining a effects by increasing intracellular formation of cGMP. The
constant tonic contraction of the external sphincter. This ANP receptor has a cytoplasmic domain with guanylyl
contraction is accomplished through continued activation cyclase activity, which is increased by hormone-receptor
of so matic nerves that travel in the pudendal nerve from binding.
the sacral spinal cord to the external sphincter. The point
at which an urge to void first occurs corresponds to a 165. A 20-year-old sexually active woman presents
bladder volume of approximately 150 ml. However, with swelling of the vaginal labia. Examination of the
bladder volume can increase more than twofold before perineal sharply defined subcutaneous bleeding
involuntary micturition occurs. At 400 ml a marked sense nodules that appear to erode through the skin. The
of fullness is present. Parasympathetic fibers originating in most likely etiologic agent in this case is
the sacral spinal cord innervate the body of the bladder, 1. Treponema pallidum
and it is activation of these fibers that leads to bladder 2. Herpes virus
contraction. 3. Calymmatobacterium
4. Neisseria gonorrhoeae
163. The external urethral sphincter is located in
which of the following regions? Ans. 4.
1. Ischiorectal fossa This patient is suffering from donovanosis, a chronic
2. Extrapertoneal space progressively destructive bacterial infection of the genital

34
region that was formerly known as granuloma inguinale or TB associated with HIV infection is probably the greatest
granuloma venereum. This disease is caused by gram- health risk to the general public and the health care
negative pleomorphic encapsulated intracellular profession. Though atypical mycobacterial infection occurs
bacterium called C. granulomatis. Although there may be relatively late in HIV infection in patients with low CD4
some patients who develop this disease on the basis of counts, active typical TB may develop relatively early in
nonintimate contact, it is most likely that this is a sexually the course of the disease. Patients with relatively high
transmitted disease. The incubation is usually 1 to 4 weeks CD4+ T cell counts may exhibit typical pulmonary
but may be longer. Most patients have disease limited to reactivation with the Clinical syndrome of weight loss,
the genital area; extragenital lesions occur in about 6% fever, cough, and dyspnea and with a chest x-ray revealing
and may include oral lesions and bleeding and/or bone apical cavitary disease. In patients with lower CD4 + T cell
disease. Diagnosis involves demonstration of intracellular counts, disseminated disease, including extra pulmonary
Donovan bodies within the mononuclear cells obtained manifestations, is more common. Fortunately, the
from a lesion. Syphilitic plaques are usually white rather treatment of active TB in an HIV patient can be quite
than the red lesions characteristic of donovanosis. successful, and the recommended therapy is generally the
Donovanosis lesions can be so necrotic that squamous cell same as it would be in a patient who does not have HIV
carcinoma m9Y be considered. infection. However, rifabutin should be substituted for
rifampin in patients who are receiving the protease
166. Impaired immune competence is the inhibitors or nonnucleoside reverse transcriptase
predisposing factor in about half of all persons who inhibitors because of adverse interactions.
develop
1. Histoplasmosis 168. Which statement concerning the pathogenesis of
2. Coccidoidomycosis fever is correct?
3. Blastomycosis 1. Aspirin inhibits the production of endogenous.
4. Cryptococcosis pyrogens
2. The major endogenous pyrogens in human in clued IL-
Ans. 4. 4 and transforming growth factor-B (TGF-B)
Fungal and yeast infections, predominantly candidiasis, 3. Endogenous pyrogens are produced by bacteria,
aspergillosis, and mucormycosis, occur frequently in protozoa, and fungi
severely immunosuppressed patients, particularly those 4. Endogenous pyrogens play a role in the cachexia of
who have received broad-spectrum antibiotics for a chronic infections.
prolonged period. A number of other types of fungal
infection occur in these patients. About 75% of all cases of Ans. 4.
Cryptococcus neoformans infection occur in persons who A host of stimuli, including infection with virtually any
have AIDS or lymphoma, are taking glucocorticoids, or are microorganism, cause macrophages, lymphocytes,
otherwise immunocompromised. The association of fibroblasts, and other cells to elaborate the key
cryptococcal meningitis and Hodgkin's disease is mediators of fever production, such as tumor necrosis
important clinically. factor (TNF) a, TNF- (lymphotoxin), IPN-, and the
interleukins, which are 17 -kDa glycoproteins that
167. Which statement concerning the relationship of promote the synthesis of E series prostaglandins in the
T6 and HIV infection is correct? hypothalamus and thus reset the central thermostat at a
1. Levels of plasma HIV RNA decrease in the setting of higher level. Aspirin and nonsteroidal anti-
active TB inflammatory agents act by inhibiting cyclooxygenase
2. TB usually develops only when the median CD4 count activity so that prostaglandin - (PG-) cannot be
is <200uL. synthesized; they do not act by reducing TNF and
3. Extrapulmonary disease is more common than IL1production. Glucocorticoids suppress fever by both
pulmonary disease in the HIV infected individual interfering with arachidonic acid metabolism and
4. Therapy for TB is the same in the HIV-infected patient downregulating the production of endogenous
as in the HIV-negative patient. pyrogens. TNF and IL-I also possess diverse effects,
including the induction of cachexia by TNF.
Ans. 4.
Tuberculosis has experienced a resurgence associated 169. A 20-year-old woman has mild pulmonary
with the HIV epidemic. In the United States, up to 5% of stenosis. Which of the following statements regarding
patients with AIDS has active TB infection which is more this situation is true?
commonly pulmonary than extra pulmonary. It is 1. Heart size on chest x-ray is likely to be enlarged
important to recognize and treat TB early in the course of 2. Electrocardiogram is likely to be abnormal
disease in patients with HIV infection, since TB can 3. There is loss of the jugular a wave
increase the level of HIV RNA in the plasma. Skin testing is 4. Compared to other valvular defects, the risk of
helpful for predicting who is at risk for TB but is not endocarditis is relatively low
helpful in the diagnosis of active infection. The epidemic of

35
Ans. 4 Digitalis glycosides are effective in increasing
Adults with mild pulmonic stenosis are generally myocardial contractility and in the treatment of certain
asymptomatic. Unlike congenital aortic stenosis, this atrial tachyarrhythmias. However, digoxin actually
condition usually does not progress; thus, follow-up increases myocardial automaticity (increase in premature
need not be frequent. The risk of endocarditis is beats) and facilitates reentry (atrial tachycardias). Digoxin
somewhat lower for pulmonic valves than for the other also slows conduction through A V nodal tissue and has
heart valves, whether normal or stenotic. Clinical signs central effects that can mimic vagal influence on the heart
of mild pulmonic stenosis include a prominent a wave and thus may produce sinus arrest. Paroxysmal atrial
on jugular venous pulse, a normal electrocardiogram, tachycardia with variable block represents the classic
and normal cardiac size on chest x-ray. rhythm of digitalis intoxication. Digoxin is profibrillatory,
but its administration should not lead to atrial flutter.
170. Each patient below is alert and oriented and has Therapeutic levels of digitalis generate characteristic ST-
a BP110/60. In which patient would adenosine segment and T-wave changes in most individuals taking
constitute appropriate initial therapy? the drug. These changes are known as the digitalis effect
1. A 65-year old man with no ischemic heart disease and and consist of ST -segment depression with flattening or
wide complex tachycardia inversion of the T wave. The digitalis effect is most
2. A 65 -year old woman with known ischemic disease prominent in leads with tall R waves. The digitalis effect is
and narrow complex tachycardia normal and predictable and does not require
3. A 25-year old woman with known preexcitation discontinuation of the- drug.
syndrome and narrow complex tachycardia
4. A 28-year-old man with known preexictation 172. The primary pathophysiologic problem in in
syndrome and wide complex tachycardia idiopathic pulmonary fibrosis is believed to be
1. Microorganism mediated activation of pulmonary
Ans. 2. Adenosine is currently approved for the neutrophils
termination of paroxysrriat supraventricular tachycardias 2. Immune complex mediated activation of alveolar
at doses of 6 mg and, if 6 mg fails, 12 mg. The primary macrophages
mechanism of adenosine is to decrease conduction velocity 3. Direct immune complex mediated pulmonary
through the A V node. As such, it is an ideal drug for acute interstitial damage
termination of regular reentrant supraventricular 4. Primary fibroblast
tachycardia involving the A V node) Side effects may
include chest discomfort and transier it hypotension. The Ans. 2.
half-life is extremely short, and the side effects tend to be Bronchoalveolar lavage in patients with idiopathic
brief. Patients with wide complex tachycardia suggestive pulmonary fibrosis, a chronic inflammatory disorder of the
of ventricular tachycardia or known preexcitation lower respiratory tract characterized by dyspnea and
syndrome should be treated with agents that decrease reticulonodular infiltrates on chest radiography, discloses
automaticity, such as quinidine and procainamide. an abundance of alveolar macrophages. Probably related
However, in patients with apparent ventricular to locally generated immune complexes, alveolar macro
tachycardia who have neither a history of ischemic heart phages become activated and then produce several
disease nor preexcitation syndrome, adenosine may be a mediators that recruit and induce fibroblast proliferation,
useful diagnostic agent to determine whether a patient has which causes secondary damage. Macrophage-derived
a reentrant tachycardia, in which case the drug may mediators believed to be important in this process include
terminate it; an atrial tachycardia, in which case the atrial fibronectin, a 200-kDa dimeric glycoprotein that interacts
activity may be unmasked; or a true, preexcited tachy- with connective tissue matrix as well as specific receptors
cardia, in which case adenosine will have no effect. While on fibroblasts, and platelet-derived growth factor, whose
adenosine is not the recommended primary therapy for chain is encoded by the c-sis protooncogene. Platelet-
patients with wide complex tachyarrhythmia, patients derived growth factor is believed to play an important role
with junctional tachycardia who have evidence of poor in recruiting fibroblasts to the site of inflammation.
ventricular function or concomitant -adrenergic blockade Macrophages also producechemotaxins such, as
may be reasonable candidates for its use. leukotriene 84 and interleukin 8, which attract neutrophils
and eosinophils into the region.
171. Which ECG findings represents a characteristic
manifestation of digitalis toxicity? 173. Which of the following would be the most
1. ST -segment depression reasonable step in the assessment of the patient of
2. T-wave inversion emphysema due to alpha 1 ATD.
3. Atrial flutter 1. Acid starch gel
4. Atrial tachycardia with variable block 2. Measurement of sweat chloride concentration
3. High resolution CT scan
Ans. 4 4. Exercise stress test

36
Ans. 1. infection, or, less commonly, a renal stone. Proteinuria
To establish baseline information in persons who have occurs in about 1/3 of patients but is typically mild, with <
emphysema, spirometry should be performed, and for 1 g per 24-h collection. The rare patient with nephrotic-
persons with significant complaints or physical findings, range proteinuria often has superimposed glomerular
arterial blood gases also should be checked. Although disease.
cigarette smoking accounts for the vast majority of cases of
emphysema, a small percentage of persons who develop 175. Which of the following statements regarding
this illness have had no exposure to tobacco products. A eosinophilic enteritis is correct?
subset of this nonsmoking, emphysematous population is 1. Peripheral blood eosinophilia is rare )
deficient in 1-antitrvpsin. which is a protease inhibitor 2. It affects only the small intestine
that normally is found in the serum. It is currently believed 3. The majority of patients have a history of food
that release of proteolytic enzymes from inflammatory allergies or asthma.
cells accounts for the lung destruction that typifies 4. It may difficult to distinguish from regional enteritis
emphysema, and (Xl-antitrypsin deficiency, a familial
disorder, the genotype of which is acid starch gel and Ans. 3.
immunoelectrophoresis, permits this destruction to occur Malabsorption caused by bacterial overgrowth results
unimpeded. Exercise testing is not necessary as an initial from bacterial utilization of ingested vitamins and the
screening test for emphysema but should be considered deconjugation of bile salts by bacteria in the proximal
before oxygen therapy is prescribed. A male who has jejunum. Deconjugated bile salts do not form micelles in
emphysematous respiratory failure, gives no history of the jejunum, and long-chain fatty acids cannot be
respiratory infections, and has children would not have absorbed. The bacteria also separate ingested vitamin 812
cystic fibrosis (affected men are sterile); therefore, a sweat from intrinsic factor, thus interfering with its absorption
chloride test would not be a useful procedure. High- from the ileum. The absorption of simple carbohydrates
resolution CT scan would unlikely add any significant generally is not impaired, though complex carbohydrates
alteration to the differential diagnosis, and both an may be metabolized by bacteria. Thus, persons with
exercise stress test as well as an echocardiogram are not bacterial overgrowth have steatorrhea, an abnormal
necessary in the evaluation of this patient. Schilling test (even with the administration of intrinsic
factor), increased metabolism of nonabsorbable
174. Which of the following statements about carbohydrates (e.g., lactulose), and increased bacterial
polycystic kidney disease is true concentrations in jejunal aspirates. Absorption of Dxylose,
1. Polycystic kidney disease is n autosomal recessive a simple carbohydrate, is often normal.
disorder linked to a causative gene on the short arm of
chromosome 16 176. As a consequence of severe liver damage, hepatic
2. Erythropoietin levels are often low due to progressive amino acid handling is deranged. In this situation,
renal failure. plasma levels of which of the following are likely to be
3. Colonic diverticular disorder is a rare finding lower than normal?
4. Nephrotic range proteinuria is an uncommon finding 1. Ammonia (NH3)
2. Ammonium (NH+4)
Ans. 4. 3. Alanine
Autosomal dominant polycystic kidney disease is one of 4. Urea
the most common hereditary disorders. Spontaneous
mutations occur but account for only 10% or fewer of all Ans. 4.
cases. The majority of cases are linked to a causative gene Amino acids (except for the branched-chain amino acids
located on the short arm of chromosome 16, referred to as leucine, isoleucine, and valine) are taken up by the liver via
ADPKD1. A second causative gene (ADPKD2) has not been the portal circulation and are metabolized to urea. Severe
identified but seems to be at a locus distinct from ADPKDI, liver damage disrupts normal amino acid metabolism and
which appears to be the cause of the gene in -90% of is reflected in elevated serum levels of non-branched-chain
families in the white population. amino acids. Since urea cannot be produced, ammonia
A rare syndrome known as autosomal recessive polycystic cannot be handled. Elevated levels of serum ammonia
kidney disease affects between 1 in 10,000 and 1 in 40,000 certainly playa large role in the development of hepatic
individuals in the United States, and the causative gene has encephalopathy in patients with liver failure and portal
been localized to chromosome. Patients are typically hypertension. Therefore, levels of ammonia and, in the
diagnosed during the first year of life, presenting with case of alkylosis, ammonium ion rise at the expense of
bilateral abdominal masses. Death in the neonatal period is urea. Other mechanisms leading to increased blood
common due to pulmonary hypoplasia. ammonia levels include excessive amounts of intestinal
The most common complication of polycystic kidney nitrogen (e.g., resulting from bleeding); decreased
disease is pain secondary to large cystic kidneys; in intestinal motility allowing greater bacterial deamination
addition, most patients are hypertensive. In addition to the of amino acids; depressed renal function leading to an
chronic pain, acute pain may represent cystic hemorrhage, increase in blood urea nitrogen and a greater opportunity

37
for bacterial urease to convert this to ammonia; alkalosis, conditions the reticuloendothelial system can be
which will preferentially lead the NHt /NH3 equilibrium in overwhelmed by immune complexes, thereby impeding
favor of ammonia; and portal hypertension, which will the removal and leading to the deposition of immune
allow ammonia from the gut to bypass hepatic complexes. Deposition of these complexes in tissues other
detoxification. than those of the reticuloendothelial system is responsible
for the signs and symptoms of immune-complex disease. In
177. Which statement regarding the renal animal models, the persistence of complexes is necessary
involvement associated with SLE is true? for the development of renal disease; also, slight antigen
1. Clinically apparent renal disease occurs in 90% of excess has been found to predispose to the formation of
affected persons antigen-antibody complexes, which persist in the
2. Interstitial nephritis is a rare finding on renal biopsy circulation and lead to inflammatory illness. Immune
3. Renal biopsy is not initially necessary in patients with complex-mediated vascular damage can lead to cutaneous
deteriorating renal function and active urine sediment necrotizing vasculitis. Electron microscopy reveals
4. Renal disease is uncommon in patients with higher subendothelial immune complexes that presumably incite
anti-double stranded DNA antibodies an array of inflammatory cells to migrate toward the
vessel.
Ans. 3.
Renal disease is clinically evident in about half of persons 179. Peripheral nerve regenerates at a rate of
with SLE. However, nearly all persons with SLE have some 1. 0.2 mm/day
evidence of renal disease on renal biopsy. Renal disease 2. 1 mm/day
associated with SLE includes both glomerulonephritis and 3. 2 mm/day
interstitial nephritis. Glomerular disease has been 4. 0.5 nun/day
classified into membranous nephritis and mesangial, focal,
and diffuse glomerulonephritis. Immune-complex Ans. 2.
interstitial nephritis occurs most commonly in persons The axonal regeneration proceeds slowly at rate of 1-2
who have diffuse glomerulonephritis. Urinalysis mm/day in human.
performed in persons with active renal disease usually
reveals microscopic hematuria, red cell casts, and 180. Km value is defined as -
proteinuria; the exception is membranous lupus nephritis, 1. Substrate concentration at Vmax/2
in which proteinuria is the dominant finding. Drug- 2. Substrate concentration of twice Vmax
induced lupus rarely leads to renal disease. Anti-dsDNA 3. Substrate concentration of twice Vmax
antibodies at high titer are associated with severe 4. Substrate concentration of one third Vmax
nephritis. Renal biopsy is not necessary in SLE patients
whose renal function is rapidly deteriorating when they Ans. is 1' i.e., Substrate concentration at Vmax/2
have an active sediment. If such patients fail to respond to Km is a special rate constant called Michaelis-Menten
the prompt initiation of glucocorticoid therapy demanded constant.
in such a situation, then biopsy should be undertaken. It is a defined as the substrate concentration at which
Patients with mild clinical disease should have a biopsy to the reaction rate is half the maximum6.
determine if they have active, severe, inflammatory At this instance the enzyme is half -saturated with its
lesions, which might respond to therapy. substrate,
Km reflects binding affinity of the enzyme for its
178. Which statement regarding immune complex substrate.
disease is correct? If affinity is more, less substrate is required to saturate
1. Even under normal condition only a small subsets of the enzyme, so that Vmax is reached at relatevely low
immune complexes are removed complexes in the substrate concentration.
reticuloendothelial system? Consequently, the substrate concentration
2. Signs and symptoms stem from the deposition of corresponding to the half V max (i.e., Km) is relatively
massive amount of immune complexes in the low.
reticuloendothelial system For this reason, high-enzyme substrate affinity implies
3. Persistence of immune complexes in the circulation a low Km value, and conversely, low affinity implies
seems to be a requirement for the development of high Km.
renal manifestations For example, higher Km of glucose for glucokinase
4. Renal lesions depend on antigen-antibody than that for hexokinase indicates that hexokinase
combinations in which antibody is in slight binds glucose with higher affinity than glucokinase.
excess
181. Primer function is in -
Ans. 3. 1. Transcription
Most antigen-antibody complexes are cleared by cells of 2. Translation
the reticuloendothelial system. It appears that in some 3. Initiation of DNA replication
38
4. Termination of DNA replication Chromosomes are arranged in pairs and decreasing
order of length.
Ans. is 3 i.e.. Initiation of DNA replication Staining
DNA synthesis cannot commence with Staining allows identification of each individual
deoxyribonuclcotides because DNA polymerase chromosome on the basis of distinctive and reliable
cannot add a mononucleotide to another pattern of alternating light and dark bands. One of the
mononucleotide. following banding techniques may be used.
Thus, DNA polymerase cannot initiate synthesis of
complementary DNA synthesis strand of DNA on a 183. During staining for Electron Microscopy, the
totally single stranded template. method which improves contrast of specimen is
For this, they require RNA primer, which is a short 1. Positive staining
piece of RNA formed by enzyme primase (RNA 2. Negative staining
polymerase6) using DNA as a template. 3. Shadow staining
RNA primer is then extended by addition of 4. None of these
deoxyribonucleotides. Later on, the ribonucleotides of
the primer are replaced by deoxyribonucleotides. Ans. 2. Negative staining.
The specimen stained with phosphotungstic acid in
182. Chromosomal study is best carried out in - electron microscope for better visualisation of the
1. Prophase structures. Negative staining with phosphotungstic acid
2. Metaphase used in electron microscope.
3. Telophase
4. Anaphase
184. The immunoglobulin mediates ADCC
Ans. is '2' i.e., Metaphase 1. IgM
Karyotyping 2. IgA
It is the study of chromosomes. 3. IgG
It is used in cytogenetics to study the chromosomal 4. IgE
abnormalities.
A karyotype is a standard arrangement of a Ans. 3. IgG
photographed or imaged stained metaphase spread in IgG Antibody
which chromosomepairs arc arranged in order of o Half life(23d),
decreasing length. o High Serum concerntration (GAMDE) ,
To produce a karyotype, one must obtain cells capable
of growth and division as karyotyping is done by Responsible for-
arrestingmitosis in dividing cells in metaphase. o Precipitation,
Samples used are o neutralization,
Amniotic fluids (cells from amnion, skin, o NK cell (ADCC)
urogenital system, respiratory system and GI o Classical complement
system). Appear late, indicates past / chronic infection
Chorionic vill.
Bone marrow 185. The immunity responsible for to control epidemic
Skin fibroblasts spread of a pathogen
Peripheral blood lymphocytes 1. Innate immunity
Fetal umblicai blood. 2. Active acquired immunity
Lymph node tissue 3. Herd immunity
Solid tumor sample 4. Passive natural immunity
Method of karyotyping
Dividing cells are arrested in metaphase by addition of Ans. 3. Herd immunity
colchicine or colcemid (deacetylmethylcolchicine). Herd immunity-
Subsequently, cells are exposed to a hypotonic Overall immunity of a community to a pathogen
solution to induce swelling of the cell for enhancing If Herd immunity is good- chance of epidemic is less
spreading of the chromosomes. Eradication of a communicable disease- depends on
The metaphase cells are then fixed with good Herd immunity
methanol/glacial acetic acid mixture and stained by Provided by mass vaccination by live vaccination to all
one of the several banding techniques. individual at same time.
After staining chromosomes are analysed under a
microscope and photographed. 186. Streptolysin O is inactivated by
Finally, a karyotype is constructed by manual or 1. CO2
automated pattern, 2. Nitrogen
39
3. Oxygen 190. Bacillary angiomatosis is caused by
4. Serum 1. Bartonella henselae
2. Hansens bacillus
Ans. 3. Oxygen 3. Kochs bacillus
Streptococci produce 2 types of Streptolysin ( O&S) based 4. H. ducreyi
on oxygen dependency. Streptolysin O is oxygen labile. It is
inactive in the presence of Oxygen. Streptolysin O is Ans. 1. Bartonella henselae
antigenic and ASO titre is a standard serological diagnosis Henselae causes bacillary angiomatosis, a condition
of Acute Rheumatic fever. Streptolysin O & S both can resembling Kaposis sarcoma in patients with AIDS. The
cause hemolysis. skin, lymph nodes or liver and spleen are primarily
involved in this condition. The bacteria also cause sub
187. Weil-Felix reaction with OXK antigen indicates acute bacterial endocarditis.
infection with
1. R. tsustsugamushi 191. The RNA virus which is inhibited by Actinomycin
2. R. mooseri D
3. R. quintana 1. Measles
4. R. akari 2. Polio
3. Influenza
Ans. 1. R. tsustsugamushi 4. Rubella
Weil-Felix reaction:
Serological diagnosis of Rickessial infection. Weil-Felix Ans. 3. Influenza
reaction is a Heterophile agglutination test. Antigens used INFLUENZA VIRUS
are Nonmotile strains of Proteus vulgaris (OX 19, OX 2), Orthomyxovirus, ss negative sense RNA
Proteus mirabilis (OX K). Possesses RNA dependent RNA polymerase
Mechanism: Proteus and Rickessiae will share alkali stable Haemagglutination is the characteristic of influenza virus
carbohydrate antigen will be basis of the Weil-Felix After some time haemagglutination is reversed by
reaction. Weil-Felix is a tube agglutination test. It is a neuraminidase(receptor destroying enzyme also present
simple and useful test for the diagnosis. The Antibody rises V.cholerae) by a process called elution
in 2nd week of illness and reaches the peak and get decline Antigenic variation is common ( Antigenic shift, Antigenic
rapidly during the course of convalescence. drift)
Actinomycin D inhibits multiplication
188. Nichol's strain is used in:
1. TP HA test 192. Relative risk is calculated in?
2. TPI test 1. Cross sectional study
3. VDRL 2. Cohort study
4. FTA-ABS 3. Case cuntiol study
4. None
Ans. 2. TPI test
Treponema pallidum immobilization (TPI) test, considered Ans. is '2' i.e.. Cohort study
both as a qualitative and as a quantitative test, and then Characteristic Cohort Case control Cross
various specific aspects of passage of the Nichols strain of s Study Study Sectional
treponeme in rabbit testes. a serological test for syphilis in Study
which a solution containing the living causative spirochete Design Prospective Retrospectiv One point
(Treponema pallidum) is combined with serum in the e in time
presence of complement with immobilization of the active Incidence Estimated Not Not
spirochetes indicating a positive result estimated estimated
Prevalence Not Not Estimate
189. Inclusion bodies of trachoma are stained with? estimated estimated d
1. Wrights stain Calculations o Incidence Odds ratio
2. Iodine rates
3. Saffranin o Relative
4. Alcian blue risk
o
Ans. 2. Iodine Attributabl
Stained with dilute Lugols iodine solution, some of the e risk
inclusions of C trachomatis appear brown because of the
glycogen matrix that surrounds the particles. Inclusions of
C psittaci are diffuse intracytoplasmic aggregates. 193. ICD chapter 21 includes all except
1. Alcohol

40
2. Tobacco 4. Per capita income
3. Injury
4. Drugs Ans. is 1' i.e., IMR
Subjective component of well being Quality of life
Ans. is 3' i.e., Injury While "level of living" is an objective component,
ICD -10 code in chaptor XXI (Factor influencing health "quality of life" comprises the individual's own
status and contact with health services) : - subjective evaluation.
i) Problem relating to lifestyle The index for quality of life is "Physical quality of life
ii) Alcohol use index (PQLI) 99.
iii) Tobacoo use The PQLI is an attemt to measure quality of life or
iv) Drug use well-being of a country,
v) Lack of physical exercise Physical quality of life index consolidates three
vi) Inappropriate diet and eating habits indicators :-
vii) High risk sexual behaviour i) Literacy rate
ii) Infant mortality rate
194. Vitamin D is maximum in iii) Life expectancy at age 1 year (LE)
1. Milk PQLI ranges from 0 to 100
2. Fish fat PQLI in India is 65.
3. Eggs
4. Cod liver oil 197. Cataract in diabetes
1. Snowflake
Ans. Is '4' i.e., Cod liver oil 2. Sunflower
Vitamin D is found only in food of animal origin (also 3. Nuclear cataract
vit B12). 4. Oil drop
Dietary sources of vitamin D in decreasing order -
1. Halibut liver oil 5. Egg Ans. is '1' i.e., Snowflake
2 Cod liver oil 6. Butter Diabetic cataract
3. Shark liver oil 7. Milk Cataract is considered as a major cause of visual
4. Fish fat impairment in diabetic patients. Senile cataract tends
to develop atan earlier age and more rapidly than
usual in diabetic patients.
195. Iron is maximum in - The enzyme aldolase reductase catalyzes the
1. Pista reduction of glucose to sorbitol through the polyol
2. Cashewnut pathway, aprocess linked to the development of
3. Meat diabetic cataract NADPH* dependent aldolase
4. Milk reductase reducesexcessive glucose to sorbitol and
intracellular accumulation of sorbitol leads to osmotic
Ans. is 3. i.e., Meat changes resulting inhydropic lens fibers that
"Red meat is the richest source of iron "- Artcle degenerate and form sugar cataract.
"Iron content of milk is low in all mammalion speies - Park The typical diabetic cataract is snowflake cataract, i.e.,
Source of iron dense white subcapsular opacities in the anterior and
There are two major forms of iron. posterior cortex resembling a snowstorm.
1. Haem-iron As blood- sugar levels fluctuate, visual acuity and
Better absorbed refractive error can change Fluctating refractive
Less important source of iron in Indian diet. error. This causes frequent change in optimal eyeglass
Sources are Liver, meat9poulty and fish (foods prescription.
of animal origin).
2. Non-haem iron
Poorly absorbed but is the important source of 198. Amaurosisfugax is due to -
iron in the diet of a large majority of Indian 1. TIA
people. 2. Tobacco
Sources are Vegetable origin, Le, cereals, green 3. Optic neuritis
leafy vegetables, Leugumes, nuts, oilseeds, jaggery 4. Papilloedema
and dried fruits.
Ans. is 1 i.e., TIA
196. PQLI includes - Amaurosis fugax is sudden temporary and painless
1. IMR loss of vison due to transient failure of retinal
2. Life expectancy at birth circulation.
3. MMR It may occur in carotid transient ischemic attack (TIA).
41
Amaurosis 4. Keratitis associated with diseases of skin and mucous
Amaurosis is complete loss of sight in one or both membrane.
eyes, in the absence of ophthalmoscopic or other 5. Keratitis associated with systemic collagen vascular
marked objective signs. disorders.
Amaurosis fugax 6. Traumatic keratitis, which may be due to mechanical
It refers to sudden, temporary and painless monocular trauma, chemical trauma, thermal burns, radiations.
visual loss occurring due to a transient failure of 7. Idiopathic keratitis e.g.,
retinal circulation. Common causes are :- a) Mooren's corneal ulcer
Carotid transient ischaemic attacks (TIA) b) Superior limbic keratoconjunctivitis
Embolization of retinal circulation c) Superficial punctate keratitis of Thygeson
Papilledema
Giant cell arteritis 200. Blind areas of the larynx when visualized through
Raynaud's disease Video Laryngoscopy are:
Migraine 1. Ventricle
As a prodromal symptom of central retinal artery 2. Subglottis
or carotid artery occlusion 3. Post Cricoid region
Hypertensive retinopathy 4. Lingual surface of epiglottis
Venous stasis retinopathy
Typically described by the patients as a curtain that Ans. 1. Ventricle
descends from above or ascends from below to occupy
the upper or lower halves of their visual fields. The
attack lasts for two to five minutes and resolves in the
reverse pattern of progression, leaving no residual
deficit.
Uraemic amaurosis
It is sudden, bilateral, complete loss of sight occuring
probably due to the effect of certain toxic materials
upon the cells of the visual centre in patients suffering
from acute nephritis, eclampsia of pregnancy and
renal failure.It is associated with dilated pupils which
generally react to light. Usually, the vision recovers in
12-48 hours.

201. Lymphatic Drainage of the Maxillary Sinus goes


199. Keratitis is caused by first to
1. Bacteria 1. Upper Deep Cervical Lymph Nodes
2. Atopy 2. Submental Nodes
3. Protozoa 3. Submandibular Nodes
4. All 4. Retropharyngeal Nodes

Ans. is '4' i.e., All Ans. 4. Retropharyngeal nodes


Causes of keratitis Through the posterior pathway, the lymphatics from the
1. Infective keratitis maxillary and ethmoid sinuses drain into the
a) Bacterial retropharyngeal lymph nodes
b) Viral
c) Fungal 202. Blood supply of the tonsil is all except:
d) Chlamydial 1. Ascending pharyngeal Artery
e) Protozoal 2. Sphenopalatine artery
f) Spirochactal 3. Facial artery
2. Allergic keratitis 4. Descending palatine artery
a) Phlyctenular keratitis
b) Vernal keratitis Ans. 2. Sphenopalatine artery
c) Atopic keratitis
3. Trophic keratitis
a) Exposure keratitis
b) Neuroparalytic keratitis
c) Keratomalacia
d) Atheromatous ulcer

42
2. First pass is to examine middle meatus
3. Second pass is to examine the superior meatus
4. Third pass is the examine the inferior meatus

Ans. 3. Second pass is to examine superior meatus


All other options are wrong the right answers are:
There are three passes
First is along inferior meatus
Second is to inspect superior meatus and
sphenoethmoid recess
Third is to inspect the middle meatus

205. Most common major salivary gland malignancy is:


1. Mucoepidermoid carcinoma
2. Pleomorphic adenoma
3. Adenoid cystic carcinoma
4. Adenocarcinoma

203. All are mechanisms of bone conduction except: Ans. 1. Mucoepidermoid carcinoma, Pg 2497 Scott
1. Vibrations of air column of the EAC Brown Otorhinolaryngology Vol2
2. Vibrational inertia of ossicles
3. Vibration of the inner ear 206. Plunging ranula refers to a mucocoele that has
4. Air conduction through the Tympanic Membrane penetrated which muscle?
1. Sternohyoid
Ans. 4. Air conduction through the tympanic 2. Stylohyoid
membrane 3. Omohyoid
This is a method of air conduction, the other 3 are the 4. Mylohyoid
mechanisms of bone conduction
Ans. 4. Mylohyoid

207. All are contents of the middle ear except:


1. Stapedius
2. Tensor tympani
3. Facial nerve
4. Chorda tympani

Ans. 3. Facial Nerve

All the other structures with the tympanic plexus is


described as contents of the middle ear cavity
The facial nerve is described as a part of the medial
and posterior wall of the tympanic cavity

204. True about Nasal Endoscopy is:


1. There are 4 passes

43
Garre's sclerosing osteomyelitis Chronic
osteomyelitisor subacute form of chronic osteomyelitis

"Brodie's abscess is a chronic localized hone abscess"


Postgraduate orthopaedics
"Brodie s disease is a localized form of chronic
osteomyelitis " Ebnezar

Brodie's abscess
Brodie's abscess is a special form of chronic
osteomyelitis which arises insidously, without a
preceding acute attack.
There is localized abscess within the bone, often near
the site of the metaphysis of long hones.
Common sites are upper end of the tibia and lower end
of the femur.
A deep 'boring* pain is the predominant symptom.
Pain may become worse at night. Radiographically, the
lesion is seen as a circular or oval cavity surrounded
by a zone of sclerosis,
Treatment is by operation.
Surgical evacuation and curretage is performed with
or without bone grafting.

There are three types of chronic


osteomyelitis:
a) Chronic osteomyelitis secondary to acute
208. True about Ackermans tumor Is osteomyelitis.
1. It is a highly malignant tumor
b) Garres osteomyelitis.
2. Verrucous carcinoma of the larynx
3. Radiotherapy is the treatment of choice
c) Brodies abscess.
4. Spreads via lymphatics in early stage itself
211. Malignancies associated with BRCA 1 and 2
Ans. 2. Verrucous carcinoma of the larynx includes all except
It is a low grade locally aggressive malignancy. Well 1. Breast cancer
differentiated squaumous cell carcinoma with very 2. Prostate cancer
low metastasis rate. 3. Papillary serous carcinoma of peritoneum
Radiotherapy is contraindicated as it can produce 4. Mesothelioma
anaplastic change.
Ans. 4
209. Most common cause of Chronic Retropharyngeal
abscess in adults in India is:
1. HIV 212. The investigative method of choice for
confirmation of H.pylori eradication is
2. Impacted foreign Body
3. Complication of tonsillitis 1. Urea breath test
4. Tuberculosis 2. Rapid urease assay
3. Culture from gastric mucosa
Ans. 4. Tuberculosis Medscape 4. Serology
All other causes produce acute symptoms and in India
Tuberculosis is still a very orevalent disease. Ans. 1
In summary, it is not necessary to perform endoscopy to
210. Brodie's abscess is - diagnose H. pylori. Serology is the test of choice for initial
1. Acute osteomyelitis diagnosis when endoscopy is not required. If, however,
endoscopy is to be performed, the rapid urease assay and
2. Subacute osteomyelitis
3. Chronic osteomyelitis histology are both excellent options, but the cost
4. Septic arthritis advantage lies with the rapid urease assay. After
treatment, the urea breath test is the method of choice but
Ans. is 3 i.e., Chronic osteomyelitis again should not be performed until 4 weeks after therapy
ends. If the breath test is unavailable, endoscopy may be
Brodie's abscess Chronic osteomyelitis
44
performed in selected patients such as those with bleeding 215. The definite diagnosis of malignancy in
ulcers or other complications of their PUD. pheochromocytoma is based on
1. Presence of metastasis
213. Splenic auto infarction is seen in 2. Cellular pleomorphism
1. Splenic amyloidosis 3. Nuclear pleomorphism
2. Thalassemia 4. Mitotic figures
3. Sickle cell disease
4. Gauchers disease Ans: 1
presence of metastasis
Ans: 3 Pheochromocytoma are neoplasms composed of
Sickle cell disease chromaffin cells, which synthesize and release
As the largest unit of the mononuclear phagocyte system, catecholamines and in some instances peptide hormones.
the spleen is involved in all systemic inflammations, It is important to recognize these tumors because they are
generalized hematopoietic disorders, and many metabolic a rare cause of surgically correctable hypertension.
disturbances. In each, the spleen undergoes enlargement Pheochromocytoma range from small, circumscribed
(splenomegaly), which is the major manifestation of lesions confined to the adrenal to large hemorrhagic
disorders of this organ. It is rarely the primary site of masses weighing kilograms. The tumors are composed of
disease. Splenic insufficiency due to splenectomy or polygonal to spindle-shaped chromaffin cells or chief cells,
autoinfarction (as in sickle-cell disease) has one major clustered with the sustentacular cells into small nests or
clinical manifestation, an increased susceptibility to sepsis alveoli (zellballen) by a rich vascular network. The
cause by encapsulated bacteria such as pneumococcus, cytoplasm has a finely granular appearance, best
meningococcus, and Haemophilus influenzae. The loss of demonstrated with silver stains, due to the presence of
filtering and antibody production functions both granules containing catecholamines. The nuclei are usually
contribute to the increased risk of sepsis, which may be round to ovoid, with a stippled salt and pepper
fatal. All asplenic individuals should be vaccinated against chromatin that is characteristic of neuroendocrine tumors.
these agents to reduce the risk of this tragic complication. There is no histologic feature that reliably predicts
clinical behavior. Several histologic features, such as
214. All are false of Brutons Agammaglobulinemic numbers of mitoses, confluent tumor necrosis, and spindle
except cell morphology, have been associated with an aggressive
1. It is X linked behavior and increased risk of metastasis, but these are
2. T cell abnormality seen not entirely reliable. Tumors with benign histologic
3. Seen only in males features may metastasize, while bizarrely pleomorphic
4. Increase in number of plasma cells tumors may remain confined to the adrenal gland. In fact,
cellular and nuclear pleomorphism, including the presence
Ans: 1 of giant cells, and mitotic figures are often seen in benign
It is X linked pheochromocytomas, while cellular monotony is
X-linked agammaglobulinemia is one of the more common paradoxically associated with an aggressive behavior. Even
forms of primary immunodeficiency. It is characterized by capsular and vascular invasion may be encountered in
the failure of B-cell precursors (pro-B cells and pre-B cells) benign lesions. Therefore, the definitive diagnosis of
to develop into mature B cells. As an X-linked disease, this malignancy in pheochromocytomas is based exclusively on
disorder is seen almost entirely in males, but sporadic the presence of metastases. These may involve regional
cases have been described in females, possibly caused by lymph nodes as well as more distant sites, including liver,
mutations in some other gene that functions in the same lung, and bone.
pathway. The disease usually does not become apparent
until about 6 months of age, as maternal immunoglobulins 216. All are true about primary CNS lymphoma except
are depleted. 1. Most primary brain lymphomas are of B-cell origin.
The classic form of this disease has the following 2. Anaplastic large cell lymphomas are the most common
characteristics: histologic group.
B cells are absent or markedly decreased in the 3. Hooping is characteristic of primary brain lymphoma.
circulation, and the serum levels of all classes of 4. Primary lymphomas of the CNS are aggressive
immunoglobulins are depressed. Pre-B cells, which
express the B-lineage marker CD19 but not membrane Ig, Ans: 2
are found in normal numbers in the bone marrow. Anaplastic large cell lymphomas are the most common
Germinal centers of lymph nodes, Peyer's patches, the histologic group
appendix, and tonsils are underdeveloped.
Plasma cells are absent throughout the body. Primary CNS lymphoma accounts for 2% of extra-nodal
T cellmediated reactions are normal. lymphomas and 1% of intra-cranial tumors. It is the most
common CNS neoplasm in immunosuppressed individuals,
including those with AIDS and immunosuppression after

45
transplantation. Most primary brain lymphomas are of B- Hyperplastic arteriolitis
cell origin. In the setting of immunosuppression, the cells Benign nephrosclerosis is the term used for the renal
in nearly all such tumors are latently infected by Epstein- pathology associated with sclerosis of renal arterioles and
Barr virus. Overall, primary lymphomas of the CNS are small arteries. The resultant effect is focal ischemia of
aggressive, with relatively poor response to chemotherapy parenchyma supplied by vessels with thickened walls and
compared with peripheral lymphomas. Diffuse large-cell B- consequent narrowed lumens. The parenchymal effects
cell lymphomas are the most common histologic group. include glomerulosclerosis and chronic tubulointersititial
Within the tumor malignant cells infiltrate the parenchyma injury, producing a reduction in functional renal mass. The
of the brain and accumulate around blood vessels. kidneys are either normal or moderately reduced in size.
Reticulin stains demonstrate that the infiltrating cells are The cortical surfaces have a fine, even granularity that
separated from one another by silver-staining material; resembles grain leather. The loss of mass is due mainly to
this pattern, referred to as hooping, is characteristic of cortical scarring and shrinking. On histologic examination
primary brain lymphoma. there is narrowing of the lumens of arterioles and small
arteries, caused by thickening and hyalinization of the
217. Durck granuloma is seen in walls (hyaline arteriolosclerosis). Corresponding to the
1. Tuberculosis fine surface granulations are microscopic subcapsular
2. Leprosy scars with sclerotic glomeruli and tubular dropout,
3. Actinomycosis alternating with better preserved parenchyma. In addition,
4. Malaria the interlobular and arcuate arteries show a characteristic
lesion that consists of medial hypertrophy, reduplication of
Ans: 4. Malaria the elastic lamina, and increased myofibroblastic tissue in
In malignant cerebral malaria caused by P. falciparum, the intima, which combine to narrow the lumen. This
brain vessels are plugged with parasitized red cells. change, called fibroelastic hyperplasia, often accompanies
Around the vessels there are ring hemorrhages that are hyaline arteriolosclerosis and increases in severity with
probably related to local hypoxia incident to the vascular age and in the presence of hypertension.
stasis and small focal inflammatory reactions (called
malarial or Durck granulomas). With more severe hypoxia, 220. The following are true about acanthosis nigrans
there is degeneration of neurons, focal ischemic softening, except
and occasionally scant inflammatory infiltrates in the 1. It is an important cutaneous marker of benign
meninges. conditions
2. It is autosomal recessive
218. All are true of Verner Morrison syndrome (WDHA 3. It is an important cutaneous marker of malignant
syndrome) except conditions
1. It is associated with Vipoma 4. Most commonly appears in the flexural areas of the
2. Presents with secretory diarrhea body
3. Can be associated with ganglioneuroblastoma
4. Associated with hyperkalemia Ans: 2
It is autosomal recessive
Ans: 4
Associated with hyperkalemia Acanthosis nigricans is a condition marked by thickened,
VIPoma (watery diarrhea, hypokalemia, achlorhydria, or hyperpigmented skin with a velvet-like texture that most
WDHA syndrome) is an endocrine tumor of pancreas that commonly appears in the flexural areas (axillae, skin folds
induces a characteristic syndrome, caused by release of of the neck, groin, and anogenital regions). It can be an
vasoactive intestinal peptide (VIP) from the tumor. Some important cutaneous marker of benign and malignant
of these tumors are locally invasive and metastatic. A VIP conditions and, accordingly, is divided into two types. The
assay should be performed on all patients with severe benign type, which constitutes about 80% of all cases,
secretory diarrhea. Neural crest tumors, such as neuro develops gradually and usually occurs in childhood or
blastomas, ganglioneuroblastoma, and ganglioneuromas during puberty. It may occur (1) as an autosomal dominant
and pheochromocytoma can also be associated with the trait with variable penetrance, (2) in association with
Vipoma syndrome. obesity or endocrine abnormalities (particularly with
pituitary or pineal tumors and diabetes), and (3) as part of
219. All are true of Benign nephrosclerosis except several rare congenital syndromes. As with seborrheic
1. Kidneys are small with fine granularity keratoses, acanthosis nigricans sometimes occurs as a
2. Histological examination revealed hyaline paraneoplastic process resulting from the production of
arteriolosclerosis growth factors by a variety of tumors. The malignant type
3. Hyperplastic arteriolitis refers to lesions arising in middle-aged and older
4. Fibroelastic hyperplasia is seen individuals in association with underlying cancers, most
commonly gastrointestinal adenocarcinomas. The
Ans: 3 epidermis and underlying enlarged dermal papillae

46
undulate sharply to form numerous repeating peaks and variants. The tumor occurs in children younger than 10
valleys. Variable hyperplasia may be seen, along with years of age and typically arises in the nasal cavity, orbit,
hyperkeratosis and slight basal cell layer middle ear, prostate, and paratesticular region. This
hyperpigmentation (but no melanocytic hyperplasia). The variant of rhabdomyosarcoma commonly has parental
familial form is associated with germline activating isodisomy of chromosome 11p15.5, which leads to
mutations in FGFR3; depending on the mutation, overexpression of the imprinted IGFII gene. The sarcoma
acanthosis may be an isolated finding or be seen together botryoides subtype develops in the walls of hollow,
with skeletal deformities, including achondroplasia and mucosal-lined structures, such as the nasopharynx,
thanatophoric dysplasia. common bile duct, bladder, and vagina. Where the tumors
abut the mucosa of an organ, they form a submucosal zone
221. The only reliable means of diagnosing acute of hypercellularity called the cambium layer.
cardiac rejection before substantial myocardial
damage has occurred is 223. The gold standard for diagnosing the Cystic
1. Echocardiography fibrosis is
2. ECG 1. Sequencing the CFTR gene
3. Endomyocardial biopsy 2. Sweat chloride test
4. Ultrasound 3. Positive newborn screening test result
4. Nasal transepithelial potential difference
Ans: 3
Endomyocardial biopsy Ans: 1
Ans: Of the major complications, allograft rejection is the Sequencing the CFTR gene
primary problem requiring surveillance; scheduled In most cases, the diagnosis of cystic fibrosis is based on
endomyocardial biopsy is the only reliable means of persistently elevated sweat electrolyte concentrations
diagnosing acute cardiac rejection before substantial (often the mother makes the diagnosis by recognizing her
myocardial damage has occurred and at a stage that is infant's abnormally salty sweat), characteristic clinical
reversible in the majority of instances. Rejection is findings (sinopulmonary disease and gastrointestinal
characterized by interstitial lymphocytic inflammation manifestations), an abnormal newborn screening test, or a
that, in its more advanced stages, damages adjacent family history. A minority of patients with cystic fibrosis,
myocytes; the histology resembles myocarditis. When especially those with at least one mild CFTR mutation,
myocardial injury is not extensive, the rejection episode may have a normal or near-normal sweat test (<60 mM/L).
is usually either self-limited or successfully reversed by Measurement of nasal transepithelial potential difference
increased immunosuppressive therapy. Advanced in vivo can be a useful adjunct test under these
rejection may be irreversible and fatal if it is not promptly circumstances; individuals with cystic fibrosis
treated. demonstrate a significantly more negative baseline nasal
potential difference than controls. Sequencing the CFTR
222. The most common type of Rhabdomyosarcoma is gene is, of course, the gold standard for diagnosis of
1. Embryonal cystic fibrosis. Therefore, in patients with suggestive
2. Alveolar clinical findings or family history (or both), genetic
3. Pleomorphic analysis may be warranted.
4. Inflammatory
224. The origin of Klatskin tumors is from
Ans: 1 1. Liver
Embryonal 2. Gall bladder
Rhabdomyosarcoma, the most common soft-tissue 3. Confluence of cystic duct and hepatic duct
sarcoma of childhood and adolescence, usually appears 4. Confluence of right and left hepatic ducts
before age 20. It may arise in any anatomic location, but
most occur in the head and neck or genitourinary tract, Ans: 4. Confluence of right and left hepatic ducts
where there is little if any skeletal muscle as a normal
constituent. Rhabdomyosarcoma is histologically sub The tumours arising at the confluence of right and left
classified into embryonal, alveolar, and pleomorphic hepatic ducts at the liver hilus are called Klatskin tumours.
variants. The rhabdomyoblastthe diagnostic cell in all These tumours are notable for their slow growing
typescontains eccentric eosinophilic granular cytoplasm behaviour, marked sclerosing characteristics, and
rich in thick and thin filaments. Rhabdomyoblasts may be infrequent occurrence of distal metastasis.
round or elongate; the latter are known as tadpole or strap
cells, and may contain cross-striations visible by light 225. Flame cells are seen in
microscopy. 1. Multiple myeloma
Embryonal rhabdomyosarcoma is the most common type, 2. Malignant melanoma
accounting for 60% of rhabdomyosarcomas. It includes the 3. Nodular colloid goiter
sarcoma botryoides as well as spindle cell and anaplastic 4. Hashimoto thyroiditis

47
4. < 50
Ans: 1. Multiple myeloma
Multiple myeloma usually presents as destructive plasma Ans: 4
cell tumors (plasmacytomas) involving the axial skeleton. < 50 cells/ mm3
Relatively normal-appearing plasma cells, plasma blasts The CD4+ T-cell count can define the risk of infection with
with vesicular nuclear chromatin and a prominent single specific organisms. As a rule of thumb, bacterial and
nucleolus, or bizarre, multinucleated cells may tubercular infections are more likely at higher CD4+
predominate. Other cytologic variants stem from the counts (>200 cells/mm3). Pneumocystis pneumonia
dysregulated synthesis and secretion of Ig, which often usually strikes at CD4+ counts below 200 cells/mm3, while
leads to intracellular accumulation of intact or partially cytomegalovirus and Mycobacterium avium complex
degraded protein. Such variants include flame cells with infections are uncommon until the very late stages of
fiery red cytoplasm, Mott cells with multiple grapelike immunosuppression (CD4+ counts <50 cells/mm3).
cytoplasmic droplets, and cells containing a variety of
other inclusions, including fibrils, crystalline rods, and 228. 20 year old patient presents with peripheral
globules. The globular inclusions are referred to as Russell neuropathy, ataxia, distal sensory loss and muscle
bodies (if cytoplasmic) or Dutcher bodies (if nuclear). In weakness, hearing loss , and ichthyosis. CSF shows
advanced disease, plasma cell infiltrates may be present in elevated protein level. The most likely diagnosis is
the spleen, liver, kidneys, lungs, lymph nodes, and other 1. Fabry disease
soft tissues. 2. Refsum disease
3. Adrenoleukodystrophy
226. Szary-Lutzner cells are seen in 4. CMT4
1. ALCL
2. Hairy cell leukemia Ans. 2
3. Mycosis fungoides Refsum disease can manifest in infancy to early adulthood
4. PTCL with the classic tetrad of (1) peripheral neuropathy, (2)
retinitis pigmentosa, (3) cerebellar ataxia, and (4) elevated
Ans: 3. Mycosis fungoides CSF protein concentration. Most affected individuals
Cutaneous T cell lymphoma (CTCL) represents a spectrum develop progressive distal sensory loss and weakness in
of lymphoproliferative disorders affecting the skin. Two the legs leading to footdrop by their 20s. Subsequently, the
different clinical types of malignant T-cell disorders were proximal leg and arm muscles may become weak. Patients
originally recognized: mycosis fungoides, a chronic may also develop sensorineural hearing loss, cardiac
proliferative process; and a more aggressive nodular conduction abnormalities, ichthyosis, and anosmia.
eruptive variant, mycosis fungoides d'emble. Lesions of Serum phytanic acid levels are elevated. Sensory and
mycosis fungoides usually involve truncal areas and motor NCS reveal reduced amplitudes, prolonged
include scaly, red-brown patches; raised, scaling plaques latencies, and slowed conduction velocities. Nerve biopsy
that may even be confused with psoriasis; and fungating demonstrates a loss of myelinated nerve fibers, with
nodules. In some individuals, seeding of the blood by remaining axons often thinly myelinated and associated
malignant T cells is accompanied by diffuse erythema and with onion bulb formation.
scaling of the entire body surface (erythroderma), a
condition known as Szary syndrome. The histologic 229. 20 year old patient presents with hypokalemia,
hallmark of CTCL of the mycosis fungoides type is the alkalosis with normal blood pressure and no edema.
presence of the Szary-Lutzner cells. These are T-helper the diagnosis is
cells (CD4+) that characteristically form band-like 1. Bartter syndrome
aggregates within the superficial dermis ( Fig. 25-19B ) 2. Liddle syndrome
and invade the epidermis as single cells and small clusters 3. Glucocorticoids remediable aldosteronism
(Pautrier microabscesses). These cells have markedly in 4. Apparent mineralocorticoid excess syndrome
folded nuclear membranes, imparting a hyperconvoluted
or cerebriform contour. Although patches and plaques Ans. 1
show pronounced epidermal infiltration by Szary-Lutzner It is characterized by hypokalemia , alkalosis, high rennin
cells (epidermotropism), in more advanced nodular level and hypercalciuria , but with normal blood pressure
lesions the malignant T cells often lose this epidermotropic and no edema.
tendency, grow deeply into the dermis, and eventually This is due to mutation Na-k-cl co transporter gene ,
spread systemically. usually begins in childhood.

227. Mycobacterium avium complex infections are 230. Combination chemotherapy ( ABV )with
common when the CD4+ counts in cells/mm3 are daunorubicin , bleomycin , vincristine is used in
1. 200 1. Kaposi sarcoma
2. < 200 2. Rhabdomyosarcoma
3. >50 3. Wilms tumour

48
4. Seminoma Hormone level in sick euthyroid syndrome usually of
1. Low T3 , nomal T4 , TSH low T3 syndrome
Ans. 1 2. Low T3, low T4 , normal TSH low T4 syndrome- severe
It is used in Kaposis sarcoma. Intraleisional danorubicin cases
and vinblastine, radiation treatment, placitaxel, interferon
alpha are also used. 236. CNS lupus manifestations include all of the
following except
231. Pallas sign seen in 1. Seizures
1. Sarcoidosis 2. Psychosis
2. Emphysema 3. Encephalopathy
3. Pulmonary embolism 4. Hemorrhage
4. Myeloma
Ans. 4.
Ans. 3
Chest radiograph signs in PE are , 237. False regarding Peutz Jeghers syndrome
Pallas sign enlarged right descending pulmonary artery 1. Autosomal recessive
Hamptons hump- wedge shaped density above diaphragm 2. Presence of hamartomatous polyps
Westermark sign- focal oligemia. 3. Presence of cutaneous pigmentation
4. Risk of developing breast, lung , ovarian cancer .
232. Bridge therapy is employed in
1. Mononeuritis multiplex Ans. 1.
2. Rheumatoid arthritis It is a autosomal dominant condition.
3. Osteoarthritis Other options are right.
4. Caplan syndrome
238. Bone marrow aspirate of a patient shows
Ans. 2 homogenous, medium sized neoplastic cells with many
In RA , acute symptoms is controlled with steroids , mitotic figures producing starry sky appearance.
followed by treatment with DMARD. This is called bridge Macrophages with pale cytoplasm are scattered. The
therapy most likely pathological identification
1. Lymphoplasmacytic lymphoma
233. In Glanzmann thrombasthenia, the defect is 2. Burkitt lymphoma
1. Decreased platelets 3. Monocytoid B cell lymphoma
2. Defective adhesion 4. Adult T cell lymphoma
3. Defective aggregation
4. Defective secretion of granules Ans. 2
Given picture describes so called starry sky pattern of
Ans. 3 Burkitt lymphoma.
It is due to defective aggregation because of surface
deficiency of GP II b- IIIa. 239. Usual presentation of cauda equina syndrome is
1. Sudden and unilateral
234. QT prolongation seen in all of the following 2. Gradual and bilateral
conditions except 3. Sudden and bilateral
1. Digitalis toxicity 4. Gradual and unilateral
2. Hypokalemia
3. Hypocalcemia Ans. 4. ref Harrison 17 th ed p no 2589.
4. Jervell syndrome Conus medullaris syndrome is usually sudden and bilateral
whereas cauda equina is grdual and unilateral.
Ans. 1 ,
Short QT interval seen in 240. All of the following are associated with thymoma
1. Digoxin toxicity except
2. Hypercalcemia. 1. Graves disease
2. Polymyositis
235. True regarding sick euthyroid syndrome 3. Hypergammaglobulinemia
1. Normal TSH, T3 , T4 level 4. Pure red cell aplasia.
2. Normal TSH, low T3, normal T4 level
3. High TSH, low T3, low T4 level Ans. 3.
4. High TSH, low T3, high T4 level. It is characterized by , hypogammaglobulinemia not
hypergammaglobulinemia.
Ans. 2.

49
241. Ascorbic acid is required for synthesis of Ans. is 2 i.e., Highly structured cell with organized cell
1. Phenylserine organelles
2. Homeserine Prokaryotesand Eukaryotes
3. Hydeoxylysine All cells fall into one of the two major classification-
4. Selenocysteine 1)Prokaryotes or 2) Eukaryotes
1. Prokaryotes
Ans. is 3 i.e., Hydroxylysine Prokaryotes are evolutionally ancient, i.e. they arc first
Hydroxylation of proline and lysine residue takes living organisms to evolve,
place during post-translational modification in rough They were here first and for billions of years were the
ER. only form of life.
The enzyme catalyzing the reactions are prolyl Prokaryotic cells are not carrying of life processes by
hydroxylase (for proline) and lysyl hydroxylase (for themselves. The reason is that they don V have alt the
lysine), organelles (e.g. nucleus, mitochondria, golgi apparatus
Both these enzymes are dioxygenases using molecular etc.) required to carry out life processes. So, these
oxygen (02) and cofactor for both these enzymes is cells are dependent on other cells in order to form
vitamin what is necessary for the life processes.
C (ascorbic acid). -Ketoglutatrate is a coreductant, Prokaryotes divide by binary fission.
which is oxidized to succinate. Prokaryotes are always single celled except when they
These hydroxylated amino acids arc not reprsented in the exist in colonics.
genetic code (i.e. there is no t-RNA for these^), therefore Examples- bacteria (including chlamydia &
they have to be synthesized post-translationally from mycoplasma). Blue green algaeu, Archea
proline and lysine residue in the polypeptides. 2. Eukarvotes
Eukaryotes are more complex, evolving from a
242. Catalase positive coagulase negative beta prokaryote like precdecessor.
hemolytic bacteria- Eukaryotic cells contain all the organelles required to
1. Strep pyogens earn out the processes oflife.
2. Staph aureus Eukaryotes may be single celled or multicelled.
3. Coagulase negative staph The most noticable feature that differentiates
4. Enterococci eukaryotes from prokaryotes is the presence of
nucleus.
Ans. is 3' i.e., Coagulase negative staph Examples-Fungi, Algae other than blue green algae,
protozoa.
Gram positive cocci
244. Fluidity buffer present in cell membrane is
Catalase test 1. Stearic acid
2. Palmitic acid
(+ve) (-ve) 3. Cholesterol
Staphylococci Streptococci 4. Linoleic acid

Coagulase protein A Ans. 3


(+ve) (-ve)
S. aureus Coagulase negative Membrane fluidity
Staphylococci Membrane fluidity changes with temperature
Transition temperature (Tm)

Novobiocin susceptible

(+ve) (- ve)
S. epidermidis S. saprophytics
S. hemolylicus

243. Eukaryotes are different in causing infection


because-
1. Divide by biniary fission
2. Highly structured cell with organized cell organelles
3. Dont have all organelles
4. Ecolutionally ancient

50
246. Which part of bacteria is most antigenic-
1. Protein coat
2. Lipopolysaccharide
3. Nucleic acid
4. Lipids

Ans. is '1' i.e., Protein coat


Proteins are most immunogenic, while lipids & nucleic
acids arc least immunogenic.
Polysaccharides (carbohydrates) are less
immunogenic than protein antigens, but are more
antigenic than lipids &nucleic acids.
Determinants of antigenicity
A number of properties of antigen determine its
antigenicity (production of antibody or induction of cell
mediated immunity):
1. Chemical nature
More is the Tm,less is the fluidity
Cholesterol-Fluidity buffer Proteins > Polysaccharide > lipids and nucleic
acids.
Saturated FAs All proteins except gelatin arc immunogenic.
increase Tm and decrease fluidity 2. Size
Unsaturated FAs High molecular weight particles > low molecular
decrease Tm and increase fluidity weight particles.
3. Foreign ness
245. Virus quantification is done by In general, molecules recognized as "self are not
1. Egg inoculation immunogenic; for immunogenicity, molecules
2. Hemadsorption must be recognized as "nonself".
3. Plaque assay 4. Susceptibility to tissue enzymes
4. Electron microscopy Only substances which are metabolized and are
susceptible to the action of tissue enzymes behave
Ans. is 3' i.e., Plaque assay as antigens.
Plaque assay and Pock assay are quantitative infective 5. Route of administration
assays. Parenteral > oral
Viral assays
Virus content of a specimen can be assayed in two ways - 247. Ehrlichia chaffeensis is causative agent of
1- With reference to total virus particles - 1. Human monocytic ehrlichiosis
Electron microscopy 2. Human granulocytic ehrlichiosis
Hemagglutination 3. Glandular fever
2- With reference to the infectious virions only 4. None
A) Quantal Assays
indicates the presence or absence of infectious virus Ans. is 1 i.e., HME
e.g. Ehrlichiae
1. Death of the experimental animal These are gram negative, obligate intracellular
2. Production of heamagglutinin in allantoic fluid bacteria which have affinity towards blood cells,
3. Appearance of cytopathological effects on ceil They grow within phagosomes of phagocytes as
cultures mulberry-like clusters called morula.
B) Quantitative infective assay They are tick-borne.
It measures the exact no. of infectious particles eg. Three human infections caused by this group of
a) Plaque assay on monolayer cell culture organisms have been identified :-
Each infectious viral particle gives rise to a 1. Human monocytic ehrlichiosis (HME)
localized focus of infected cells that can be seen It is caused by E. chaffeensis, and transmitted by
with the naked eye. Such foci are known as Amblyomma ticks.
plaques. Deer and rodents are believed to be reservoir
Each plaque indicates an infectious virus. hosts.
b) Pock Assay on chick embryo CAM (Chorioallantoic There is leukopenia, thrombocytopenia and
membrane) elevated liver enzymes.
Viruses that form pocks on CAM can be assayed by 2) Human granulocytic ehrlichiosis (HGE)
counting the no. of pocks.
51
It is caused by E. equi (E. phagocytophila), and 3. Campylobacter jejuni.
transmitted by ixodes ticks.
Deer, cattle and sheep are suspected reservoirs. 249. Immunoglobulin changes in variable region-
There is leucopenia and thrombocytopenia. 1. Idiotype
3) Glandular like fever 2. Isotype
It is caused by Ehrlichia sennetsu 3. Allotype
No arthopod vector has been identified and 4. Epitope
infection is caused by ingestion offish carrying
infected flukes. Ans. is 1' i.e., Idiotype
Theres is lymphoid hyperplasia with atypical Variability of amino acid sequences at the variable
lymphocytosis. regions form the specific antigen binding site, i.e.
o Doxycvcline is recommended for treatment of idiotype.
ehrlichiosis. The aminoacid sequences of the variable regions arc
not uniformly variable along their length, but consist
248. Invasive infection caused by all except of relatively invariable and some highly variable
1. Shigella zones. The highly variable zones are involved with the
2. Salmonella formation of antigen binding sites. The sites on the
3. V. Cholerae hyervariable regions that make actual contact with the
4. Yersinia epitopes are called "complementarity determining
regions (CDRs)
Ans. is '3' i.e., V. cholerae "The idiotype is defined as the specific region of the
o Based on the depth of intestinal invasion, there are fab portion of the Ig molecule to which antigen binds".
different clinical manifestations of infection with The infinite range of the antibody specificity of
organisms causing diarrhea; immunoglobulin depends on the variability of the
1. No cell invesion (noninflammatory) aminoacid sequences at the variable regions of the H
o The bacteria bind to intestinal epithelial cells but and L chains which form the antigen combining sites.
do not enter the cell.
o Diarrhea is caused by the release of entcrotoxins. 250. Which part of chlorine disinfection is most
o Watery diarrhea with no fecal leukocytes and no effective?
systemic symptoms (e.g. fever) occurs, 1. Hypochlorous acid
Organisms are: 2. Hypochlorousion
1. V. cholerae 6. Giardia 3. Hypochlorite ion
2. ETEC 7. Cryptosporidia 4. None
3. Staph, aureus 8. Microsporidia
4. Clostridium perfringens 9. Rotavirus Ans. is '1' i.e., Hypochlorous acid
5. B. cereus 10. Adenovirus Chlorination
2. Invasion of the intestinal epithelial cells o Chlorination is one of the greatest advances in water
(Inflammatory) purification.
The organisms have virulence factors that allow o Chlorine kills pathogenic bacteria, but has no effect on
binding and invasion into cells, spores, certain viruses (e.g., polio, viral hepatitis) and
Toxins may be then released that destroy the cell. cyst of E. histolytica, in usual doses.
The cell penetration results in a systemic immune Mechanism of action
response with fecal leukocytes as well as fever, o When chlorine is added to water, there is formation of
The cell death results in RBC leakage into the stool hypochloric and hypochlorous acid
(dysentery). o The hypochloric acid is neutralized by the alkanity of
Organisms are: the water.
1. Shigella 6. Yersinia enterocolitica o The hypochlorous acid ionizes to form hydrogen ions
2. Salmonella Sp. 7. C. difficale and hypochlorite ions.
3. Campylobacter jejuni 8. E. histolytica o The disinfecting action of chlorine is mainly due to the
4. EHEC & ETEC hypochlorous acid and to a small extent due to the
5. Vibrio parahemolyticus hypochlorite ions.
3. Invasion of lymph nodes and blood stream o The hypochlorous acid is the most effective form of
(Penetrating) chlorine for water disinfection, it is 70-80 times more
Along with abdominal pain and diarrhea containing effective than hypochlorite ion.
WBCs and RBCs, this deeper invasion results in Method of Chlorination
systemic symptoms of fever, headache and o For disinfecting large bodies of water, following methods
leukocytosis, of chlorination are available:
Examples are 1. Salmonella typhi, 1. Chlorine gas First choice
2. Yersinia enterocolitica, and
52
2. Chloramine o The vaccine is best used in interepidemic period and
3. Perchloron or high test hypochlorite (HTH) should be offered to the most vulnerable and high risk
group.
251. Cell organelle affected in Wolmans Disease is o Currently, the three types of JE vaccines are in use:
1. Peroxisome (i). The mouse brain derived killed vaccine, which is
2. Lysosome based on either the Nakayama or beijing strains of
3. Liposome JE virus.
4. Dyctyosome (ii). The cell culture derived inactivated vaccine based
on the Beijing P-3 strain.
Ans. 2 (iii). The cell culture - derived live attenuated vaccine
Characteristics of Lysosomes based on the SA 14-14-2 strain.
1. Discovered by Christian de Duve o When immunizing children 1 -3 years of age the
2. Contains acid hydrolases,acid lipases,collagenase mouse brain derived vaccine provides adequate
3. ATP-dependent proton pumps in the membrane protection throughoutchildhood following 2 primary
maintain an acidic environment (pH 4.05.0) doses 2 weeks apart and boosters after 1 year and
subsequently at 3 yearlyintervals until the age of 10-
Lysosomal Diseases 15 years.
Tay-Sachs disease o Protective immunity develops in about one month
Hexosamidase A time (30 days) after the second dose.
Gauchers disease o Equally good childhood protection is obtained by a
Beta galactocerebrosidase single dose of the cell culture based live attenuated
I-cell disease vaccinefollowed by a single booster given at an
Absent mannose-6-phosphate tag interval of about one year,
o For travellers aged more than one year, visiting rural
252. Amplifier for Japanese encephalitis areas of endemic countries for at least 2 weeks, the
1. Horse establishedcurrent practice is to administer 3 primary
2. Pigs doses at days 0, 7 and 28.
3. Dogs o Alternatively 2 primary doses can be given 4 weeks
4. Monkey apart.
When continued protection is required booster should be
Ans. is '2 i.e., Pigs given after one year and then every 3 years.
In Japanese encephalitis
Pigs Amplifiers. 254. Age limit for HDI?
Cattle and buffalos Mosquito attractants 1. 25 to 85 yrs
Man Incidental dead-end host 2. 25 to 50 yrs
Herons Reservoir hosts 3. 15 to 45 yrs
4. 50 to 75 yrs
253. SA-14-14-2 -
1. Diploid cell inactivated Ans. is '1' i.e., 25 to 85 yrs
2. Killed vaccine o According to previous editions of Park (21st/e) the
3. Life long immunity minimum and maximum established values for life
4. Primary immunization 2 doses expectancy atbirth was 25 years and 85 years,
respectively.
Ans. is 3 i.e., Life long immunity (most probably) o However, Now the minimum established value is 20
o SA-14-142 is a cell-culture-derived live attenuated years and maximum value is the highest observed
vaccine. value during time series, i.e. the country which has the
o Primary immunization is given by a single dose highest value is taken as reference for maximum value.
followed by a booster after 1 years. For example themaximum life expectancy is observed
o I am not sure about option c, because no textbook has in Japan (83.4 in 2011). Thus 83.4 years is taken as
mentioned whether, SA-14-14-2 vaccine provides life upper (maximum limit).
longimmunity or not. But, almost all live o That means minimum value is established at 20 years
attenuatedvaccines provide life long immunity, and maximum value is established at 83.4 years.
therefore it may provide life longimmunity. Human development index (HDI)
"Protection for several years may be achieved with a single o HDI is a composite index combining indicators
dose of this vaccine _Vaccines: Expert Consult representing three dimensions
JE vaccine (i). Longevity (life expectancy at birth)
o Vaccination of population at risk has been (ii). Knowledge (Adult literacy rate and mean years
recommended. ofschooling, i.e. Gross enrolement ratio)

53
(iii). Income (real GDP per capita in purchasing power Primary- Post central gyrus
parity in US dollars) Association area- parietal lobe
o The HDI is an index used to rank countries by level of Secondary area- sylvian fissure
human development.
o According to HDI countries are divided - size of the cortical receiving area for impulses from a
(i). Developed countries (High HDI 0.8) USA, particular part of the body is proportional to the use of the
Canada, Norway part
(ii). Developing countries (medium HDI 0.5 -
0.79)India Thumb and lips-maximum representation
(iii). Underdeveloped countries (Low HDI Vertical columnar arrangement
0.5)Seiera, Ethopia
257. All of the following are direct DNA interacting
Following information has been added in 22nd/e of cytotoxic agents except
Park 1. Melphalan
o Components of human development index have 2. Hydroxyurea
changed. Now it includes expected years of schooling 3. Carmustine
(insteadof adult literacy rate) and GNI per capita 4. Ifosfamide
(instead of GDP per capita).
Components of HDI Ans , 2
Recent (22nd/e of Park) Old (Previous editions)
1)Longevity:- Life 1) Longevity: - Life All other agents are alkylating agents , whereas
expectancy at birth expectancy at birth hydroxyurea is an anti metabolite acts at S phase of cell
2) Knowledge 2) Knowledge cycle indirectly affect DNA molecules.
Mean years of schooling Adult literacy rate
Expected years of 3) Income: - GDP per capita 258. The least common cause of neonatal mortality in
schooling India is -
3) Income: - GNI per capita 1. Prematurity
2. Congenital malformations
255. Stimulation of which brain area can modulate the 3. Birth asphyxia
sensation of pain? 4. LBW
1. Superior olivary complex
2. Locus ceruleus Ans. is '2' i.e., Congenital malformations
3. Periaquaductal gray o All the given options are causes of neonatal death in
4. Amygdala India; However, amongst the given options
congentialanomaly is the least common cause.
Ans. 3 o Causes of neonatal mortality in decreasing order of
frequency:
Endogenous analgesic system Prematurity & LBW > Severe infections > Birth
A. periaqueductal gray (PAG)-projects to nucleus raph asphixia > Congential anomalies > Neonatal
magnus and rostral ventromedial medulla tetanus > Diarrheal disease > other.
B. project to the dorsal horn of the spinal cord- release
serotonin and norepinephrine 259. 1ST sign of anterior uveitis -
C. Locus coerulus-Nor epinephrine 1. Keratic precipitate
D. Stress induced analgesia: 2. Aqueous flare
release of endogenous cannabinoids such as 2- 3. Hypopyon
arachidonoylglycerol (2AG) and anandamide acts on 4. Miosis
CB1(Euphoria) and CB 2 (Pain)receptors
Ans. is '2 i.e., Aqueous flare
256. Which body part has maximum representation in o Earliest sign of anterior uveitis Aqueous flare.
CNS? o Pathognomonic sign Keratic precipitate.
1. Urinary Bladder Clinical features of acute anterior uveitis
2. Lower limb o Acute anterior uveitis is the most common form of
3. Abdomen uveits, accounting for 60-70% of cases. It is
4. Thumb characterized bysudden onset and duration less than 3
months.Presentation is typical with sudden onset of
Ans. 4 unilateral pain,photophobia, redness, tacrimation and
blepherospasm. Vision may be normal initially.
However, later there may bedisturbance in vision due
Somatosensory cortical areas

54
to ciliary spasm (induced myopia), corneal haze (due 5. Posterior staphyloma : - Occurs at posterior pole and
to edema & KPs) and aqueous turbidity. is lined internally by choroid. Degenerative high
o External examination shows following signs:- axial myopia is the most common cause.
(i). Circumcorneal (ciliary) congestion Which has a
violaceous hue. 261. Which is not a cataract surgery-
(ii). Corneal edema, posterior corneal opacities. 1. Lensectomy
(iii). Keratic precipitates (KPs):- are proteinaceous cellular 2. Goniotomy
deposits at the back of cornea. 3. Phacoemulsification
(iv). Anterior chamber signs:- Aqueous cells. Aqueous flare, 4. IOL
hypopyon, hyphacmia (in hemorrhagic uveitis), deep
anterior chamber(lf posterior synachiac occurs). Ans. is 2' i.e., Goniotomy
(v). Ins signs:- Blurred & indistinct iris, i.e. muddy iris, o Goniotomy is done for glaucoma.
instead of being clear and sharply defined. o Extracapsular cataract extraction (ECCE) by
(vi). Pupillary signs: - Narrow (miotic) pupil, irregular phacoemulsification with posterior chamber IOL is the
pupil due to segmental posterior syncchiac which procedure ofchoice for cataract.
givesfestooned appearance (festooned pupil) when o Lensectomy is done in congenital cataract.
dilated with atropine, sluggish pupillary reaction, Operative techniques available
ectropionpupillae (evertion of pupillary margins), o Following operative procedures are available for
occlusio pupillac (due to complete occlusion of pupil cataract extraction :-
by organized exudate). 1)Extracapsular cataract extraction (ECCE)
The nucleus, cortex and central part of the anterior
260. Posterior staphyloma, most common cause capsule are removed leaving behind intact posterior
1. Trauma capsule, peripheral part of anterior capsule and zonules.
2. Myopia Presently, extracapsular cataract extraction
3. Iridocyclitis with posterior chamber lens implantation (ECCE with PCL)
4. Glaucoma is the surgery of choice for almost all types of
cataract. The only absolute contraindication for ECCE is
Ans. is 2' i.e., Myopia subluxation and dislocation of lens.
Staphyloma Techniques for ECCE are:-
o Staphyloma is an abnormal protrusion of uveal tissue i) Conventional ECCE
through a weak and thin portion of cornea or sclera ii) Manual small incision cataract surgery
o So, a staphyloma is lined internally by uveal tissue and iii) Phacoemulsification
externally by weak cornea or sclera. 2) Intracapsular cataract extraction (ICCE)
o Staphyloma is divided anatomically into : - The lens is removed in toto as one single piece, i.e., the
1. Anterior staphyloma: - Protrusion and adhesion of nucleus and cortex are removed with the
iris to ectatic cornea. The most common cause is a capsule after breaking the zonules. There is no support left
sloughing corneal ulcer which perforates and heals for posterior chamber IOL, therefore, only
with the formation of pseudocornea by the anterior chamber IOL can be implanted. At present, the
organization of exudates and laying down of fibrous only indication of ICCE is markedly subluxated or
tissue. It is lined internally by iris. dislocated lens in which zonules arc already broken, so
2. Intercalary staphyloma: - It occurs at the limbus. It is ECCE cannot be done. ICCE is contraindicated
lined internally by the root of iris and the anterior in young patients (< 40 years) because zonules are too
most portion of the ciliary body. The causes are strong.
perforating injuries to limbus, marginal corneal ulcer, 3) Phacoemulsification
anterior scterilis, Scleromalacia perforans, o It is nothing else but an advancement in the method of
Complicated cataract surgery, secondary angle closure doing ECCE. Here the nucleus is converted into
glaucoma. pulp or emulsified using high frequency sound waves,
3. Ciliary staphyloma : This affects the ciliary zone that and then sucked out of the eye through a small
includes the region upto 8 mm behind the limbus. (3.2 mm) incision. A special foldable IOL is then
The ciliary body is incarcerated in the region of scleral inserted into the posterior chamber through the
ectasia. Causes arc Developmental glaucoma. Primary same incision. ECCE by phacoemulsification with
or secondary glaucoma end stage, scleritis, trauma to foldable posterior chamber IOL is the procedure
ciliary region. ofchoice for cataract.
4. Equatorial staphyloma : - This occurs at the o The steps in phacoemulsification include :(i)
equatorial region of the eye with incarceration of the Corneoscleral incision, (ii) Continuous curvilinear
choroid. capsulorrhexis, (iii) Hydrodissection and
Causes are scleritis, degenerative myopia and chronic hydrodelineation (iv) Emulsification and aspiration of
uncontrolled glaucoma. nucleusand then cortex, (v) Foldable IOL implantation
in posterior chamber,

55
o Recovery with phacoemulsification is fastest as 3. Retrobulbar neuritis
incision is very small and no sutures are taken. 4. Keratoconus

Ans. is 3' i.e., Retrobulbar neuritis


262. Vitamin B12 deficiency causes - o Swinging flash light test is used for relative afferent
1. Centrocaecal scotoma pathway dfect (RAPD), which is most characteristic of
2. Binasal hemianopia lesions in the optic nerve,
3. Constriction of peripheral field o Thus it is positive in retrobulbar neuritis.
4. Bitemporal hemianopia Marcus Gunn pupil
o Marcus Gunn pupil is seen in relative afferent pathway
Ans. is '1' i.e., Centrocaecal scotoma defect (RAPD)
o Vitamin B12 deficiency causes optic neuritis, o An incomplete optic nerve lesion or retinal disease
o Most common visual field defect in optic neuritis is cause it (in contrast to amaurotic light reflex, where
central or centrocaecal scotoma. there is total afferent pathway defect). It is best tested
Important facts by swinging flash light test.
o Earliest visual field defect in primary open angle o To perform this test, B bright flash light is shone to
glaucoma Isopeteric contraction. one pupil and constriction of that pupil is noted
o Earliest clinically significant visual field defect in o Then the flashlight is quickly moved to the
primary open angle glaucoma Paracentral scotoma. contralateral pupil and the response in that pupil is
o Visual field defect in rheugmatogenous retinal noted,
detachment Loss ofperipheral field. o This swinging to and fro of the flashlight is repeated
o Visual field defect in retinitis pigmentosa Tubular several times while the pupillary response is observed,
vision. o Normally, both pupils constrict equally and the pupil
o Most common visual field defect in optic neuritis to whom light is transferred remains tightly
Central or centrocaecal. constricted,
o Usual field defect in papilledema Enlargement of o In the presence of relative afferent pupillary defect in
blind spot and progressive contraction of visual field. one eye, both the pupil will dilate when the flash light
o Visual field defect in Lebers optic neuropathy is moved from the normal to the abnormal eye.
Centrocaecal. o This is a paradoxical resposne.
o Characteristic visual field defect in anterior ischemic o This is called Marcus Gunn pupil and is the earliest
optic neuropathy Altitudinal visual field defects. indicator of optic nerve disease even in the presence of
a normal visual acuity.
263. Intact cornea can be penetrated by - o Relative afferent pupillary- defect (RAPD) or Marcus
1. Gonococcus Gunn pupil is a reliable and objective sign of unilateral
2. Pseudomonas or assymmetric disease or any lesion in afferent
3. Streptococcus pupillary pathway, Le., retina, optic nerve, optic
4. Pneumococcus chiasma, optic tract or midbrain (pretectal nucleus).
o However, RAPD is most characteristic of lesions in the
Ans. is 1' i.e., Gonococcus optic nerve.
o A few species of bacteria are capable of penetrating an
intact corneal epithelium - 265. Not a symptom of angle closure glaucoma -
i) Neisseria gonorrhoea 1. Blurring of vision
ii) Coryncbacterium diphtheriea 2. Coloured Halos
iii) Haemophilus agyptus 3. Metamorphosia
iv) Neisseria meningitidis 4. Headache
v) Listeria species
o Bacteria which cause corneal ulcer after epithelial Ans. is 3' i.e., Metamorphosis
injuries are - o Transient blurring of vision and headache is seen in
i) Pseudomonas stage of constant instability.
ii) Pneumococcus o Coloured halos are seen in acute congestive stage.
iii) Staphylococcus aureus Clinical features of primary angle clasure glaucoma
iv) Morexella o The disease is generally bilateral, though assymetrical.
v) Streptococcus epidermids A number of clinical subtypes have been described,
vi) Enterobacterias whichmay or may not show a stepwise progression in
a given eye.
1) Latent PACG (PACG suspect or Prodromal stage):- These
264. Swinging light test is positive in - eye arc identified by shallow anterior chamber
1. Conjunctivitis associated with an occludable angle. There are no
2. Glaucoma symptoms except for occasionally complain of halos

56
around light and transient haziness of vision at this stage. 266. Eale's disease is -
The only gonioscopic finding is very narrow angle 1. Retinal hemorrhage
(<20).IOP is normal. Two provocative test have been 2. Vitreous hemorrhage
described i) Prone-darkroom provocative test: - It is 3. Conjunctival hemorrhage
best physiological test. Patient is made to lie prone in a 4. Choroidal hemorrhage
darkroom. There is an increase in IOP > 8 mm Hg.
ii) Mydriatic / cycloplegic provocation test; Is not Ans. is 2 i.e., Vitreous hemorrhage
preferred because it is not physiological. A mydriaticis Eale's disease
used to dilate the pupil and there is > K mm rise in IOP o Eale's disese is an idiopathic inflammatory venous
2) Subacute / intermittent PACG (Stage of constant occlusion that primarily affects the peripheral retina of
instability):- Patients have shallow anterior chamber with young adult (20-30 yrs) male.
occludable angle and physiological factors like dim light or o It is characterized by recurrent bilateral vitreous
watching a movie in a darkened cinema hallprecipitate hemorrhage; therefore, also referred to as primary
pupillary block and rise in IOP. The patient complains of vitreous hemorrhage.
transient blurring of vision,coloured halos arround light, o Disease is bilateral and patients commonly present
headache, browache and eyeache. On examination the eye with sudden onset of floaters (black spots) in front of
is white hut notcongested. Provocative tests arc positive. the eyeor sudden painless loss of vision due to
Between these recurrent attacks, eye is free of symptoms vitreous hemorrhage,
and onlyshows signs of a narrow angle recess, clumping of o The hemorrhage clears up spontarcously, but
pigment in the angle and occasional peripheral synechia. recurrences are very common.
3) Acute primary angle glaucoma (acute congestive o On examination, retinal changes include perivascular
glaucoma or acute angle closure glaucoma): - Is caused by phlebitis, perivascular exudates, superficial (flame
sudden occlusion of the entire angle with a resultant acute shaped)hemorrhages, peripheral nonperfusion
rise of IOP to extremely high level. The attack ischemia and neovascularization
usually does not terminate of its own. The clinical features o Medical treatment in the form of oral corticosteroid is
are:- the treatment of choice.
A) Symptoms: Sudden onset of very severe pain in the o A course of ATT has also been recommended for
eye which radiates along the branches of 5thnerve and is selected cases as many workers consider Eale's
frequently associated with nausea, vomiting and disease to be ahypersensitivity reaction to tubercular
prostrations. Patient developsrapidly progressive loss of protein, a Laser photocoagulation is indicated for
vision, redness, photophobia and lacrimation. Coloured retinal neovascularization.
halos are seen. o Vitroretinal surgery is required for non - resolving
B) Signs vitreous hemorrhage and retinal detachment, a
o Lid edema Complications of hales disease include proliferative
o Conjunctiva is chemosed and congested (both vitreorelinopathy, fractional retinal detachment,
conjunctival & ciliary vessels are congested) a Cornea rubeosis iridis and neovascular glaucoma.
is oedematous and insensitive
o Anterior chamber is very shallow & with aqueous flare 267. Neurotransmitter employed by rods and cones is
o Angle of anterior chamber is closed (on gonioscopy) 1. Dopamine
o Iris is discoloured 2. Serotonin
o Pupil is semidilated, vertically oval and fixed. It is non 3. Glutamate
reactive to both light and accommodation. 4. Nitric oxide
o IOP is markedly raised (40- 70 mm Hg)
o Optic disc is oedematous & hypercmic Ans. 3
4) Chronic primary angle closure glaucoma:- It results
from gradual synechial closure of the angle ofanterior
chamber. There is constantly elevated IOP, decreased
visual aucity, Visula field defect (like POAG).
The eyeball is white and not congested.
5) Absolute primary angle closure glaucoma (absolute
glaucoma):- If chronic phase is untreated it gradually
passes into the final phase of absolute glaucoma where the
eye is painful, irritable and completely blind.
Onexamination there is perilimbal reddish blue zone,
caput medusae, hazy cornea with bullous keratopathy or
filamentry keratitis, atrophic iris, raised IOP, fixed dilated
pupil, glaucomatous optic disc atrophy andstony hard
eyeball.

57
Most common malignant tumor of cornea
Squamous cell carcinoma

269. A person with neck nude and B type


tympanogram
1. Acoustic neuroma
2. Nasopharyngeal CA
3. Angiofibroma
4. None

Ans. is 2' i.e., Nasopharyngeal CA


o Information in this question:
i) Cervical adenopathy (Seek node)Most common
presentation of nasopharyngeal carcinoma,
ii) Type B tympanogram:- Indicates serous otitis
media, which may be a complication of
nasopharyngeal carcinoma due to eustachian tube
blockade,
o Thus, among the given options, nasopharyngeal
carcinoma is the answer of choice.
268. MC orbital tumor - Clinical features of nasopharyngeal carcinoma
1. Nerve sheath tumor o Symptomatology of nasopharyngeal carcinoma is
2. Hemangioma divided into five main groups:-
3. Lymphoma 1) Nasal symptoms
4. Meningioma o Nasal obstruction, nasal discharge, denasal speech
(rhinolalia clausa), epistaxis.
Ans. is '2' i.e., Hemangioma 2) Otologic symptoms
o Most common orbital tumor Cavernous o Otologic symptoms are due to obstruction of
hemangioma. eustachian tube.
o Most common malignant orbital tumor Lymphoma. o This results in serous or suppurative otitis media.
IMPORTANT FACTS Which causes conductive deafness.
Most common intraocular tumor in adults is o Presence of unilateral serous otitis media in adult
metastasis. Metastasis is particularly common from should raise the suspicion of nasopharyngeal
carcinoma of breast and lung. carcinoma a Tinnitus and dizziness may occur.
Most common primary intraocular tumor in adults is 3) Ophthalomoneurological symptoms
uveal melanoma. Most of the uveal malignant o These symptoms occur due to spread of tumor to the
melanoma arise in choroid. surrounding regions.
Most common primary intraocular tumor is children is o Squint and diplopia due to involvement of VI th cranial
retinoblastoma nerve.
Most common orbital tumors in adults are benign o Ophthalmoplagia due to involvement of IVrd, IVth& Vim
cranial nerve.
vascular tumors Cavernous hemangioma
o Facial pain X reduced corneal sensations due to Vth
Most common malignant orbital tumor in adult
nerve involvement a Exophthalmos and blindness due
lymphoma
to direct extension into the orbit.
Most common orbital tumors in children are benign o Jugular foramen syndrome (IXth, Xth, XIth cranial nerve
tumors Dermoid cyst capillary hemangioma involvement) due to pressure by enlarged lateral
Most common malignant orbital tumor in children retropharyngeal lymph nodes.
rhabdomyosarcoma o Cranial nerve XIIth involvement due to extension of
Overall most common primary malignant tumor of eye growth to hypoglossal canal.
is malignant melanoma followed by retinoblastoma o Horners syndrome due involvement of cervical
Most common malignant eyelid tumor Basal cell sympathetic chain.
carcinoma 4) Neck mass:- It is the most common presentation
Most common epithelial tumor of lacrimal gland (60-90%)
Pleomorphic adenoma (benign mixed tumor) o Due to cervical lymphadenopathy.
Overall most common tumor of lacrimal gland 5) Distant metastasis
Lymphoid tumour and inflammatory pseudo - tumors o Involve bone, lung, liver and other sites.
Most common malignant tumor of conjunctiva
Squamous cell carcinoma 270. Impedence matching occurs d/t -

58
1. Difference of surface area of tympanic membrane Ans. 3
and foot plate Parvovirus B 19 is the causative agent of erythema
2. Semicircular canal fluid infectiosum (fifth disease). It is associated with transient
3. Utricle & Saccule aplastic crisis in persons with hereditary hemolytic
4. None anemia. In adults, it is also associated with polyarthralgia.

Ans. is 1 i.e., Difference of surface area of tympanic 272. Transport medium for Streptococcus will be
membrane and foot plate 1. Pikes medium
o Impedence matching or transformer action of middle 2. Stuarts medium
ear is due to:- 3. VR medium
i) Hydraulic action of tympanic membrane due to 4. Selenite F medium
difference in surface area of TM and stapes footplate.
ii) Lever action of ossicles. Ans. 1
Impedence matching Transport media: used to transport the specimen from
o The middle ear couples sound signals from the ear collection place to laboratory. Transport media mainly
canal to the cochlea primarily through the action of the used to avoid multiplication of commensals in the
tympanic membrane and ossicular chain. Sound particular sample.
energy that reaches the tympanic membrane is
conducted across ossicular chain and is concentrated o Stuart (charcoal swab), Amies medium -Gonococcus
in the stapes footplate. The transformer action of o Pikes - Streptococcus
middle ear is accomplished by :- o Venkatraman Ramakrishnan medium, autoclaved sea
1) Hydraulic action of tympanic membrane water, Alkaline salt water- Vibrio
o The main transformer action of middle ear comes from o Buffered glycerol saline Typhoid bacilli
the tympanic membrane. The ratio of functioning area o Carry Blair medium Enteric pathogen
(affective vibrating area) to the area of stapes
footplate is 17: 1, and this is the mechanical advantage
provided 273. The effect of progesterone on lipids
by the tympanic membrane which increases the sound 1. Lowers LDL, increases HDL
pressure at the footplate. You should keep in mind that 2. Lowers LDL and HDL
though the ratio of functional area of tympanic 3. Lowers HDL and increases LDL
membrane to footplate is 17: 1, ratio of total area of 4. Increases LDL and HDL
tympanic membrane is 21 : 1 (As the effective
vibratory area of tympanic membrane is 2/3rdof total Ans. 3.
area, mechanical advantage is provided only by
2/3rdof the total area).
2) Lever action of the ossicles While part of the administered progestogen is metabolized
o Ossicular lever ratio is 1.3: 1 as handle of malleus is to oestrogen, it is also partly metabolized to testosterone.
1.3 times longer than the long process of the incus. If administered to a patient during pregnancy, some
This provides mechanical advantage of 1.3. progestogens have virilizing effect upon a female fetus.
o So, the combined (total) mechanical advantage Lipid metabolism decreases HDL but increases low-density
(product of area ratio and lever action of ossicles is) - lipoprotein. Thus, it is harmful to the heart.
17*1.3 = 22:1. It improves the immune response
3) Curved membrane effect
o Movements of tympanic membrane are more at 274. Identify the procedure
the periphery than at the centre where malleus
handle is attached. This too provides some leverage.

271. Which of the following viruses causes an acute


febrile rash and produces disease in
immunocompetent children but has been associated
with transient aplastic crises in persons with sickle
cell disease?
1. Rubeola
2. Varicella-zoster
3. Parvovirus
4. Rubella

59
1. Marshall burns
2. Mauriceau smellie veit technique
3. Leopold maneuvre
4. Ritgen maneuver

Ans. 3.

277. The first sign of puberty in girls is


1. Menarche
1. Amniocentesis 2. Breast budding
2. Hegars sign elicitation 3. Growth spurt
3. Bimanual compression 4. Pubic and axillary hair growth
4. Ritgen maneuver
Ans. 2.
Ans. 2.
Tanner and Marshall described five stages of pubertal
275. All are true about PCOS except changes in a normal girl. These are in the following
1. Persistently elevated LH sequence (Figure 4.3):
2. Increased LH/FSH ratio Physical growth and weight gain.
3. Increased DHEAS Development of secondary sex orgAns. A.nd breast
4. Increased prolactin development-around 10-12 years.
Pubic and axillary hair growth a year later.
Ans. None Development of ovaries and the genital organs.
Growth spurt and menstruation.
Endocrinological changes are as follows:
1. Oestrone/E2 level rises. 278. Identify 2 in the picture
2. LH level is raised over 10 IU/ml.
FSH level remains normal, but FSH/LH ratio falls.
3. SHBG level falls due to hyperandrogenism.
4. Testosterone and epiandrostenedione levels rise.
Testosterone >2 ng/ml, free T >2.2 pg/ml (Normal level
0.2-0.8 ng/ml)
Normal androstenedione level is 1.3-1.5 ng/ml.
DHEA > 700 ng/ml suggests adrenal tumour.
5. Prolactin is mildly raised in 15% cases.
6. Fasting insulin is more than 10 mIU/l in PCOS.
7. Thyroid function tests may be abnormal
(hypothyroidism).
8. 17 -hydroxyprogesterone in the follicular phase > 300
ng/dl suggests adrenal hyperplasia due to 21 hydroxylase
deficiency.

276. Identify the maneuver 1. Obstetric conjugate


2. Diagonal conjugate
3. Anatomic conjugate
4. None

Ans. 1.

279. The pH of vagina in adults is


60
1. 3.5- 4.5 DeLancey introduced three level systems of support.
2. 4.5-5.5 Level I-Uterosacral and cardinal ligaments support the
3. 5.5- 6.5 uterus and vaginal vault.
4. 6.5- 7.5 Level II-Pelvic fascias and paracolpos which connects the
vagina to the white line on the lateral pelvic wall through
Ans. 2. arcus tendinous.
Level III-Levator ani muscle supports the lower one-third
The vaginal secretion is small in amount in healthy of vagina.
women and consists of white coagulated material. When it
is examined under the microscope, squamous cells which 282. Identify the type of placenta
have been shed from the vaginal epithelium and
Doderlein's bacilli alone are found. Doderlein's bacillus is
a large gram-positive rod-shaped organism, which grows
anaerobically on acid media. The vaginal secretion is acidic
due to the presence of lactic acid, and this acidity inhibits
the growth of pathogenic organisms. The pH of the vagina
averages about 4.5 during reproductive life. The acidity,
which is undoubtedly oestrogen-dependent, falls after
menopause to neutral or even alkaline. Before puberty, the
pH is about 7. This high pH before puberty and after
menopause explains the tendency for the development of
mixed organism infections in these age groups.
1. Succinate lobe
2. Battledore placenta
280. Comment on this CTG 3. Velamentous placenta
4. Marginal cord insertion

Ans. 3

283. Minimum effective dose of oestrogen in OCPs is


1. 20 mcg
2. 30 mcg
3. 40 mcg
4. 50 mcg

Ans. 1.

1. Normal Combined oral pills


2. Early deceleration Combined oral pills contain a mixture of ethinyloestradiol
3. Late deceleration (EE2) in a dose of 20 to 30 g and an orally active
4. Variable deceleration progestogen which is a 19-norsteroid. Mala-D contains 0.5
mg of d-norgestrel and Mala-N contains 1 mg
Ans. 4. norethisterone; these are available free of cost in India.
The tablets are taken starting on the second day of the
281. All are primary supports of uterus except cycle for 21 days. A new course of tablets should be
1. Transcervical ligament commenced 7 days after the cessation of the previous
2. Pubocervical ligament course. They should be taken at a fixed time of the day,
3. Uterosacral preferably after a meal.
4. Broad ligament

Ans. 4. 284. Identify the pelvis type A


The supports of the uterus and the bladder are seen to be
triradiate condensation of endopelvic fascia:
1. The anterior spoke is the pubocervical fascia or so-
called pubocervical ligament.
2. The lateral spoke is Mackenrodt's ligament.
3. The posterior spoke is the uterosacral ligament. 1. Android
2. Gynecoid
SUPPORTS OF THE GENITAL TRACT 3. Anthropoid
4. Platypelloid
61
luteum. The menstruation comes on time, is prolonged but
Ans. 2. not heavy. Progestogen can suppress the bleeding, but
needs to be taken on tapering dose for 20 days to complete
285. Best Hormonal contraceptive for sickle cell the cycle.
anemia patient is
1. Ocp 289. In ca ovary inguinal lymph nodes are involved in
2. Pop stage
3. Cu-T 1. Stage II
4. None 2. Stage III a
3. Stage IIIc
Ans. 2. POP progesterone only pill 4. Stage IVb

POP is contraindicated in liver diseases, vascular disorders Ans. 4.


and breast cancer. It is safe in sickle cell anaemia.
FIGO revised the staging of carcinoma ovary in January
286. Hypokalemic paralysis is side effect of 2014 and according to the new staging inguinal LN
1. Gossypol involvement is stage IV b.
2. DMPA
3. Testosterone enanthate 290. Mc benign epithelial ovarian tumor is
4. Cyproterone acetate 1. Serous cystadenoma
2. Mucinous cystadenoma
Ans. 1. Gossypol 3. Brenner tumor
Its use as a male contraceptive was discovered in China. 4. Ovarian fibroma
Gossypol is a yellow pigment isolated from cottonseed oil.
It is administered orally 10 to 20 mg daily for 3 months Ans. 1
and thereafter 20 mg twice weekly. The action is directly
on the seminiferous tubules inhibiting spermatogenesis Serous cystadenoma and cystadenocarcinoma
without altering FSH and LH levels. The side effects such as Serous cystadenoma and cystadenocarcinoma are amongst
weakness, hypokalaemia and permanent sterility in 20% the most common of cystic ovarian neoplasms, accounting
cases limit its use. about 50% of all ovarian tumours; of these, 60-70% are
benign, 15% borderline and 20-25% are malignant.
287. Red degeneration of uterine fibroid is mc in
1. First trimester 291. Most common CNS manifestation of HIV Infection
2. Second trimester is:
3. Third trimester 1. Acute Meningitis
4. Peurperium 2. Encephalopathy
3. Vacuolar myelopathy
Ans. 2. 4. Dementia
Red degeneration
This complication of uterine myomas develops most Ans. 4
frequently during pregnancy, although it is not rare in HIV Encephalopathy also called HIV - Associated
cases of painful myomas in women over the age of 40. The dementia or AIDS dementia.
myoma becomes tense and tender and causes severe This is generally a late complication of HIV infection, it
abdominal pain with constitutional upset and fever. The can be seen patients with CD4 count >350 cells.
tumour itself assumes a peculiar purple red colour and
HIV encephalopathy is a sub-cortical dementia while
develops a fishy odour. If the tumour is carefully examined,
. Alzheimer's is a cortical dementia
some of the large veins in the capsule and the small vessels
in the substance of the tumour will be found thrombosed Alzheimer's disease is the most common cause of
dementia in elderly.
288. Halban disease is due to Most important risk factor for Alzheimer'^ disease is
1. Persistent corpus luteum age.
2. Deficient corpus luteum
3. Persistent trophoblast 292. All are the risk factors for coronary artery disease
4. Deficient trophoblast except:
1. LDL
Ans. 1. 2. HDL
3. Smoking
Irregular shedding (Halban's disease) 4. Hypertension
It is rare and self-limited. It is due to persistent corpus

62
Ans. 2 systemic sclerosis and serologically by high titres of
Some important risk factors for Atherosclerosis: antibodies to RNP Particle containing UI RNP. Two factors
Fixed are important in lending distinctiveness to MCTD are:
Family history The paucity of renal disease.
Age Extremely good response to corticosteroid.
Sex (male) Modifiable Some important points about connective tissue
Smoking disorder:
Haemostatic variables Antinuclear antibodies in autoimmune disease.
Hypertension Anti double stranded DNA-S (more specific
Sedentary life styles than anti SM antibody).
Lipid disorders Anti Histone Antibodies-Z)rz/g induced lupus
Obesity erythrocytosis
DM Anti sm antibody>SLE
Diet Anti DNA topoisornerasc antibody
sclerosis
Atherosclerotic vascular disease may manifest as
Antibodies of Ribonucleoprotein Antigen
coronary heart disease, cerebrovascular disease or
containing ULRNP> Mixed connective tissue
peripheral vascular disease. Occult coronary artery
disorder.
disease is common in those who present with other Antimitochondrial antibody>Primary biliary
forms of atherosclerotic vascular disease. So risk factors cirrhosis.
for atherosclerosis is also are the risk factors for CAD.
295. Microangiopathic Hemolytic anemia seen in:
293. Lung carcinoma most common associated with: 1. HUS
1. Asbestosis 2. HTN
2. Silicosis 3. SLE
3. Byssinosis 4. All
4. Beryllium
Ans. 4
Ans. 1 Micro-angiopathic Hemolytic anemia occurs when
Remember some important points. red blood cells are forccd to squeeze through
Silicosiscurrently the most prevalent chronic abnormally narrowed small vessels. Narrowing is
occupational disease in the world most often caused by fibrin deposition in
Silicosis is associated with an increased association with DIC. Other causes of micro-
susceptibility to Tuberculosis. The radio logical angiopathic hemolytic anemia are:
featureEGG SHELL calcification in the hilar Malignant hypertension
lymphnodes. SLE
Berylliosis cause an interstitial granulomatous Thrombotic Thrombocytopenic purpura.
disease similar to Sarcoidosis. HUS
The most dangerous particles to cause Disseminated cancer.
Pneumoconiosis from 1 to 5 micrometer in The common feature among all these disorders is a Micro
diameter because they may reach in the vascular Lesion that cause mechanical injury to
terminal small airways, air sacs and settle in circulating Red blood cells. In blood smear in the form of
their lining. red cell fragments (schisocytes) burr cells, helmet cells
Cotton dust is associated with Byssinosis. and triangle cells are seen. Some important Point:
Sugarcane dust is associated with Bagassois Most common cause of Anemia in world is Iron
deficient Anemia.
294. Mixed connective tissue disease includes all the Most sensitive and specific test for diagnosis of from
following except: deficiency is serum ferritin levels.
1. Polymyositis Hyper segmented neutrophilis are a feature of
2. RA megaloblastic anemia and not iron deficient anemia.
3. Osteoarthritis Cause of microcytic hypochromic anemia Mnemonics:
4. Systemic sclerosis Tiny CLIPS ' Iron deficient Anemia
Thaiassaemia
Ans. 3 Sideroblastic anemia,
Mixed connective tissue diseases (MCTD) are sometimes
Lead poisoning
used to describe the disease seen in a group of patient Anemia of chronic disease.
who are identified clinically by the coexistence of features Pyridoxin deficit anemia.
suggestive of SLE. Polymyositis, Rheumatoid arthritis and
63
296. "a" wave in JVP indicates:
1. Atrial systole Ans. D
2. Ventricular systole Renal failure consequent on liver failure can occur
3. Atrial relaxation
in cirrhosis. The kidney's they are intrinsically
4. Tricuspid regurgitation
normal and renal failure is thought to result from
Ans. A altered systemic blood flow including diminished
A wave: Arc positive waves due to distention produced renal blood flow:
by right atrial contraction Large a waves: Indicate atrium It occurs in advanced cirrhosis at most always
is contracting against increased resistance tricuspid and ascites
stenosis, pulmonary stenosis, pulmonary hypertension. Absence of proteinuria or abnormal urinary
Cannon S waves sediment.
Regularly: During junctional rhythm. Urine sodium excretion below lOmmol/day.
Irregularly: complete Heart Block. Serum Na t- concentration < 130 m Eq/L?
Absent a waves: Atrial Fibrillation Urinary osmolality greaterthan
plasma osmolality.
297. Anti mitochondrial Ab seen typically in: Urine volume < 500 ml/day.
1. Primary Biliary cirrhosis
2. Hepatitis B
3. Hepatitis C 299. In infant LRTI is MC caused by:
4. Hepatitis A 1. Streptococci
2. RSV Viruses
Ans. 1 3. H. Influenza
Primary Biliary cirrhosis is a disease characterized by 4. Mycoplasma
chronic inflammation and fibrous obliteration of
intrahepatic bile duclioles due to an unknown (primary) Ans. B
stimulus. Etiology RSV virus is m/c cause of bronchiolitis in Infants. Doc-
PBC is believed to due to an autoimmune etiology as it is Ribavirin. Epiglottis is most commonly caused by H.
often associated with other auto immune disorders such Influenzae.
as Some Important points:
CRCST syndrome: Calcinosis, Raynaud's Encephalitis>is the Inflammation of the brain
phenomena, Esophageal dysmotility, Sclero- parenchyma Meningitises an acute purulent infection with
dactily. Telangiectasia in the subarachnoid space. The meninges, the
Sicca syndrome subarachnoid space and brain parenchyma are all
Autoimmune thyroiditis frequently involved in the inflammatory reaction with
Renal tubular acidosis meningoencephalitis. The most common cause of sporadic
Anti mitochondrial antibody (IgG is positive) viral encephalitis HSV -I
The most common cause of Epidemic viral
Clinical feature
encephalitis- Arbovirus
Most patients arc asymptomatic.
Japanese encephalitis virus is an arbovirus is one
Pruritis is commonest/earliest symptom
of the most common causes of encephalitis
Jaundice
outbreak.
Deficiency of fat soluble vitamin due to
The most common cause of viral meningitis is ->
malabsorption leads to:
Enterovirus
Vit D deficiency Bone painOsteomyelitis
Vit K deficiency- bleeding bruising
Vit A deficiency~Night Blindness IXanthelisma 300. Megaloblastic anemia is caused by:
and Xanthomas arc due to elevation of serum 1. Diphyllobothriumlatum
lipids.Lab finding 2. H-NaNa
Increased Serum alkaline phosphates.Positive 3. Taeniasaginata
mitochondrial antibody sensitive and specific. 4. Echinococcusgranulosis

298. Hepatorenal syndrome characterised by all Ans. 1


except: Habitat: Adult worm lives in small intestine (ileum or
1. No proteinuria jejunum) of man D. Latum is the longest tapeworm found
2. Normal intrinsic'kidney. in man. Life cycle:
3. Liver failure
4. Urine osmolality less than plasma osmolality
64
The worm passes its life cycle in one definitive host and
Two intermediate host. Definitive host: Man is the main
definitive host Intermediate host:
First intermediate host: Small copepods mainly of genra
Diaptomus and Cyclops Second intermediate host:
Freshwater Fishes. Humans become infected when they
eat undercooked raw or lightly salted meat or roe from
infected freshwater fishes.
Pathogenicity:
Most human infections arc caused by only one tape worm
and cause no or very vague ill effects.
Patient may develop fatigue, weakness, diarrhea and
numbness of extremities. Some Patient develops
mechanical obstruction. In few cases pernicious anemia
may develop due to B12 deficiency. D. Latum has been
shown to absorb as much as 80-100% of a single oral
dose of vit- B12 thereby competing with the host for
this important vitamin. Treatment of choice:
Niclosamide and praziquantel Important:
Vit B12 Absorb in the ILEUM.
Iron is absorbed in the Duodenum.

65

Vous aimerez peut-être aussi